WAE Flashcards

1
Q

A died, survived by his wife (B) and daughter(C). His validly executed will includes only the following gifts:

(a) My house to my wife

(b) £50,000 to my daughter

A’s succession estate comprises the following property (all solely owned):

A house

Bank accounts containing £100,000

Personal chattels

Which of the following correctly explains how A’s estate will be distributed?

A is testate because he has a validly executed will. The house will pass to B. C will receive the contents of the bank accounts. The personal chattels will be divided equally between B and C.

A is partially intestate because his will does not dispose of his entire estate. The house will pass to B. C will receive the contents of the bank accounts. The personal chattels will be dealt with under the intestacy rules.

A is intestate because his will does not dispose of his entire estate. All his property will be dealt with under the intestacy rules.

A is intestate because his will does not dispose of his entire estate. The house will pass to B. C will receive £50,000. The remaining £50,000 and A’s personal chattels will be dealt with under the intestacy rules.

A is partially intestate because his will does not dispose of his entire estate. The house will pass to B. C will receive £50,000. The remaining £50,000 and A’s personal chattels will be dealt with under the intestacy rules.

A

A is partially intestate because his will does not dispose of his entire estate. The house will pass to B. C will receive £50,000. The remaining £50,000 and A’s personal chattels will be dealt with under the intestacy rules.

Correct: Any property covered by the will is dealt with in accordance with that will. The remainder passes in accordance with the intestacy rules.

How well did you know this?
1
Not at all
2
3
4
5
Perfectly
2
Q

Which of the following would be included in a deceased person’s succession estate?

A life interest in a trust, a share of a house owned as tenants in common and cash held in a bank account.

A vested remainder interest in a trust, a share of a house owned as joint tenants and cash held in a bank account.

A remainder interest in a trust which is contingent on the deceased surviving the life tenant, a share of a house owned as tenants in common and a life assurance policy written in trust.

A vested remainder interest in a trust, a share of a house owned as tenants in common and cash held in a bank account.

A remainder interest in a will trust which is contingent on the deceased surviving the life tenant, a share of a house owned as tenants in common and a life assurance policy not written in trust.

A

A vested remainder interest in a trust, a share of a house owned as tenants in common and cash held in a bank account.

Correct: All of these assets would be included in the succession estate. Note that the position would be different if the deceased held a life interest in the will trust or the house was held as joint tenants.

How well did you know this?
1
Not at all
2
3
4
5
Perfectly
3
Q

A died yesterday. His estate consisted of the following assets (all solely owned unless otherwise stated:

House £400,000 Antiques worth £180,000 (owned with B as joint tenants). Bank accounts £25,000 Life Assurance Policy £50,000 (not written in trust)

A was also the life tenant of C’s estate. There was £100,000 in the trust fund at A’s death. B is the remainderman.

What is the value of A’s succession estate?

£655,000

£475,000

£575,000

£565,000

£425,000

A

£475,000

Correct. A was the sole owner of the house, the bank accounts and the life policy proceeds (which were not written in trust). The antiques pass to B via survivorship.

How well did you know this?
1
Not at all
2
3
4
5
Perfectly
4
Q

A person dies intestate, survived only by their long-term boyfriend and two brothers (both under 18). How will their estate be distributed?

The whole estate passes to the boyfriend providing he survives the deceased by 28 days. The brothers are not entitled to anything because they are not issue.

The boyfriend is entitled to the deceased’s personal chattels and a statutory legacy of £270,000. The residue is shared equally between the boyfriend and the brothers.

The estate is shared equally between the boyfriend and the brothers.

The estate will be held on the statutory trusts equally for the two brothers (whose interests are contingent upon reaching 18 or marrying earlier). The boyfriend is not entitled to anything.

The boyfriend is entitled to the deceased’s personal chattels, a statutory legacy of £270,000 and half the residue. The remaining half share is held on the statutory trusts for the brothers.

A

The estate will be held on the statutory trusts equally for the two brothers (whose interests are contingent upon reaching 18 or marrying earlier). The boyfriend is not entitled to anything.

Correct: The brothers are the first people entitled in the statutory order. The boyfriend has no entitlement under the intestacy rules.

How well did you know this?
1
Not at all
2
3
4
5
Perfectly
5
Q

A dies intestate, leaving no spouse and three unmarried children, B (18), C (15) and D (14). How will A’s estate be distributed?

The whole estate passes to B providing they survive A by 28 days. C and D are not entitled to anything because they have not reached the age of 18 or married by the date of A’s death.

The estate will be shared equally and absolutely by B, C and D, providing they survive A by 28 days.

B is entitled to A’s personal chattels, a statutory legacy of £270,000 and half the residue. The remaining half of the residue is held on the statutory trusts for B, C and D.

The estate will be held on the statutory trusts equally for B, C and D, providing they survive A by 28 days. B has a vested interest. C and D’s interests are contingent upon reaching 18 or marrying earlier.

The estate will be held on the statutory trusts equally for B, C and D. B has a vested interest. C and D’s interests are contingent upon reaching 18 or marrying earlier.

A

The estate will be held on the statutory trusts equally for B, C and D. B has a vested interest. C and D’s interests are contingent upon reaching 18 or marrying earlier.

Correct: A left issue but no spouse, so they inherit the estate on the statutory trusts. The contingency limb has already been satisfied in respect of B but not C or D.

How well did you know this?
1
Not at all
2
3
4
5
Perfectly
6
Q

A person dies intestate and is survived by his mother, his spouse and his son. His estate amounts to £790,000 cash, plus personal chattels worth £10,000.

Which of the following correctly states the value of the spouse’s entitlement (providing that they survive by 28 days)?

£395,000

£530,000

£540,000

£405,000

£270,000

A

£540,000

Correct. The spouse is absolutely entitled to the personal chattels (worth £10,000), a statutory legacy of £270,000 and half the £520,000 residue.

How well did you know this?
1
Not at all
2
3
4
5
Perfectly
7
Q

A woman executes her will in accordance with the requirements of s.9 Wills Act 1837. There is no doubt that she has testamentary capacity. The will gives the whole of the woman’s estate to her son and nothing to her daughter.

The daughter tells you that her brother encouraged their mother to make a will leaving everything to him and nothing to the daughter. The son confirms that he did ask his mother to make a will leaving everything to him because his sister is “not to be trusted with money”.

The daughter tells you that her mother would never have agreed or intended to give everything to her son.

Is the woman’s will valid?

Yes, provided that the son can produce evidence that his sister is reckless with money.

No. The son encouraged the testator to make a will that favoured him over his sister and therefore the will does not reflect her true intention.

Yes, unless the daughter can provide evidence that her brother placed her mother under so much pressure that she only signed the will because she felt forced to and then when she signed it the woman did not actually want her son to inherit the whole of her estate.

No, unless the son can provide evidence that his mother intended for him to inherit the whole of her estate.

Yes. The woman had testamentary capacity and executed the will properly. This raises a presumption that she had the requisite intention to make this will.

A

Yes, unless the daughter can provide evidence that her brother placed her mother under so much pressure that she only signed the will because she felt forced to and then when she signed it the woman did not actually want her son to inherit the whole of her estate.

W1 assessment MCQ
(forgot to add in explanation for correct answer)

How well did you know this?
1
Not at all
2
3
4
5
Perfectly
8
Q

A woman died intestate two months ago and her estate was distributed between her three adult children. The woman had never married or entered a civil partnership. On the date of her death she had been living with her long-term partner and his son (aged 15) for the previous 10 months. The woman provided financial maintenance for both her partner and her step-son throughout the period they all lived together.

Who is entitled to bring a claim against the woman’s estate under the Inheritance (Provision for Family and Dependants) Act 1975?

The woman’s step-child only.

No one is entitled to bring a claim because the woman did not leave a will.

The woman’s children and step-child.

The woman’s partner only.

The woman’s partner, children and step-child.

A

The woman’s partner, children and step-child.

Correct. The woman’s partner could apply as a person maintained at the date of death under s.1(1)(e). However, he could not apply as a cohabitee under s.1(1A), as they were not living together for the two years immediately preceding her death.

The woman’s step-child could possibly apply as a person treated as a child of the deceased under s.1(1)(d), or if the facts did not support this, then as a person being maintained under s.1(1)(e).

The woman’s children could apply under s.1(1)(c). That they are adult children and have received her estate under intestacy does not preclude a claim but would affect the likelihood of them receiving any award.

How well did you know this?
1
Not at all
2
3
4
5
Perfectly
9
Q

A man died 8 months ago and the grant of probate was issued four months later. By his will the man left all of his estate to his son and nothing to his partner with whom he had been living for the last 3 years. The man’s partner wishes to bring a claim under the Inheritance (Provision for Family and Dependants) Act 1975 (the ‘Act’).

Is the man’s partner able to bring a claim under the Act?

No - because it has been less than 6 months since the date of the grant of probate.

Yes

No - because the man died more than 6 months ago.

Only if a court order granting permission for the deadline to be extended is obtained.

No - because they were not married to or in a civil partnership with the man.

A

Yes

Correct. The deadline for bringing a claim (without court permission) is 6 months from the date of the grant of representation. A claim can be brought under the Act before the grant is issued.

How well did you know this?
1
Not at all
2
3
4
5
Perfectly
10
Q

A new client aged 40 wishes to give instructions to make a will.

Which of the following most accurately describes best practice when taking the instructions?

Contact the client’s doctor for an opinion on whether the client has testamentary capacity to make a will.

Ask questions to establish if the client meets the requirements of the test in Parker v Felgate.

Ask the client if they have testamentary capacity to make a will.

Ask questions to establish if the client meets the requirements of the test in Banks v Goodfellow.

Ask questions to establish if the client meets the statutory test for capacity under the Mental Capacity Act 2005.

A

Ask questions to establish if the client meets the requirements of the test in Banks v Goodfellow.

This is best practice in a case where your client is young and healthy.

How well did you know this?
1
Not at all
2
3
4
5
Perfectly
11
Q

A woman died intestate a month ago. At the time of her death she was separated from her husband of 10 years. The woman and her husband had discussed getting divorced but no formal proceedings had begun. When she died, the woman was living with her son (aged 17 years) and her step-son (aged 18 years). The woman’s daughter (aged 25) lives with her husband and their new baby, the woman’s only grandchild.

Who is entitled to share in the distribution of the woman’s estate?

The husband, son, step-son and daughter only.

The husband, son and daughter only.

The son and daughter only.

The son and step-son only.

The husband, son, daughter and grandchild only.

A

The husband, son and daughter only.

Correct. Where a person dies intestate and is survived by a spouse and issue, only those relatives will take a share of the deceased’s estate. Here, the woman was survived by her husband (it does not matter that they were not living together), and her son (it does not matter how old her son is) and daughter (it makes no difference whether she is living with the deceased or not). The step-son has no entitlement to the estate. Nor does the grandchild if the parent is alive at the date of the intestate’s death.

How well did you know this?
1
Not at all
2
3
4
5
Perfectly
12
Q

A woman died intestate a month ago. She had never been married or in a civil partnership. The woman was living with her only grandchild (aged 1). The woman’s son, the grandchild’s father, died before the woman.

The woman’s brother (her only sibling), both of her parents, and her daughter-in-law (the wife of her son) are still alive.

Who is entitled to share in the distribution of the woman’s estate?

Her grandchild only.

Her parents only.

Her daughter-in-law only.

Her granddaughter and brother only.

Her daughter-in-law and grandchild only.

A

Her grandchild only.

Correct. Where a person dies intestate and is survived by issue but no spouse/civil partner, the issue will inherit the whole of the estate. If a child of the intestate dies before their parent, but leaving their own issue, the intestate’s grandchild will inherit the share of the estate their parent would have inherited.

The other options were incorrect because:

-the parents and siblings of an intestate will only inherit if the intestate is not survived by either spouse or issue.

-Where a person due to inherit dies before the intestate’s (here her son) then the son’s spouse would not be entitled to any share of the intestate’s estate.

How well did you know this?
1
Not at all
2
3
4
5
Perfectly
13
Q

A man died intestate a month ago. He had never been married or in a civil partnership and had never had any children. Both of the man’s parents died before him. The man was survived by his brother (aged 20), his sister (aged 16, and pregnant with her first child when her brother died), and his aunt (his mother’s sister). The man’s sister gave birth to a baby boy (the man’s nephew) 6 months after the man died. The man’s sister died during childbirth, aged 17.

Who is entitled to share in the distribution of the man’s estate?

His brother and his sister’s estate only.

His brother and his nephew only.

His aunt only.

His brother only.

His aunt, his brother and his nephew only.

A

His brother and his nephew only.

Correct. After spouse, issue and parents the next category of relative entitled to inherit are the siblings of the intestate. The siblings inherit on the terms of the statutory trust so in this case, the brother and sister are entitled to half of the estate each on the terms of the statutory trust i.e. contingent on reaching the age of 18.

The sister died after the intestate but before reaching the contingent age. This means her share never vested and is therefore not part of her estate. Any option suggesting the sister’s estate would benefit is therefore incorrect.

However, because the sister died leaving her own issue, the substitution limb of the statutory trust would apply. The share that would have passed to the man’s sister, passes instead to her children who are alive or en ventre sa mere (a French expression meaning “in its mother’s belly” which refers to a child conceived but not yet born) at the date of the intestate’s death, contingent upon their attaining the age of 18 or marrying earlier. Although the man’s nephew was not born until after his death, his sister was pregnant at this time and therefore the nephew satisfies the criteria assuming he reaches 18.

The man’s aunt would only inherit if the man had no siblings, or nieces /nephews.

How well did you know this?
1
Not at all
2
3
4
5
Perfectly
14
Q

A man died intestate a month ago. His estate comprised a property owned as joint tenants with his brother, a bank account in his sole name, personal possessions, cash in the house, and a discretionary lump sum payable under the terms of the man’s pension scheme (nominated in favour of the man’s brother).

Which of the following lists the assets that will be distributed in accordance with the intestacy rules?

The property, bank account and personal possessions only.

The property, bank account, personal possessions and cash only.

Bank account, personal possessions and cash only.

The property, bank account, personal possessions, cash, and the amount payable under the discretionary pension scheme.

Bank account, personal possessions, cash, and the amount payable under the discretionary pension scheme only.

A

Bank account, personal possessions and cash only.

Correct. Items owned as joint tenants will pass in accordance with the rules of survivorship. Therefore, his property will pass directly to his brother and not under the intestacy rules. The discretionary pension lump sum is payable in accordance with the nomination that was made, so will pass directly to his brother and not under the intestacy rules. The remainder of the estate assets are included in the deceased’s succession estate.

How well did you know this?
1
Not at all
2
3
4
5
Perfectly
15
Q

A man died intestate a month ago. He was survived by his civil partner and their son (aged 21). The man’s estate comprises a ½ share of a property owned as tenants in common with his civil partner (value of whole £500,000), a savings account (£40,000), a vehicle used solely for business purposes (£5,000) and personal possessions (£20,000).

Which of the following best describes what the man’s civil partner and son are entitled to under the intestacy rules?

The man’s civil partner will receive chattels (worth £20,000), a statutory legacy of £270,000, and half of the remainder. The man’s civil partner is entitled to have the property appropriated to them in addition to her entitlement under intestacy. The man’s son will receive a contingent interest in the other half of the remainder.

The man’s civil partner will receive chattels (worth £25,000), a statutory legacy of £270,000, and half of the remainder absolutely. The man’s son is entitled to a vested interest in the other half of the remainder.

The man’s civil partner will receive chattels (worth £20,000), a statutory legacy of £270,000, and half of the remainder. The man’s civil partner is entitled to have the property appropriated to them as part of their entitlement under the intestacy. The man’s son will receive a vested interest in the other half of the remainder.

The man’s civil partner will receive chattels (worth £25,000), a statutory legacy of £270,000, and half of the remainder. The man’s son will receive a contingent interest in the other half of the remainder.

The man’s civil partner will receive the entire estate passing under the intestacy rules. The man’s son will receive nothing.

A

The man’s civil partner will receive chattels (worth £20,000), a statutory legacy of £270,000, and half of the remainder. The man’s civil partner is entitled to have the property appropriated to them as part of their entitlement under the intestacy. The man’s son will receive a vested interest in the other half of the remainder.

Correct. The civil partner is entitled to receive chattels worth £20,000 (the personal possession but not the business assets), plus a statutory legacy of £270,000, and half of the remainder. As the half-share of the property falls within the estate passing under intestacy, the civil partner can request that the property is transferred to them as part of their entitlement (not in addition to). The son will receive the other half of the remainder on statutory trusts. As he is over the age of 18 his share will be vested.

How well did you know this?
1
Not at all
2
3
4
5
Perfectly
16
Q

If you are acting for a client who lacks testamentary capacity, should you take their instructions for preparing a will?

No

Yes

A

No

correct
If a client lacks capacity they cannot make a valid will and a solicitor should not accept their instructions to prepare one.

How well did you know this?
1
Not at all
2
3
4
5
Perfectly
17
Q

A client gives you instructions to prepare a new will which leaves the whole of their estate to their civil partner. You consider the test in Banks v Goodfellow and are certain that your client has testamentary capacity.

Five days after giving the instructions the client was involved in a car accident which left them disorientated and confused. You visit the client in hospital and bring with you the will you have prepared. The client remembers meeting you to give instructions for a will and is happy to sign the will you have prepared in accordance with those instructions. However, when talking with the client you have some doubt whether they still satisfy the test for capacity and their memory of your meeting and the points discussed seems a little unclear.

Which one of the following is correct?

The client must have capacity at the time of execution for the will to be valid.

The client could execute a valid will in the hospital but only if they satisfy the test for capacity in the Mental Capacity Act 2005.

The client could make a valid will in the hospital provided a doctor acts as a witness.

It is possible for the client to make a valid will in the hospital even if they lack capacity when it is executed because they had capacity when giving instructions and understand they are now signing a will for which they had previously given instruction.

It is possible for the client to make a valid will in the hospital even if they lack capacity when it is executed because they had capacity when giving instructions and the terms of the will are rational.

A

It is possible for the client to make a valid will in the hospital even if they lack capacity when it is executed because they had capacity when giving instructions and understand they are now signing a will for which they had previously given instruction.

Correct. The rule in Parker v Felgate is an exception to the requirement that the client has capacity at point of execution. Provided they had capacity when giving instructions, the will was prepared in accordance with those instructions and the testator understands they are signing a will for which they had previously given instructions they don’t need to satisfy the Banks v Goodfellow test at the point of signing.

The other answers were incorrect because:

  • The test in the Mental Capacity Act 2005 is not the test used for testamentary capacity
  • There is no requirement for a doctor to witness a will
  • Whether or not the will is rational is not a factor to take into account when applying the rule in Parker v Felgate
How well did you know this?
1
Not at all
2
3
4
5
Perfectly
18
Q

You have been instructed by a client whose elderly mother is in a nursing home and suffering from dementia. Your client’s mother currently lacks capacity which she will not regain.

Your client’s mother is a widow. Her current will divides her estate equally between her three children, including your client. The will was duly executed by the client’s mother and witnessed by two of her friends.

Your client is concerned that their mother may have lacked capacity when she made her will six months ago as your client was expecting to receive more than a 1/3 share of the estate.

Which one of the following most accurately describes the legal position should your client wish to challenge the validity of the will?

There is a presumption that your client’s mother lacked capacity when she made her will because of her current medical condition.

There is a presumption that your client’s mother lacked capacity when she made her will because she is elderly (unless the ‘golden rule’ was followed).

There will be a presumption that your client’s mother had capacity when she made her will unless your client has evidence to suggest otherwise.

If your client’s mother did not have capacity at the time she made the will it is invalid. As she will not regain capacity it is no longer possible for her to make a new will.

There is always a presumption in favour of capacity and therefore your client must prove their mother did not have capacity when she made her will.

A

There will be a presumption that your client’s mother had capacity when she made her will unless your client has evidence to suggest otherwise.

Correct.

Where the will on the face of it does not raise any concerns, because its terms are rational and it has been properly signed, then capacity is presumed. However, if evidence that raises doubt is provided however the presumption is rebutted and the burden of proof reverts to the propounder of the will to demonstrate the testator satisfied the Banks v Goodfellow test.

The other options were incorrect because:

  • If a testator lacks capacity it may be possible for a statutory will to be made for them
  • The age of the testator, and whether or not the golden-rule was followed, do not affect the general rule regarding presumption in favour of capacity where the will is rational and has been properly executed
  • It is not correct that the person challenging a will must prove the lack of capacity
  • The current medical condition is not a determining factor- the testator’s health at the time of execution is
How well did you know this?
1
Not at all
2
3
4
5
Perfectly
19
Q

You are reviewing two wills.

When the first will was signed the testator told the witnesses “I’ve not bothered to read anything the lawyer has sent me – it’s all a bit complicated to be honest, but I’m sure they’ve done the right thing”.

When the second will was executed the testator had hurt their hand and arranged for someone else to sign the will on their behalf.

Both wills were drafted by solicitors and duly executed with a standard form attestation clause. Both testators had testamentary capacity.

Assuming there is no other relevant information, which one of the following is correct with regards knowledge and approval of the wills?

Knowledge and approval will be presumed in respect of both wills because the testator had capacity when the wills were made.

Affidavit evidence that knowledge and approval were present at execution is required in respect of both wills.

Knowledge and approval will be presumed in respect of both wills because the testator had capacity when the wills were made and they were duly executed.

The first will is invalid due to lack of knowledge and approval. The second will is invalid because the attestation clause was not updated to make it clear the testator did not sign the will themselves.

The first will is invalid due to lack of knowledge and approval. Affidavit evidence that knowledge and approval were present at execution is required in respect of the second will.

A

The first will is invalid due to lack of knowledge and approval. Affidavit evidence that knowledge and approval were present at execution is required in respect of the second will.

Correct
A testator must intend to make the particular will they sign. This means they should know and approve of its contents and the contents of the will must have been read and understood. The first testator has not read or understood his will so did not have knowledge of its contents when it was signed.

The second will was signed by someone else on behalf of the testator. This means knowledge and approval will not be presumed. As the attestation clause was not amended to reflect this, affidavit evidence to prove knowledge and approval will be required. The other options were incorrect because: - facts suggest the first testator lacked knowledge and approval – so affidavit evidence will not help. - although testamentary capacity and due execution usually lead to a presumption of knowledge and approval, this does not apply where someone signs on behalf of a testator - an inadequate attestation clause does not render a will invalid

How well did you know this?
1
Not at all
2
3
4
5
Perfectly
20
Q

A will written in English contains the following attestation clause:

“Signed by the above named Testator in our joint presence and then by us in his after this document had been read to the Testator when the testator seemed thoroughly to understand and approve the contents”

In which circumstances should this form of attestation be used?

Where a testator is blind or illiterate.

Where a testator is blind, illiterate or does not understand English.

Where suspicious circumstances surround the execution of the will.

Where a testator is blind.

Where a testator is blind or is not signing the will on their own behalf.

A

Where a testator is blind or illiterate.

Correct
A blind or illiterate testator cannot read the will and therefore to raise a presumption that knowledge and approval was present it needs to be clear that the will was read out loud and that the testator understood what was said before it was signed. The other options were incorrect because: - A testator who does not understand English is not aided by the will being read out loud unless it is clear it was read in the testator’s preferred language by someone who was able to translate it - A testator who does not sign the will himself doesn’t necessarily need it to be read out loud, but the attestation clause should refer to the fact that he did not sign - Where there are suspicious circumstances this form of attestation is not helpful

How well did you know this?
1
Not at all
2
3
4
5
Perfectly
21
Q

A testator died recently. Three weeks before he died, the testator made a new will leaving all of his assets, apart from a small cash gift to a local charity, to the children of his current marriage. The testator’s previous will made provision for his child from an earlier marriage, but this child does not benefit under the current will.

You have been instructed by the child of the testator from the earlier marriage. Your client spoke to the testator a few days before he died. The testator explained that his wife “kept on at him all the time” to make a will in favour of their children only, and to keep her quiet he signed a new will that reflected what she wanted.

The testator had capacity and knew and approved of the terms of the new will when it was duly executed.

Which of the following is correct regarding a claim of undue influence?

A claim of undue influence will succeed if your client provides evidence that when the testator signed the will his true intention was not reflected in its terms.

A claim of undue influence will succeed if your client provides evidence that the testator’s spouse persuaded the testator to make the will which excluded your client.

A claim of undue influence is not possible because the testator had both capacity and knowledge and approval when they executed their will.

If a claim of undue influence succeeds the will would be invalid and the local charity cannot receive the cash gift.

A claim of undue influence is not possible because the testator’s spouse does not benefit under the new will.

A

A claim of undue influence will succeed if your client provides evidence that when the testator signed the will his true intention was not reflected in its terms.

Correct
Even where a testator has capacity and knowledge and approval, if the will was made as a result of undue influence it will not be valid because it does not reflect the testator’s true intention. The other options were incorrect because: - it is not unlawful to encourage someone to make a will or persuade them that certain provisions should be included. Undue influence occurs where a testator is coerced into making a will that is contrary to their true intention. - It is not a requirement that the person committing the wrong unduly influences the testator to make direct provision for themselves - A testator can know and approve of their will but still be acting as a result of undue influence - Where a testator has been unduly influenced in respect of a particular gift (here the gift to the children) it is still possible for the remainder of the will to take effect

How well did you know this?
1
Not at all
2
3
4
5
Perfectly
22
Q

Consider the following attestation clause:

“Signed by the above named [TESTATOR] in our joint presence“

Which one of the following most accurately summarises the effect of a will containing this clause?

If a will included this attestation clause it would be incomplete because there is no reference to the will having been read out loud to the testator.

If a will included this attestation clause there would be a presumption that the will was executed in accordance with the requirements of s 9 Wills Act 1837.

If a will included this attestation clause the will would not be valid.

If a will included this attestation clause proof of the testator’s intention would be required when seeking to admit the will to probate following the testator’s death.

If a will included this attestation clause, proof of due execution would be required when seeking to admit the will to probate following the testator’s death.

A

If a will included this attestation clause, proof of due execution would be required when seeking to admit the will to probate following the testator’s death.

Correct. A properly drafted attestation clause raises a presumption that the will was executed in accordance with the requirements of s 9 WA (a presumption of due execution). Therefore it should refer to the testator and the witnesses signing in the presence of each other. This clause makes no reference to the witnesses signing in the presence of the testator so is incomplete. However, there is no legal obligation to include an attestation clause, nor is any specific form of attestation required, so a poorly drafted clause does not invalidate the will. However, proof of due execution is required, usually an affidavit of due execution sworn by the witnesses.

The other answers were incorrect because:

  • A missing or poorly drafted attestation clause does not invalidate the will
  • A properly drafted attestation clause does not give rise to a presumption of the testator’s intent
  • The attestation clause would only mention the will have having been read out loud to a testator where the testator unable to read the will for himself
How well did you know this?
1
Not at all
2
3
4
5
Perfectly
23
Q

Your client arranges to sign their will at home and invites a married couple who live next door to act as witnesses. The three of them gather in the same room and the testator produces the will ready for signing. One of the witnesses leaves the room to take a phone call. Before they return, the testator signs the will in the presence of the other witness, who then signs the will in the presence of the testator. When the first witness returns to the room, the testator acknowledges their signature and the first witness then signs the will in the testator’s presence.

Has the will been executed in accordance with the formal requirements of s.9 Wills Act 1837?

The will has not been properly executed because the witnesses are married to each other.

The will has not been executed properly because the witnesses signed the will in the presence of the testator but not each other.

The will has not been properly executed because the testator and both witnesses must all be in the same room throughout the execution process.

The will has been properly executed.

The will has not been properly executed because one of the witnesses did not see the testator sign the will.

A

The will has been properly executed.

Correct. S 9 Wills Act 1837 requires the testator to sign the will (or acknowledge his signature) in the presence of two witnesses.

The other options were incorrect because:

  • S 9 permits the testator to acknowledge his signature
  • The marital status of the witnesses is irrelevant
  • Provided s. 9 requirements are met there is no additional obligation for everyone in involved to be present throughout the execution process (although this is good practice)
  • There is no requirement for the witnesses to sign the will in the presence of each other, only the testator
How well did you know this?
1
Not at all
2
3
4
5
Perfectly
24
Q

A testator died recently. The testator’s will contains a gift of £10,000 to the testator’s brother and gives the remainder of the estate equally to the testator’s spouse and children. The testator’s spouse, and the spouse of the testator’s brother, witnessed the will.

Which one of the following accurately describes the effect of the man’s will?

The will has no effect as it was not properly executed.

Only the man’s children will inherit under his will.

The man’s spouse and brother will inherit under his will.

The man’s spouse, brother and children will inherit under his will.

The man’s brother and children will inherit under the will.

A

Only the man’s children will inherit under his will.

Correct
Correct. Where a beneficiary (or their spouse) witnesses a will the beneficiary cannot inherit under the will. In this example the testator’s spouse and the spouse of the testator’s brother witnessed the will so neither the spouse nor brother will receive any benefit. Only the gift to the children is effective.

The other options were incorrect because:

S.15 Wills Act only applies to gifts under the will – it does not affect whether or not the will was properly executed

S.15 applies to a gift to a beneficiary who witnesses a will and any gift to their spouse

How well did you know this?
1
Not at all
2
3
4
5
Perfectly
25
Q

Review the following extracts from a valid will:

Clause 1: I hereby REVOKE all former wills and testamentary dispositions and declare this to be my last will

Clause 2: I APPOINT ANDREW BATES of […] and CATHY DENNIS of […] (hereinafter called “my Trustees” which expression where the context admits includes any trustee hereof for the time being) to be the executors and trustees of this my will but if either or both of them shall die in my lifetime or be unable or unwilling to act as my executor I appoint ESTHER FISHER of […] to fill any vacancy thereby arising

The testator died last week. Andrew died before the testator. You may assume anyone entitled to act as executor or trustee is willing and able to do so.

Which one of the following is correct?

Cathy and Esther will be appointed as executors of the estate. If any trust arises following the testator’s death, Cathy and Esther will also be the trustees.

Clause 1 needed to include the date the will was signed to avoid any confusion about which is the last valid will the testator made.

Cathy would act as the sole executor of the estate. If any trust arises following the testator’s death Esther would need to act as trustee with Cathy so there are two trustees.

Cathy and Esther will be appointed as executors of the estate. However, if any trust arises following the testator’s death, it is not clear who will act as the trustees.

Clause 1 did not revoke any codicils made before this will.

A

Cathy and Esther will be appointed as executors of the estate. If any trust arises following the testator’s death, Cathy and Esther will also be the trustees.

Correct. Andrew cannot act as he has pre-deceased the testator and Clause 2 appoints Esther in his place. Clause 2 also confirms that the executors should also act as trustees of any trust.

The other options were incorrect because:

  • The revocation clause revokes any will or codicil made previously

-Esther is appointed as a replacement for Andrew (there is no option for Cathy to act alone)

-A will should contain a date but this can be at the end or as part of the commencement clause – it does not need to be part of the revocation clause

incorrect:

Clause 1 needed to include the date the will was signed to avoid any confusion about which is the last valid will the testator made.

Incorrect. Refer to your materials on will drafting – in particular the drafting of the commencement clause. A date should be included in the will, but this could be part of either the commencement or attestation clause.

Cathy would act as the sole executor of the estate. If any trust arises following the testator’s death Esther would need to act as trustee with Cathy so there are two trustees.

Incorrect. Refer to your materials on will drafting – in particular the drafting of the executor clause. Clause 2 makes it clear that if one of the original executors is unable to act (in this case Andrew) then Esther will fill the vacancy.

How well did you know this?
1
Not at all
2
3
4
5
Perfectly
26
Q

Review the following extracts from a valid will:

Clause 2: I WISH for any part or parts of my body where possible to be used for transplantation and for the treatment of others

Clause 4: I APPOINT my brother and sister jointly to be the guardians of any of my children who have not attained the age of 18 at the death of the survivor of myself and my civil partner

The testator died last week and is survived by her civil partner and their first daughter (aged 25), second daughter (aged 17) and their son (aged 14).

Which one of the following is correct?

Either the testator’s brother or the testator’s sister will be appointed as sole guardian of the testator’s second daughter and son.

The testator’s brother and sister will be appointed as guardians for the three children.

No guardians are appointed following the testator’s death.

The executors must ensure the testator’s body is used for transplantation and for the medical treatment of others.

The testator’s brother and sister will be appointed as guardians for the testator’s second daughter and son only.

A

No guardians are appointed following the testator’s death.

No guardians are appointed following the testator’s death.

How well did you know this?
1
Not at all
2
3
4
5
Perfectly
27
Q

Your client is a lay executor appointed under a valid will that contains a number of express administrative powers for executors, however, no express charging provisions are included.

Which one of the following is correct?

If there is a conflict between any of the express powers in the will and the equivalent statutory power your client may act in accordance with whichever confers the widest power.

If there is a conflict between any of the express powers in the will and the equivalent statutory power, your client must act in accordance with the statutory power.

Your client can charge for their time but only in accordance with the statutory power.

Your client can charge for their time as there is no express power preventing this.

Your client cannot charge for their time as there is no express power allowing this.

A

Your client cannot charge for their time as there is no express power allowing this.

Correct. There is no express power to charge in the will and the statutory power to charge applies only to professionals. The other options were incorrect because:

Your client cannot charge for their time

Express powers take priority over the implied statutory terms in a will where there is conflict

How well did you know this?
1
Not at all
2
3
4
5
Perfectly
28
Q

A valid will contains the following clause:

“ I give my gold diamond engagement ring absolutely and free of tax and costs of transfer to my niece”

You have just discovered that the testator sold the gold diamond engagement ring for £4,000 a month ago. However, there is still a platinum engagement ring among the testator’s personal possessions.

What will the testator’s niece receive?

Platinum engagement ring but only if it has not been left to another beneficiary.

The testator’s niece may choose either £4,000 or the platinum ring but only if it has not been left to another beneficiary.

Nothing

A gold engagement ring that the personal representatives must buy as a replacement.

£4,000

A

Nothing

Correct. The gift of the ring is a specific legacy. If the testator no longer owns the item at the date of their death then the gift in the will cannot be given effect to. In the absence of any express clause providing for an alternative gift, the clause fails and the beneficiary will receive nothing.

The other options were incorrect because:

· The original gift adeems because the testator does not own the item specifically left by the will

-There is no cash or chattel alternative which can be given as there are no express words permitting this

How well did you know this?
1
Not at all
2
3
4
5
Perfectly
29
Q

A testator made a valid will 2 years ago that contains the following clause:

“I give to my nephew absolutely and free of tax and costs of transfer all my personal chattels as defined by section 55(1)(x) of the Administration of Estates Act 1925”

The residue of the estate passes to the testator’s brother. There are no other relevant clauses within the will.

The testator died yesterday. Included among their assets is a van which the testator used only for business purposes, cash in the house worth £35 and a ring that was purchased as an investment. The testator wore the ring on special occasions a few times each year.

Which of the following is correct?

The testator’s nephew will receive the van. The testator’s brother will receive the ring and £35 cash.

The testator’s brother will receive the ring, £35 cash and the van.

The testator’s nephew will receive the ring and the van. The testator’s brother will receive the £35 cash.

The testator’s nephew will receive the ring, £35 cash and the van.

The testator’s nephew will receive the ring. The testator’s brother will receive the £35 cash and the van.

A

The testator’s nephew will receive the ring. The testator’s brother will receive the £35 cash and the van.

Correct. The definition of chattels excludes business assets (the van), cash (£35) and assets owned solely as an investment. The ring was not owned solely as an investment so will be included in the gift of chattels, the other items are not chattels so will pass under the gift of residue.

How well did you know this?
1
Not at all
2
3
4
5
Perfectly
30
Q

A testator owns two properties. A residential house and a holiday home that is let out.

The testator owns the residential house as joint tenants with their civil partner and owns the holiday home as tenants in common with their sister. The testator wants their civil partner to inherit the residential house and their sister to inherit the holiday home.

The rest of the estate (the residue) will be left to the testator’s children.

When you draft the testator’s will which of the following are required?

A clause giving the testator’s share of the holiday home to their sister.

No express clauses are required to achieve the testator’s wishes.

A clause severing the joint tenancy of the residential house.

A clause giving the testator’s share of the residential house to their civil partner.

A clause giving the testator’s share of the residential house to their civil partner and another clause giving the testator’s share of the holiday home to their sister.

A

A clause giving the testator’s share of the holiday home to their sister.

Correct. The holiday home is owned as tenants in common and therefore the testator’s share passes in accordance with the will and is not automatically inherited by the sister as the other co-owner. As we know the residue will not be left to the testator’s sister, an express clause giving the testator’s share of the holiday home to their sister is required. The residence owned as joint tenants will pass by survivorship to the surviving co-owner automatically. As the testator wants their civil partner to inherit in any event no express clause in the will is required to achieve the testator’s wishes.

The other options were incorrect because: - No severance of the joint tenancy is required as the testator wants the surviving co-owner to inherit the residential house - No express clause within the will is needed re the residential house; the law of survivorship achieves testator’s aim - An express clause giving the share of the holiday home to the sister is required as this interest will otherwise pass as part of the gift of residue to the testator’s children.

How well did you know this?
1
Not at all
2
3
4
5
Perfectly
31
Q

A valid will contains the following gifts:

“I give my car to my sister”

“I give my collection of stamps to my brother”

“I give £100 to my cleaner”

The testator owned a blue car when the will was made but had replaced this with a red car by the time they died.

After the testator made the will they sold over half of the stamps in the collection but later added new purchases to the collection.

There are no other relevant provisions.

Which one of the following is correct?

The testator’s sister will receive the red car.

The gift to the testator’s sister will fail and she will receive nothing.

The testator’s cleaner at the date of the testator’s death will receive £100.

The testator’s brother will receive only the stamps that were in the collection at the date of the will.

The testator’s brother will receive only the new stamps purchased after the will was executed.

A

The gift to the testator’s sister will fail and she will receive nothing.

Correct
Correct. The use of the word “my” in the specific gift overrides the general rule in s.24 Wills Act 1837 and the property is identified as at the date of the will. As the testator no longer owns the blue car then this gift adeems and the testator’s sister receives nothing.

The other options were incorrect because:

The identify of people is established with reference to the date of the will

Gifts of collections comprise the assets that fall within the collection as at the date of the testator’s death whether acquired before or after the will was executed

incorrect
The testator’s cleaner at the date of the testator’s death will receive £100.

Incorrect. Refer to your materials on the construction of clauses in a will. People are usually identified with reference to the date of the will.

How well did you know this?
1
Not at all
2
3
4
5
Perfectly
32
Q

A testator leaves the following legacy in their valid will:

“I give the piano I own at the date of my death to my neighbour”

The testator’s estate will be subject to inheritance tax.

Which one of the following is correct?

The gift of the piano is made subject to tax and costs of transfer.

The gift of the piano is made free of tax but subject to the costs of transfer.

The gift of the piano is made subject to tax but free of costs of transfer.

The gift of the piano is made free of tax and costs of transfer.

The executor may choose whether the tax and transfer costs are borne by the beneficiary or treated as a general administration expense.

A

The gift of the piano is made free of tax but subject to the costs of transfer.

Correct. Where a will is silent, a legacy in a will is made free of tax but subject to the costs of transfer.

The other options were incorrect because the general rules were incorrectly stated and the executors do not have a general discretionary power to decide where the costs fall.

How well did you know this?
1
Not at all
2
3
4
5
Perfectly
33
Q

By her will a testator left a property, which was charged with a mortgage during her lifetime, to her son. The residue of the testator’s estate is left to her daughter.

The will is silent on the liability to pay any outstanding mortgage. The testator did not have a mortgage protection policy.

Which of the following is correct?

The burden of repaying the mortgage will be shared equally between the property and the residue of the estate.

The burden of repaying the mortgage will be shared proportionately according to value between the property and the residue of the estate.

The mortgage will be repaid from the residue of the estate.

The mortgage must be repaid by the executors using the sale proceeds of the property before they distribute the balance to the testator’s son.

The testator’s son will inherit the property subject to the mortgage.

A

The testator’s son will inherit the property subject to the mortgage.

Correct. The general rule under s.35 Administration of Estates Act 1925, which applies here in the absence of any express wording to the contrary, states that the property charged shall bear the primary liability for repayment.

The other options were incorrect because:

The executors are not required to sell the property - they can transfer the property to the beneficiary subject to the charge

The residue of the estate is not liable for repayment of the mortgage

How well did you know this?
1
Not at all
2
3
4
5
Perfectly
34
Q

A valid will contains the following gift:

“I give £400 to each of my children who are alive at the date of my death contingent on them reaching the age of 21”

When the testator dies they have three children A (aged 24), B (aged 19) and C (aged 15).

Which of the children has a contingent interest under the will?

B and C only

C only

All of them

None of them

A only

A

B and C only

Correct. The gift is made contingent on the children reaching the age of 21. A has satisfied this contingency by the time the testator dies so their gift vests immediately. The other two children have not satisfied the contingency so their gifts are contingent. That one is an adult and the other a minor makes no difference.

The other options were incorrect as A has a vested interest, and B and C are under the age of 21 so have not satisfied the contingency

How well did you know this?
1
Not at all
2
3
4
5
Perfectly
35
Q

A testator recently died leaving a valid will that contains the following gifts:

“I give £40,000 to my sister”

“I leave the residue of my estate equally between my children”

The testator’s sister died a year ago and is survived by her only child (the testator’s niece). One of the testator’s three children died 5 years ago, leaving two of her own children (the testator’s grandchildren).

There are no other relevant provisions in the will.

Which one of the following correctly describes how the testator’s estate will be distributed?

The testator’s two surviving children and two grandchildren each inherit 1/4 of the residue.

The testator’s niece will receive £40,000. The testator’s two surviving children will each inherit 1/2 of the residue.

The testator’s niece will receive £40,000. The testator’s two surviving children will each inherit 1/3 of the residue. The two grandchildren will each inherit 1/6 of the residue.

The testator’s two surviving children will each inherit 1/3 of the residue. The two grandchildren will each inherit 1/6 of the residue.

The testator’s two surviving children will each inherit 1/2 of the residue.

A

The testator’s two surviving children will each inherit 1/3 of the residue. The two grandchildren will each inherit 1/6 of the residue.

Correct. The gift to the testator’s sister will lapse because she predeceased.

S.33 Wills Act 1837 will not create an implied substitution in favour of the testator’s niece because the original gift was not made to the testator’s issue. S.33 does apply to the gift to the children. Each child would be entitled to a 1/3 share, but one child has pre-deceased the testator. Their 1/3 share will be inherited equally by their two children, who receive 1/6 each.

The other options were incorrect because: - The testator’s niece does not receive anything - The testator’s grandchildren are entitled to share between them the 1/3 of the residue their parent would otherwise have inherited

How well did you know this?
1
Not at all
2
3
4
5
Perfectly
36
Q

You are reviewing a will that includes a class gift to all of the testator’s grandchildren contingent on them reaching the age of 18. The will does not state when the class of ‘grandchildren’ closes.

When the testator made the will they had five grandchildren. When the testator died they had seven grandchildren, all of whom are currently minors.

When will the class of ‘grandchildren’ close?

When it is no longer possible for any further grandchildren to be born.

When the first of her grandchildren reaches 18.

The class is already closed and only includes the grandchildren the testator had when they died.

The class is already closed and only includes the grandchildren the testator had when they made the will.

As the testator’s will does not specify when the class of ‘grandchildren’ closes the clause fails for uncertainty.

A

When the first of her grandchildren reaches 18.

Correct. The class closing rules mean the class of beneficiaries is identified when the first beneficiary has a vested interest. This has not yet occurred as all of the testator’s grandchildren are under the age of 18. Once the first of them reaches 18 the class will close.

The other options were incorrect because:

  • The clause will not fail for uncertainty (the class closing rules create certainty)
  • No beneficiary has a vested interest at the date of her death
  • Without express wording to suggest otherwise, the date of the will is not the correct reference point
  • It is possible to close the class without having to wait for it to be impossible for further members to be born
How well did you know this?
1
Not at all
2
3
4
5
Perfectly
37
Q

Which of the following is the standard applied when assessing the claim of a surviving spouse or civil partner under the IPFDA 1975?

Such financial provision as it would be reasonable in all the circumstances for any applicant to receive for his/her maintenance.

Such financial provision as it would be reasonable in all the circumstances for a husband or wife or civil partner to receive whether or not that provision is required for his or her maintenance.

Such financial provision as it would be reasonable in all the circumstances for a husband or wife or civil partner to receive for his/her maintenance.

A

Such financial provision as it would be reasonable in all the circumstances for a husband or wife or civil partner to receive whether or not that provision is required for his or her maintenance.

Correct: The surviving spouse/civil partner standard does not require the court to consider whether the provision is required for the applicant’s maintenance.

How well did you know this?
1
Not at all
2
3
4
5
Perfectly
38
Q

Which of the following is the court not expressly required to consider under section 3(1) IPFDA 1975?

Any physical or mental disability of any applicant or beneficiary.

The financial resources and financial needs of any beneficiaries of the deceased’s estate.

The financial resources and financial needs of any applicants.

The size and nature of the net estate of the deceased.

The applicant’s age and the duration of their relationship with the deceased.

A

The applicant’s age and the duration of their relationship with the deceased.

Correct: Although the court is required to consider any matter that they consider relevant in the circumstances, the applicant’s age and duration of their relationship with the deceased are not expressly referred to in section 3(1) IPFDA.

How well did you know this?
1
Not at all
2
3
4
5
Perfectly
39
Q

Which of the following most accurately reflects the guidelines applicable to spouses and civil partners?

The court must consider the applicant’s age and duration of the marriage or civil partnership, as well as whether, and if so the length of time for which and the basis on which the deceased maintained the applicant, and the extent of the contribution made by way of maintenance. The court must also take into account what the applicant might reasonably have expected to receive on divorce or dissolution of the civil partnership.

The court must consider the applicant’s age and duration of the marriage or civil partnership, as well as the length of any period of cohabitation prior to the marriage or civil partnership. The court also considers the contributions made by the applicant to the welfare of the deceased (including any contributions made by looking after the home or caring for the family). The court must also take into account what the applicant might reasonably have expected to receive on divorce or dissolution of the civil partnership.

The court must consider the applicant’s age and duration of the marriage or civil partnership, as well as any contributions made by the applicant to the welfare of the deceased (including any contributions made by looking after the home or caring for the family) and the liability of any other person to maintain the applicant. The court must also take into account what the applicant might reasonably have expected to receive on divorce or dissolution of the civil partnership.

The court must consider the applicant’s age and duration of the marriage or civil partnership, as well as any contributions made by the applicant to the welfare of the deceased (including any contributions made by looking after the home or caring for the family). The court must also take into account what the applicant might reasonably have expected to receive on divorce or dissolution of the civil partnership.

The court must consider the applicant’s age and duration of the marriage or civil partnership, as well as any contributions made by the applicant to the welfare of the deceased (including any contributions made by looking after the home or caring for the family) and whether, and if so to what extent, the deceased assumed responsibility for the maintenance of the applicant. The court must also take into account what the applicant might reasonably have expected to receive on divorce or dissolution of the civil partnership.

A

The court must consider the applicant’s age and duration of the marriage or civil partnership, as well as any contributions made by the applicant to the welfare of the deceased (including any contributions made by looking after the home or caring for the family). The court must also take into account what the applicant might reasonably have expected to receive on divorce or dissolution of the civil partnership.

Correct: The court considers all the factors above. There is no express requirement to consider whether the deceased maintained the applicant, or the length of their relationship prior to marriage/civil partnership (although these may be considered under the common guidelines if the court considers them relevant in the circumstances).

How well did you know this?
1
Not at all
2
3
4
5
Perfectly
40
Q

A client seeks your advice following the death of her brother 18 months ago. By her brother’s will your client is due to inherit the whole of his estate, with a substitution in favour of her children if she did not inherit for any reason.

The client is unhappy that her brother did not include their step-sister in his will and now wants to change the distribution of his estate so that she keeps half and the other half passes to their step-sister.

The client does not want to be taxed for either IHT or CGT purposes as though she personally made this gift to her step-sister.

Which of the following is correct?

It is not possible for the client to enter into either a variation or disclaimer because her brother died more than a year ago.

The client’s objectives can only be met if she varies her inheritance.

The client’s objectives can be met if she enters either a variation or disclaimer because the writing back effect for IHT and CGT purposes applies to both.

It is not possible for all of the client’s objectives to be met by either a variation or disclaimer.

The client’s objectives can only be met if she disclaims her inheritance.

A

The client’s objectives can only be met if she varies her inheritance.

Correct. A variation would achieve all of her objectives. By way of variation the client can re-distribute part of her inheritance and elect who should benefit instead. The writing back effect for both IHT and CGT will apply provided the variation is made in writing within 2 years following her brother’s death. The other options were incorrect because:

A disclaimer could not be used because the client cannot disclaim part of her inheritance, neither can she nominate the alternative beneficiary (under the will any disclaimed inheritance would pass directly to her children, not the step sister)

The time limit for varying the distribution is 2 years following death.

How well did you know this?
1
Not at all
2
3
4
5
Perfectly
41
Q

A client seeks your advice following the death of their father. The client recently inherited a share of her father’s estate but does not wish to keep her inheritance because she is wealthy in her own right. Instead your client wishes to pass part of her inheritance (a cash sum of £50,000) to her adult children in the most tax efficient manner. Inheritance tax was payable following her father’s death. You advise your client to enter a deed of variation.

Which of the following is the best advice for your client?

Your client should make an election for s 62 TCGA to apply to the variation.

Your client should make an election for s 142 IHTA to apply to the variation.

There is no benefit to claiming the writing back effect under either s 142 IHTA or s 69 TCGA because no refund of tax can be claimed.

Your client will need to seek the approval of her father’s PRs before she can vary her inheritance.

Your client should make an election for s 142 IHTA and s 62 TCGA to apply to the variation.

A

Your client should make an election for s 142 IHTA to apply to the variation.

Correct. If the client enters a variation and claims the writing back effect for IHT under s 142 IHTA her father’s estate will be taxed as though £50,000 was given to the client’s children and the remainder to the client. There is no change to the IHT liability on her father’s estate as a result (none of the beneficiaries are exempt) therefore consent from her father’s PRs is not required. The advantage to claiming the writing back effect under s 142 is that the client will not be making PET and so avoids the risk of this failing if she died within 7 years.

The other options were incorrect because:

No writing back for capital gains tax purposes should be made because the subject matter of the variation is cash- which is exempt from CGT.

There is an advantage to making an election under s 142 IHTA – the gift of £50,000 to the client’s children is treated as having been made from her father’s estate and will not be a PET by the client.

There would be no change to the IHT liability following the variation so PR consent is not required.

How well did you know this?
1
Not at all
2
3
4
5
Perfectly
42
Q

A client seeks advice with regards the following clause in her father’s will.

“I give all of my chattels to my daughter and hope that she will distribute them in accordance with the instructions I have left with my will”.

There is a letter of instruction attached to the father’s will requesting that your client distributes his chattels equally between all of his grandchildren as she sees fit.

Which one of the following is correct?

The effect of s 143 IHTA is to treat the gift of chattels to the client as being made free of IHT.

The client has an obligation to distribute the chattels to her children.

If the client distributes the chattels to her children she would make a disposal for CGT purposes.

By this clause the client is appointed as trustee of the chattels.

If the client distributes the chattels to her children she would make a PET for IHT purposes.

A

If the client distributes the chattels to her children she would make a disposal for CGT purposes.

Correct. The clause in the will is a precatory trust. The client is the beneficiary of the chattels. However, her father hoped that she would distribute them to his grandchildren. No legal trust is created so the client is not a trustee and has no obligation to comply with her father’s wishes. If she does give the chattels to the grandchildren, s 143 IHTA applies automatically to treat these gifts as if they were made by her father’s will (and not the client) for IHT purposes. There is no equivalent for CGT however, so the gifts will be a disposal by the client for CGT purposes.

The other options were incorrect because:

S 143 does not create a tax-free gift under the will

A gift of chattels made by the client to the grandchildren would ordinarily be a PET but the effect of s 143 is to treat these gifts as having been made by the will of the client’s father, and so she will not make a PET

No formal trust is created

The client has no obligation to comply with the wishes of her father

How well did you know this?
1
Not at all
2
3
4
5
Perfectly
43
Q

A woman died intestate two months ago and her estate was distributed between her three adult children. The woman had never married or entered a civil partnership. On the date of her death she had been living with her long- term partner and his son (aged 15) for the previous 10 months. The woman provided financial maintenance for both her partner and her step-son throughout the period they all lived together.
Who is entitled to bring a claim against the woman’s estate under the Inheritance (Provision for Family and Dependants) Act 1975?

the woman’s partner, children and step-children

no one is entitled to bring a claim because the woman did not leave a will

the woman’s partner only

the woman’s step-child only

the woman’s children and step-child

A

the woman’s partner, children and step-children

who is entitled?
even though the estate has been distributed to the children, they are still entitled

step child 1(1)(d)

partner 1(1)(e): maintenance

in this case, someone independent would be appointed on behalf of the step child since the partner has an interest

How well did you know this?
1
Not at all
2
3
4
5
Perfectly
44
Q

A man died 8 months ago and the grant of probate was issued four months later. By his will the man left all of his estate to his son and nothing to his partner with whom he had been living for the last 3 years. The man’s partner wishes to bring a claim under the IPFDA 75 (the ‘Act’).

Is the man’s partner able to bring a claim under the Act?

No - because the main died more than 6 months ago

No - because it has been less than 6 months since the date of the grant of probate

No - because they were not married to or in a civil partnership with the man.

Only if a court order granting permission for the deadline to be extended is obtained.

Yes

A

Yes

within the 6 months time period as the grant of probate was only 4 months ago

How well did you know this?
1
Not at all
2
3
4
5
Perfectly
45
Q

A woman makes a PET with a chargeable value of £50,000. She does the same 11 years later. A year after the second gift, she dies. She makes no other lifetime gifts. What is the cumulative total on her death?

£0

£175,000

£100,000

£325,000

£50,000

A

£50,000

Correct
Correct: The woman has made one PET of £50,000 in the last seven years.

How well did you know this?
1
Not at all
2
3
4
5
Perfectly
46
Q

A man gifts £500,000 into trust. If the man lives for another ten years, which one of the following is correct in relation to the inheritance tax treatment of the gift?

Tax will be payable on the man’s death at the death rate of 40%

Tax will be payable at the time of transfer at the death rate of 40%

Tax will initially be payable at the lifetime rate of 20% then reassessed at the death rate of 40%

No tax will be payable at the time of the transfer

Tax will be payable at the lifetime rate of 20%

A

Tax will be payable at the lifetime rate of 20%

Correct: The gift is an LCT. The man survives more than seven years after the LCT so it is not reassessed at the death rate.

How well did you know this?
1
Not at all
2
3
4
5
Perfectly
47
Q

Which one of the following scenarios does not include a chargeable event for inheritance tax purposes?

A PET which is made 9 years before death

An LCT and a PET made together

The death of an individual who has made no lifetime gifts

An LCT which is made 15 years before death

A PET which is made 3 years before death

A

A PET which is made 9 years before death

Correct: A PET made more than 7 years before death is not chargeable.

How well did you know this?
1
Not at all
2
3
4
5
Perfectly
48
Q

A man makes three gifts in his lifetime, each with a chargeable value of £10,000. The first gift is made 15 years before his death, the second is five years before his death and the final one is gifted in the week of his death. What is the basic Nil Rate Band available to his death estate? (There is no TNRB available.)

£295,000

£20,000

£325,000

£30,000

£305,000

A

£305,000

Correct: Two of the gifts were made in the seven years before the man’s death so are included in the cumulative total.

How well did you know this?
1
Not at all
2
3
4
5
Perfectly
49
Q

What is the maximum combined value of the nil rate bands available to the estate of a surviving spouse?

£825,000

£650,000

£500,000

It depends how many times the deceased was married as they can receive TNRB from each former spouse

£1,000,000

A

£1,000,000

Correct: A surviving spouse can make use of their own basic NRB (£325,000) and RNRB (£175,000) and receive a transfer of up to 100% of the basic NRB and RNRB from the estate of their deceased spouse.

How well did you know this?
1
Not at all
2
3
4
5
Perfectly
50
Q

A woman died two years ago and left her entire estate worth £400,000 to her husband. She made no lifetime gifts. Her husband died recently. He left his entire estate to his children. His estate was valued at £900,000 including a family home worth £500,000. He made three PETS in his lifetime totalling £500,000. All three were made over 20 years ago.

Which one of the following is correct in terms of available Nil Rate Bands?

His estate can claim a basic NRB and his wife’s basic NRB only

His estate will have no NRB available to it

His estate can claim a basic NRB, his wife’s basic NRB, his RNRB and his wife’s RNRB

His estate can claim a basic NRB only

A

His estate can claim a basic NRB, his wife’s basic NRB, his RNRB and his wife’s RNRB

Correct: The man and his wife both died with full basic NRB available (his PETs were made more than 7 years ago so the man’s cumulative total when he died was zero). The wife also did not use her RNRB so this is also available to transfer.

How well did you know this?
1
Not at all
2
3
4
5
Perfectly
51
Q

A woman died six months ago. Her estate now includes the following debts:

i. Legal fees for probate work ii. Income tax bill for period up to death iii. Credit card bill for pre-death spending iv. Funeral bill v. Post-death house clearance invoice

Which of the following correctly lists the debts that can be deducted for inheritance tax purposes?

All of them

The legal fees, income tax, credit card bill and funeral bill

The income tax, credit card bill and funeral bill

The funeral bill only

None of them

A

The income tax, credit card bill and funeral bill

Correct: The deceased’s income tax and pre-death liabilities are deductible, as are funeral expenses. The other expenses are not deductible for tax purposes (but can be paid from the estate assets).

How well did you know this?
1
Not at all
2
3
4
5
Perfectly
52
Q

A solicitor has calculated the inheritance tax due on his client’s estate. The client had never married and left her entire estate to her brother. The estate consisted of personal chattels, cash in a bank account and a house owned as joint tenants with her brother.

The solicitor has reached the wrong figure for the inheritance tax due.

Which of the following has caused the error?

When calculating the tax due, the solicitor skipped step 6 of the IHT calculation and went straight to step 7.

When calculating the cumulative total, the solicitor included three LCTs (made 4, 5 and 6 years ago) but did not include a PET (made 8 years ago).

When calculating the value of the taxable estate the solicitor deducted funeral expenses.

When valuing the taxable death estate, the solicitor did not include the deceased’s share of a house held as joint tenants with her brother because joint tenancy assets pass via survivorship.

When calculating the tax due at step 7 of the IHT calculation, the solicitor only applied the basic NRB and did not consider the availability of the transferrable NRB.

A

When valuing the taxable death estate, the solicitor did not include the deceased’s share of a house held as joint tenants with her brother because joint tenancy assets pass via survivorship.

Correct: Although assets owned as joint tenants pass via survivorship (and therefore do not form part of the succession estate) the deceased’s interest is still part of the taxable death estate. By failing to include the value of the deceased’s share of the house, the solicitor will have reached the wrong figure for the taxable death estate.

How well did you know this?
1
Not at all
2
3
4
5
Perfectly
53
Q

Which of the following is the correct order of the 7 step calculation?

Step 1 - Calculate the cumulative total Step 2 - Identify the taxable estate Step 3 - Value the taxable estate Step 4 - Apply the RNRB and NRB Step 5 - Deduct debts Step 6 - Apply exemptions and reliefs Step 7- Calculate tax

Step 1 - Calculate the cumulative total Step 2 - Identify the taxable estate Step 3 - Value the taxable estate Step 4 - Deduct debts Step 5 - Apply exemptions and reliefs Step 6 - Apply NRB Step 7- Apply RNRB and calculate tax

Step 1 - Identify the taxable estate Step 2 - Calculate the cumulative total Step 3 - Value the taxable estate Step 4 - Deduct debts Step 5 - Apply exemptions and reliefs Step 6 - Apply RNRB Step 7- Apply NRB and calculate tax

Step 1 - Calculate the cumulative total, Step 2 - Identify the taxable estate, Step 3 - Value the taxable estate, Step 4 - Deduct debts, Step 5 -Apply exemptions and reliefs, Step 6 - Apply RNRB, Step 7 - Apply NRB and calculate tax

Step 1 - Identify the taxable estate Step 2 - Calculate the cumulative total Step 3 - Value the taxable estate Step 4 - Apply the RNRB and NRB Step 5 - Deduct debts Step 6 - Apply exemptions and reliefs Step 7- Calculate tax

A

Step 1 - Calculate the cumulative total, Step 2 - Identify the taxable estate, Step 3 - Value the taxable estate, Step 4 - Deduct debts, Step 5 -Apply exemptions and reliefs, Step 6 - Apply RNRB, Step 7 - Apply NRB and calculate tax

Correct. It is necessary to start by calculating the cumulative total, then identify and value the taxable estate. Deduct debts from the gross value of the estate to reach the net value, then apply exemptions and reliefs. RNRB should be applied before the basic NRB, then calculate the tax.

How well did you know this?
1
Not at all
2
3
4
5
Perfectly
54
Q

True or False: IHT exemptions can only be used to reduce the IHT liability on a person’s death estate, or in respect of lifetimes transfers, but not both.

False

True

A

False

Correct
Correct: There are exemptions and reliefs that apply to both lifetime transfers and the death estate.

How well did you know this?
1
Not at all
2
3
4
5
Perfectly
55
Q

True or False: The basis of most IHT exemptions and reliefs is common law.

False

True

A

False

Correct
Correct: Exemptions and reliefs are available by virtue of statutory authority and are set out in the Inheritance Tax Act 1984. Case law relating to exemptions and reliefs usually concerns a dispute over the application of statutory rules.

How well did you know this?
1
Not at all
2
3
4
5
Perfectly
56
Q

True or False: The nil rate band (‘NRB’) is not an IHT exemption or relief.

True

False

A

True

Correct: The NRB is a 0% rate of tax. To the extent the value of a transfer falls within the NRB there is no tax to pay, which is why people sometimes confuse the NRB as an exemption/relief.

How well did you know this?
1
Not at all
2
3
4
5
Perfectly
57
Q

A man wants to give away some of his assets to his civil partner. Both parties are domiciled in the UK.

The man is considering making a gift of cash now and then giving the remainder of his estate to his civil partner by his will to take effect on the man’s death.

Which of the following represents the most accurate advice for the man?

The gift of cash will be exempt from IHT provided the man survives 7 years after making the transfer.

The gift of cash and the gift of the estate will be exempt from IHT irrespective of their value.

The gift of cash will be exempt from IHT. The gift of the man’s estate cannot be exempt from IHT if the man choses to leave his assets into a life interest trust.

The gift of cash and the gift of the estate will be exempt from IHT up to the value of the man’s nil rate band.

The gift of cash will not be exempt from IHT. The gift of the man’s estate will not be exempt from IHT if it is conditional upon the man’s civil partner surviving him by 28 days.

A

The gift of cash and the gift of the estate will be exempt from IHT irrespective of their value.

Correct
Correct: Spouse/civil partner exemption applies to lifetime transfers and to gifts made under a will – any assets passing to the man’s civil partner will be entirely free of IHT.

The other options were incorrect because:

There is no upper limit to the amount of the spouse exemption where both parties are UK domiciled.

Although the gift of cash to a person during the man’s lifetime would usually be a PET, this cash transfer is exempt by virtue of spouse/civil partner exemption and it does not matter when the man dies

The man could leave his assets into a life interest trust and spouse exemption could still apply provided the man’s civil partner is the life tenant

A standard survivorship clause in a will does not prevent the application of spouse/civil partner exemption

How well did you know this?
1
Not at all
2
3
4
5
Perfectly
58
Q

A woman’s will contains the following clause:

“I give £8,000 to Macmillan Cancer Support (registered charity number 261017) of [ address ] for its general purposes”

The woman has a full nil rate band (NRB) available.

How much IHT is payable on this gift?

£0 – if the value of her estate is below the NRB.

£0 – irrespective of the value of her estate.

£2,880 (£8,000 x 36%) – if her estate is valued above the NRB.

£3,200 (£8,000 x 40%) – if her estate is valued above the NRB.

£0 – because the value of the gift is below the NRB.

A

£0 – irrespective of the value of her estate.

Correct: Charity exemption applies and this gift is fully exempt. The size of the gift and the value of the other assets in her estate are irrelevant. The reduced death rate of IHT at 36% is only ever applied to the chargeable assets within an estate – it is not a rate of tax applied to a charitable gift itself.

How well did you know this?
1
Not at all
2
3
4
5
Perfectly
59
Q

True or false: All of the following are available to use in respect of both lifetime gifts and the death estate

  • Political party exemption

-Exemptions for gifts for national purposes or to heritage maintenance funds

-Exemption for gifts to EBTs

-Exemption for gifts to housing associations

True

False

A

True

Correct
Correct: All of these are available to use in respect of lifetime transfers and the death estate. You are not required to know about these exemptions in detail.

How well did you know this?
1
Not at all
2
3
4
5
Perfectly
60
Q

A man entered into a business partnership with a friend. They agreed to hold the shares in their new venture on a 50:50 basis. The man died six months later.

Based on the information provided, which one of the following is correct in relation to the inheritance tax position following the man’s death?

APR will be available at 100%

APR will be available at 50%

BPR will be available at 100%

No IHT reliefs will be available

BPR will be available at 50%

A

No IHT reliefs will be available

Correct: The qualifying period of ownership has not been satisfied for either BPR or APR so neither relief is relevant.

How well did you know this?
1
Not at all
2
3
4
5
Perfectly
61
Q

A woman owns shares in a trading company called “It’s all going to be OK Ltd”. The woman purchased the shares 2.5 years ago and they are now worth £100,000. The woman does not have a controlling interest.

The woman has just died and leaves all of her taxable estate to her daughter.

Which one of the following is correct with regards business property relief?

No BPR can be claimed as she did not survive more than 3 years after having acquired the shares

BPR of £50,000 can be claimed

BPR can only be claimed in respect of the woman’s estate if her daughter keeps the shares for more than 2 years

No BPR can be claimed as she did not have a controlling interest

BPR at £100,000 can be claimed

A

BPR at £100,000 can be claimed

Correct
Correct. The woman owned shares in a private company (it ends with “ltd”) for more than 2 years before she died. BPR applies at 100%, ie £100,000.

The other options were incorrect because:

The 50% rate is not applicable (these are not quoted shares)

There is no requirement to have a controlling interest for a private company shareholding to qualify

The qualifying period of ownership is 2 years, not 3

The period of time her daughter owns the shares for is irrelevant when considering the availability of BPR for the woman’s estate

How well did you know this?
1
Not at all
2
3
4
5
Perfectly
62
Q

An unmarried woman dies owning land at an agricultural value of £300,000. The open market value of the land is £360,000. She has farmed the land herself for 5 years.

Which one of the following is correct in relation to her death estate?

APR worth £150,000 can be claimed.

If the land qualifies for BPR, this will be claimed instead of APR.

APR worth £300,000 can be claimed.

Neither APR nor BPR can be claimed.

APR worth £360,000 can be claimed.

A

APR worth £300,000 can be claimed.

Correct: The woman has owned and occupied the land for the purposes of agriculture for more than 2 years. The conditions of APR are met. APR can be claimed at 100% of the agricultural (not open market) value of the land. The other options are incorrect because: APR applies at 100% in this case, not 50% APR applies to the agricultural value, not market value, of the land Where both APR and BPR apply, APR takes priority.

How well did you know this?
1
Not at all
2
3
4
5
Perfectly
63
Q

A man dies leaving the whole of his estate to his son. No inheritance tax is payable following his death.

The man’s estate comprises only his bank accounts (£4,000), a house and its surrounding woodland (combined value of £250,000) and his personal possession (£1,500). The man purchased the house and woodland six years ago. The timber value of the land is £15,000.

Which of one of the following is correct?

Woodlands relief is not applicable.

A claim for woodlands relief can be made (with reference to the value of £15,000).

A claim for woodlands relief would be made when the man’s son sells the land.

A claim for woodlands relief can be made (with reference to the value of £250,000).

It would be preferable for the man’s estate to make use of BPR instead of woodlands relief.

A

Woodlands relief is not applicable.

Correct
Correct. Woodlands relief is a deferral of inheritance tax (‘IHT’) that would otherwise be payable. This is estate is not subject to IHT so there is nothing to defer.

incorrect
A claim for woodlands relief can be made (with reference to the value of £15,000).

Incorrect. Refer to your notes on woodland relief. Woodlands relief is not relevant to this estate as there is no IHT due.

How well did you know this?
1
Not at all
2
3
4
5
Perfectly
64
Q

A woman dies leaving a will that gives all of her estate to her sister. The woman and her sister were joint tenants of a property. The woman’s estate was taxable.

The woman’s sister dies 1 year later leaving her own estate (which includes assets inherited from the woman) to her children. The sister’s estate was not subject to IHT.

Which of the following is correct with regards quick succession relief (‘QSR’)?

QSR would apply to the woman’s estate but cannot apply to the value of the property as this passed automatically to the sister by survivorship.

QSR does not apply to either estate.

QSR would apply to the sister’s estate only.

QSR would apply to the woman’s estate only.

QSR would apply to both the woman and the sister’s estate.

A

QSR does not apply to either estate.

Correct
Correct. For QSR to apply both relevant estates must have been taxable –here the sister’s estate was not subject to IHT. The other options were incorrect as QSR does not apply.

incorrect
QSR would apply to the woman’s estate only.

Incorrect. Refer to your materials relating to QSR. Ensure you appreciate when this relief will apply; crucially there needs to have been an IHT charge in respect of both estates. The sister’s estate cannot benefit from a credit against an IHT charge of zero.

How well did you know this?
1
Not at all
2
3
4
5
Perfectly
65
Q

A man dies and his estate comprises cash, a partnership interest, and a car (which he inherited from his brother 6 months ago). The man leaves a will that includes cash gifts to his spouse (£10,000) and a local registered charity (£2,000).

Which one of the following exemptions cannot be considered when calculating the IHT liability in respect of the man’s death estate?

Business property relief

Annual exemption

Quick succession relief

Charity exemption

Spouse exemption

A

Annual exemption

Correct. The annual exemption cannot be claimed in respect of a person’s death estate. The other exemptions and reliefs were all potentially relevant:

BPR in respect of the partnership interest

Spouse exemption in respect of the £10,000 gift in his will

Charity exemption in respect of the £2,000 gift in his will

Quick succession relief in respect of the car he inherited from his brother

How well did you know this?
1
Not at all
2
3
4
5
Perfectly
66
Q

The deceased’s gross estate is valued at £410,000. The estate includes foreign property worth £150,000. The deceased is a life tenant of a life interest trust with a capital value at date of death of £20,000. The deceased had never married and had made no lifetime gifts and left his entire estate to his two children.

Which of the following correctly describes the status of the estate?

An exempt excepted estate - the estate is worth less than £650,000.

A non-excepted estate.

A low value excepted estate - the estate is worth less than £650,000.

An exempt excepted estate - the estate is worth less than £325,000.

A low value excepted estate - the estate is worth less than £325,000.

A

A non-excepted estate.

Correct
Correct. The gross value of the estate is above the nil rate band (only one is available) and no spouse or charity exemption applies. IHT is payable and also the value of foreign property is greater than £100,000.

The other options were incorrect because tax was payable on this estate so the estate could not be excepted.

How well did you know this?
1
Not at all
2
3
4
5
Perfectly
67
Q

The deceased’s gross estate is valued at £300,000. The estate includes foreign property worth £90,000. The deceased is a life tenant of a life interest trust with a capital value at date of death of £270,000. The deceased had never married and had made no lifetime gifts.

Which of the following correctly describes the status of the estate?

A low value excepted estate - the estate is worth less than £325,000.

An exempt excepted estate - the estate is worth less than £325,000.

An exempt excepted estate - the estate is worth less than £650,000.

A low value excepted estate - the estate is worth less than £650,000.

A non-excepted estate.

A

A non-excepted estate.

Correct
Correct. Although the gross value of the estate is below the NRB and no IHT is payable, because the value of the trust property is above £250,000 this estate cannot be excepted.

The other options were incorrect as they did not take into account the value of the trust interest.

How well did you know this?
1
Not at all
2
3
4
5
Perfectly
68
Q

A deceased’s estate is valued at £200,000 (which includes foreign property worth £75,000). In addition, the deceased is a life tenant of a life interest trust with a capital value at date of death of £100,000. The deceased had never married but had made relevant lifetime gifts in the past seven years totalling £10,000.

Which of the following correctly describes the status of the estate?

A non-excepted estate.

A low value excepted estate - the estate is worth less than £650,000.

A low value excepted estate - the estate is worth less than £325,000.

An exempt excepted estate - the estate is worth less than £325,000.

An exempt excepted estate - the estate is worth less than £650,000.

A

A low value excepted estate - the estate is worth less than £325,000.

Correct
Correct. The gross estate is less than the nil rate band, foreign property is less than £100,000, trust property is less than £250,000 and lifetime gifts are less than £250,000. The deceased had never married so no transferrable NRB applies and the relevant NRB amount to consider was £325,000.

The other options were incorrect because no spouse or charity exemption applied (so the estate could not be an exempt excepted estate), the relevant NRB threshold to consider was a single NRB, and the value of the assets described were below the values at which the estate would be prevented from being excepted.

How well did you know this?
1
Not at all
2
3
4
5
Perfectly
69
Q

A man died and by his valid will left the whole of his estate to his adult children.

The man’s civil partner died before the man and left the man the whole of his estate.

Neither the man nor his civil partner had made any lifetime transfers.

The man’s estate was valued at £1.5 million, which includes his home worth £500,000.

What is the value of the inheritance tax nil rate band available for the man’s estate?

£1,000,000

£650,000

£500,000

£325,000

£825,000

A

£1,000,000

Correct
Correct.

The man has his own NRB of £325,000.

The man’s PRs can also claim the TNRB in respect of his civil partner who died before him. We know that the man’s civil partner left all of his estate to the man. Therefore, the man’s civil partner did not use his own NRB because his whole estate qualified for civil partner exemption.

The RNRB of £175,000 also applies. The man left a qualifying residential interest (his home) to his lineal descendants (his children). As the man’s civil partner did not use his own RNRB, this can also be claimed by the man.

As neither of them made any lifetime transfers the maximum amounts are available.

How well did you know this?
1
Not at all
2
3
4
5
Perfectly
70
Q

A woman died and by her valid will left the whole of her estate to her adult children.

The woman’s civil partner died before the woman and left the woman the whole of her estate.

Neither the woman nor her civil partner had made any lifetime transfers.

The woman’s home is worth £700,000 and she had savings of £400,000 and other assets worth £1.6 million.

What is the value of the inheritance tax nil rate band available for the woman’s estate?

£1,000,000

£650,000

£325,000

£500,000

£825,000

A

£650,000
Correct.

The woman has her own NRB of £325,000.

The woman’s PRs can also claim the TNRB in respect of her civil partner who died before her. We know that the woman’s civil partner left all of her estate to the woman. Therefore, the woman’s civil partner did not use her own NRB because her whole estate qualified for civil partner exemption.

As neither made any lifetime transfers the maximum amounts are available.

The RNRB does not apply as the total value of the woman’s estate is above the threshold.

£825,000

There is no RNRB available at all for net estates worth £2,350,000 or more.

How well did you know this?
1
Not at all
2
3
4
5
Perfectly
71
Q

A property was purchased as tenants in common by an unmarried couple in unequal shares. The woman owned 80% and the man owned 20%.

The woman died last month and by her will leaves her share of the property to her son absolutely.

The current market value of the property is £100,000. There is no mortgage.

The woman’s estate will be subject to inheritance tax (IHT).

What will be the value of woman’s share of the property for IHT purposes?

£80,000 less a discount to reflect the fact that the property was co-owned.

£80,000 - no discount is available because the related property rules apply.

£0 as the property passes by survivorship.

£50,000 less a discount to reflect the fact that that the property was co-owned.

£80,000 - no discount is available because the couple were not married.

A

£80,000 less a discount to reflect the fact that the property was co-owned.

Correct
Correct: The value of the woman’s share is £80,000 (she owned 80% of the total worth £100,000). The woman co-owned the property and a discount to reflect the fact that the property was co-owned can be applied.

The other options were incorrect because:

  • The property does not pass by survivorship, but in any event, the value of the woman’s share is taxable whether owned as joint tenants or tenants in common
  • The value of her share is proportionate to share she owned, not the number of co-owners. A 50/50 split applies to joint tenants and those owning as tenants in common in equal shares.
  • The related property rules apply to married couples and are not relevant here.
  • A discount is available because the couple were not married. It would not have been available if they were.
How well did you know this?
1
Not at all
2
3
4
5
Perfectly
72
Q

A man died yesterday leaving his whole estate to his son. The man had never married or entered a civil partnership and had not made any lifetime gifts.

His estate comprised his home worth £150,000 (free of mortgage) which he had lived in for over 20 years and various bank accounts totalling £340,000. He also owned a house worth £180,000 free of mortgage which was rented out to tenants and had never been his residence.

The man’s debts and funeral expenses totalled £20,000.

In the tax year of the man’s death, the basic nil rate band is £325,000 and the main residence nil-rate band is £175,000.

How much inheritance tax will be payable on the man’s estate?

£0

£70,000

£60,000

£78,000

£130,000

A

£70,000

Correct
Correct.

Step 1: The man’s cumulative total was 0 as he made no lifetime gifts.

Steps 2/3: The man’s taxable estate includes both properties and the bank accounts with a total of £670,000.

Step 4: The man’s debts and funeral costs which total £20,000 can be deducted, leaving £650,000.

Step 5: No exemptions or reliefs apply.

Step 6: The man can claim his own RNRB. No transferrable RNRB applies as he does not have a spouse who pre-deceased. His main residence is a QRI and it passes to his son (a lineal descendent) absolutely. The amount of the RNRB is £150,000 (capped at the value of the QRI). The let property is not a QRI so an RNRB can be claimed. The total RNRB is: £150,000. Once applied to his estate, £650,000 - £150,000 = £500,000 remains.

Step 7: The man’s basic NRB is £325,000 (his cumulative total was 0 – so no deduction is made). No transferrable NRB applies as he did not have a pre-deceasing spouse.

0 - £325,000 @ 0%. Balance of £175,000 (£500,000 - £325,000) @ 40% = £70,000.

How well did you know this?
1
Not at all
2
3
4
5
Perfectly
73
Q

A man died yesterday leaving his whole estate to a life interest trust. The man’s spouse and children survive him. The man’s spouse is the life tenant of his will trust and his children are the remaindermen.

The man had not made any lifetime gifts.

His estate comprised his home worth £500,000 (free of mortgage) and various bank accounts totalling £120,000. The man’s debts and funeral expenses totalled £8,000.

In the tax year of the man’s death, the basic nil rate band is £325,000 and the main residence nil-rate band is £175,000.

How much inheritance tax will be payable on the man’s estate?

£244,800

£130,000

£78,000

£0

£114,800

A

£0

Correct
Correct: The whole of the man’s estate passes to a life interest trust in which his spouse is the life tenant. Spouse exemption applies to the whole estate and no inheritance tax is payable. The other options were incorrect because spouse exemption was not applied.

£114,800

Incorrect
Incorrect: Review your materials relating to inheritance tax. You have not considered the IHT implications of a person leaving their estate to a life interest trust and naming their spouse as the life tenant.

How well did you know this?
1
Not at all
2
3
4
5
Perfectly
74
Q

Is the following statement True or False?

“Estate Administration is the process of collecting in the deceased’s assets and distributing them to the correct beneficiaries”.

A

False
Correct
This was an incomplete statement. The administration process also includes other obligations, including payment of the deceased’s debts and administration expenses.

How well did you know this?
1
Not at all
2
3
4
5
Perfectly
75
Q

Is the following statement True or False?

“Where an executor is appointed by the will their power to act as PR derives from this appointment. Therefore, executors will not usually need to apply for a grant of representation.”

A

False
Correct
The authority for an executor to act as PR does derive from the will, whereas the authority for an administrator to act derives from the grant of representation. However, an executor will still need to apply for a grant of representation because many institutions will insist on seeing the grant as proof of the executor’s authority to act before releasing funds/assets to them.

How well did you know this?
1
Not at all
2
3
4
5
Perfectly
76
Q

Is the following statement True or False?

“Once the grant of representation is obtained the PRs have authority to collect in and deal with all the assets that the deceased owned when they died.”

A

False
Correct
The grant of representation confers power on the PRs to deal with the succession estate assets, ie those items passing under the will or under intestacy, and does not, for example, give PRs the right to have assets passing by survivorship transferred into their name.

How well did you know this?
1
Not at all
2
3
4
5
Perfectly
77
Q

True/False: An executor may also be a trustee of the estate they administer.

False

True

A

True

Correct
The executor of an estate may be expressly appointed in the role of trustee of any trust that arises following death, but that this is not always the case.

How well did you know this?
1
Not at all
2
3
4
5
Perfectly
78
Q

True/False: Acting as a PR confers a fiduciary duty and therefore a beneficiary should not act as PR because this will give rise to a conflict of interest.

False

True

A

False

Correct
The role of a PR is fiduciary in nature but there is no prohibition on a PR also being a beneficiary. There is no conflict of interest as the PR does not receive benefit in their capacity as PR.

How well did you know this?
1
Not at all
2
3
4
5
Perfectly
79
Q

True/False: A solicitor may be involved with the estate administration even if they have not been appointed as PR.

True

False

A

True

Correct
A solicitor may become involved if they are appointed as PR by a will or if the PRs instruct a solicitor to carry out administrative services.

How well did you know this?
1
Not at all
2
3
4
5
Perfectly
80
Q

Is the following statement true or false?

“If an estate has not been fully administered 12 months after the deceased’s death the PRs will be in breach of their duty under s.44 Administration of Estates Act 1925?”

A

False
Correct
Correct. There is no duty to complete the administration within the “executor’s year”. A PR is not obliged to distribute the estate before the one-year anniversary of death, although after this period they must be able to justify any delay.

How well did you know this?
1
Not at all
2
3
4
5
Perfectly
81
Q

Your client will be appointed as the PR of an estate. Your client has not yet made an application for a grant.

Which of the following most accurately describes the current statutory duties of your client?

A duty to report to HMRC about the estate assets and liabilities and pay any inheritance tax due.

A duty to collect in the deceased’s assets and administer them according to law.

A duty to report to HMRC about the estate assets and liabilities and pay any inheritance tax due and administer the estate according to the law.

There are no statutory duties that apply until the grant has been issued.

A duty to collect in the deceased’s assets, administer them according to law and keep estate accounts.

A

A duty to report to HMRC about the estate assets and liabilities and pay any inheritance tax due.

Correct
Correct. Prior to the grant the PR is not able to collect in and administer the deceased’s assets and the primary duty the PR has is to report to HMRC and pay the IHT, both of which must be completed before the grant is issued.

How well did you know this?
1
Not at all
2
3
4
5
Perfectly
82
Q

You are advising a client who has been appointed as the sole executor of his father’s estate. The executor has just obtained the grant of probate and does not act in a professional capacity.

The executor has asked whether he must complete the administration of the estate now he has started and whether he may buy one of the properties from the estate.

There are no relevant express provisions in the will.

Which of the following is correct?

The executor has a duty to administer the estate. The executor has the power as PR to sell estate assets and fiduciary duties do not apply because he is not acting as a trustee.

The executor has a duty to administer the estate. The executor has the power as PR to sell estate assets. If he acted as the purchaser he would be in breach of his fiduciary duty, even if a fair price was paid (unless consent of the court or beneficiaries was obtained).

The executor does not have to administer the estate but any administrative steps he does take must be done with due diligence. The executor has the power as PR to sell estate assets but if he acted as the purchaser, he would be in breach of his fiduciary duty even if a fair price was paid.

The executor does not have to administer the estate but any administrative steps he does take must be done with due diligence. The executor has the power as PR to sell estate assets and may act as the purchaser provided the price paid was full market value.

The executor has a duty to administer the estate. The executor has the power as PR to sell estate assets and may act as the purchaser provided the price paid was full market value and there is no loss to the estate.

A

The executor has a duty to administer the estate. The executor has the power as PR to sell estate assets. If he acted as the purchaser he would be in breach of his fiduciary duty, even if a fair price was paid (unless consent of the court or beneficiaries was obtained).

Correct
Correct. An executor who accepts office has a duty to administer the estate in full under s.25 AEA. A PR is subject to fiduciary duties which include the duty to avoid a position of conflict. Acting as both buyer and seller of the estate assets (without court or beneficiary authorisation) would place him in a position of conflict and would therefore breach that duty, even if there was no loss suffered to the estate and fair market value was paid.

How well did you know this?
1
Not at all
2
3
4
5
Perfectly
83
Q

By her will a testator left a legacy of £500 to her granddaughter (G), who was 16 at the date of T’s death. The will does not amend any of the statutory powers or include any express administrative provisions.

Assuming T’s PRs wish to distribute this amount as soon as possible, which of the following is the most appropriate course of action?

The PRs have the power to pay the money to G because she is over the age of 16.

The PRs cannot give the money to G directly but can appoint trustees of the legacy and pay the money to the trustees.

The PRs must wait until G reaches the age of 18 before distributing the legacy.

The PRs must wait until G reaches the age of 18 or marries before distributing the legacy.

The PRs have discretion to pay the money to G if they consider she is responsible enough to receive the money direct.

A

The PRs cannot give the money to G directly but can appoint trustees of the legacy and pay the money to the trustees.

Correct
Correct. By s.42 Administration of Estes Act 1925 the PRs may appoint trustees of a legacy and pay the amount of the legacy to the trustees, who hold on trust for the minor beneficiary. The trustees will provide confirmation of the receipt to the PRs and so the administration can be completed.

The other options were incorrect because: - There is no discretion to pay the money directly to a minor - The PRs do not have to wait until the minor reaches 18 or married - The power to pay the money to a beneficiary aged 16 or older must be conferred by the will and in this scenario we know that the will does not contain any administrative provisions

How well did you know this?
1
Not at all
2
3
4
5
Perfectly
84
Q

A testator died yesterday leaving an estate worth £3 million. The assets include £1 million cash in a current account (being the sale proceeds from a recent property sale). The current rate of interest is very low.

The PRs are concerned about their obligations in relation to the estate assets and believe the administration will not be completed for some time.

Which of the following statements is correct?

The PRs would only be subject to the statutory duty of care when exercising their power to invest the estate assets if they were acting in capacity as a trustee.

The PRs have a duty to invest the cash in the bank account to achieve a greater rate of return than the current rate of interest.

The PRs have complete discretion to invest cash in the bank account in any kind of assets as if they were absolutely entitled.

The PRs should retain the bank account as it is because this was the original asset owned by the deceased.

The statutory power to invest does not apply to the PRs unless an express trust has been created by the testator’s will.

A

The PRs have a duty to invest the cash in the bank account to achieve a greater rate of return than the current rate of interest.

Correct
Correct. As the administration is likely to take time the PRs have a duty to maintain the estate and actively invest to achieve better rates of return than are being paid in respect of the current account.

The other answers were incorrect because: - Although the general power of investment gives the PRs the power to invest as if they were absolutely entitled, their discretion is not absolute. They must invest having regard to the standard investment criteria and must seek and follow appropriate advice. - The statutory power to invest (s.3 Trustee Act 2000) does apply to PRs - There is no obligation to retain original assets - The statutory duty of care applies to the exercise of investment powers and is applicable to PRs as well as trustees.

How well did you know this?
1
Not at all
2
3
4
5
Perfectly
85
Q

A testator died recently leaving a cash legacy of £10,000 to his friend.

The executors have identified a small portfolio of quoted shares in the estate currently worth £8,000 that they would like to transfer to the friend in part satisfaction of his legacy. The shares have not been specifically gifted to anyone under the will and the residue of the estate passes to the testator’s spouse.

The will does not contain any express administrative provisions.

Assuming the shares do not subsequently increase in value to more than £10,000, which of the following would be the most appropriate advice for the executors?

Transfer the shares to the friend plus a cash sum equivalent to the difference in value between the shares (at the date of appropriation) and £10,000.

Transfer the shares to the friend.

Transfer the shares to the friend, but only if the spouse consents.

Transfer the shares to the friend plus a cash sum of £2,000.

Transfer £10,000 to the friend. They have no power to transfer shares in partial satisfaction of the cash legacy.

A

Transfer the shares to the friend plus a cash sum equivalent to the difference in value between the shares (at the date of appropriation) and £10,000.

Correct
Correct. S.41 AEA provides the PRs with the power to appropriate assets in or towards a beneficiary’s entitlement. As the value of the shares is less than the legacy the PRs may appropriate the shares and then make a balancing cash transfer of the difference. The consent of the residuary beneficiary is not required as the shares were not specifically given to them.

How well did you know this?
1
Not at all
2
3
4
5
Perfectly
86
Q

You are advising a creditor of an estate who is concerned they will not receive money they are owed.

Assuming there are sufficient funds available to meet all debts and legacies but the creditor is not paid, which of the following is correct?

The creditor may bring a devastavit claim against the PRs on the grounds of maladministration.

There is no cause of action that can be brought against the PRs as their duties are only owed to the beneficiaries of an estate.

The creditor may bring a claim against the PRs on the grounds of breach of fiduciary duty.

If the creditor cannot bring a claim against the PRs there is no alternative option for them to pursue.

The creditor may only bring a claim against the PRs if the administration of the estate is not yet complete and enough money remains to pay the debt.

A

The creditor may bring a devastavit claim against the PRs on the grounds of maladministration.

Correct
Correct. A claim against a PR is referred to as a devastavit. Where the estate has not been distributed to those who are entitled to it the grounds for a claim would be maladministration.

The other options were incorrect because:

  • Duties are owed to both beneficiaries and creditors
  • The wrongdoing would not amount to a breach of fiduciary duty
  • An unpaid creditor could also claim against the estate beneficiaries
  • The PRs owe a personal liability to a creditor who was not paid from the estate
How well did you know this?
1
Not at all
2
3
4
5
Perfectly
87
Q

The PRs of an estate are due to distribute assets to the deceased’s children. They have been in contact with the children of the deceased’s marriage but are concerned that the testator could have fathered other children who the PRs and immediate family are unaware of.

Which of the following would help protect the PRs from personal liability should any additional children approach the PRs for a share of the estate after it has been distributed?

Application to court for an administration action.

Making a payment into court.

Compliance with the s.27 Trustee Act 1925 notice procedure.

Retaining funds and delaying the completion of the administration indefinitely.

Application to court for a Benjamin Order.

A

Compliance with the s.27 Trustee Act 1925 notice procedure.
Correct
Correct. S.27 notices are intended to help protect the PRs from personal liability to unknown creditors and beneficiaries. The other options are possible steps a PR might take if they know about a beneficiary but cannot locate them.

incorrect
Making a payment into court.
Incorrect
Incorrect. Refer to your materials relating to PR protection. A payment into court would not be useful in relation to an unknown beneficiary.

How well did you know this?
1
Not at all
2
3
4
5
Perfectly
88
Q

You are advising a lay executor about their duties. The deceased left a will that includes an express clause stating “no executor shall be personally liable for any action or inaction taken in the course of the administration”.

Which of the following is correct?

The clause is effective and the executor will not have any personal liability.

Despite the express clause in the will the executor would remain liable for fraud.

The clause is not required because the executor is not acting in a professional capacity.

The clause is not effective as it is not possible to limit the liability of an executor.

The clause would not protect the executor from loss resulting from an honest mistake.

A

Despite the express clause in the will the executor would remain liable for fraud.

Correct
Correct. A clause which limits the executor’s liability is enforceable but not to the extent it attempts to avoid liability for fraud.

The other options were incorrect because:

  • it is not possible to remove all personal liability irrespective of how the executor behaves.
  • both lay and professional executors have personal liability
  • limitation of liability clauses are enforceable in respect of an honest mistake
How well did you know this?
1
Not at all
2
3
4
5
Perfectly
89
Q

A deceased died leaving a valid will that appoints their brother as executor. The deceased’s will leaves a property to their spouse and the rest of the estate to their two minor children.

The deceased is survived by their brother and children. Their spouse has pre-deceased and the deceased has died partially intestate.

Which is the correct grant?

Grant of letters of administration (with will)

Grant of letters of administration

Grant of probate

Grant of double probate

Grant of Representation

A

Grant of probate

Correct
Correct. The deceased left a will that appoints an executor. It does not matter that the deceased died partially intestate and only one executor is required even where minor interests arises.

How well did you know this?
1
Not at all
2
3
4
5
Perfectly
90
Q

A deceased died leaving a valid will that appoints their brother and sister as joint executors.

The will gives all of the estate to the deceased’s brother.

The deceased is survived by their sister and she is willing to act. The deceased’s brother pre-deceased him so some of the estate will pass under the intestacy rules.

Which is the correct grant?

Grant of probate

Grant of letters of administration

Grant of letters of administration (with will)

Grant of Representation

Grant of double probate

A

Grant of probate

Correct
Correct. The deceased left a will that appoints an executor. It does not matter that one of the executors has pre-deceased, the remaining executor can apply. It does not matter that the will fails to dispose of all of the deceased’s assets.

How well did you know this?
1
Not at all
2
3
4
5
Perfectly
91
Q

A testator died leaving a valid will appointing their father as executor. The will gives all of the estate to the testator’s brother. The testator is survived by their brother, but their father has pre-deceased.

Which is the correct grant?

Grant of letters of administration (with will)

Grant of probate

Grant of double probate

Grant of letters of administration

Grant of Representation

A

Grant of letters of administration (with will)

Correct
Correct. The deceased left a will which does not appoint any executors who can act.

How well did you know this?
1
Not at all
2
3
4
5
Perfectly
92
Q

You are instructed to assist with the administration of an estate which contains the following assets:

  • Property owned as tenants in common
  • Building Society account with £2,000
  • Furniture
  • Life policy where proceeds of £100,000 are payable to the estate

Which of the following correctly states whether or not a grant of representation is needed to administer these assets?

A grant is required to deal with the property and the life policy only.

A grant is required to deal with the building society account, furniture, and life policy proceeds only.

A grant is required to deal with the property, building society account and life policy proceeds only.

A grant is required to deal with all of the assets.

The estate could be administered without the need for a grant.

A

A grant is required to deal with the property and the life policy only.

Correct
Correct. A grant is required to deal with assets within the succession estate unless an exception applies. A property owned as tenants in common, and a life policy where proceeds are payable to the estate, are items that fall within the succession estate and will require the PRs to take out a grant to deal with them. The account with a balance of less than £5,000 could be collected under the exception in the Administration of Estates (Small Payments) Act 1965 and no grant is required to deal with household possessions.

incorrect
A grant is required to deal with all of the assets.

Incorrect
Incorrect. Review your materials and ensure you appreciate which items may be dealt with without the need for a grant. Consider the treatment of building society accounts with less than £5,000 and personal household possessions.

How well did you know this?
1
Not at all
2
3
4
5
Perfectly
93
Q

You are instructed to assist with the administration of an estate which contains the following assets:

  • Property owned as joint tenants - Building society account with £2,000 - Furniture - Life policy where proceeds have been nominated for the deceased’s spouse

Which of the following correctly states whether or not a grant of representation would be required to deal with the assets?

The estate could be administered without the need for a grant.

A grant is required to deal with the property, building society account and life policy proceeds only.

A grant is required to deal with the property and the life policy only.

A grant is required to deal with all of the assets.

A grant is required to deal with the building society account, furniture and life policy proceeds only.

A

The estate could be administered without the need for a grant.

Correct
Correct. A property owned as joint tenants does not fall within the succession estate and passes under survivorship rules. The life policy proceeds do not fall within the succession estate and may be paid directly to the deceased’s spouse. The building society account with a balance of less than £5,000 could be collected under the Administration of Estates (Small Payments) Act 1965 and no grant is required to deal with household possessions.

How well did you know this?
1
Not at all
2
3
4
5
Perfectly
94
Q

You are instructed to assist with the administration of an estate which contains the following assets:

  • Property owned as tenants in common
  • Bank account with £60,000
  • Furniture
  • Life policy where proceeds have been nominated for the deceased’s spouse

Which of the following correctly states whether or not a grant of representation would be required to deal with the assets?

A grant is required to deal with all of the assets.

A grant is required to deal with the property and the bank account only.

The estate could be administered without the need for a grant.

A grant is required to deal with the bank account, furniture, and life policy proceeds only.

A grant is required to deal with the property and the life policy only.

A

A grant is required to deal with the property and the bank account only.

Correct
Correct. The life policy proceeds do not fall within the succession estate and may be paid directly to the deceased’s spouse and no grant is required to deal with household possessions. The deceased’s share of property owned as tenants in common will pass as part of the succession estate and a grant is required. As there is more than small amount in the bank account this would usually require a grant to be provided before the bank would release the funds.

How well did you know this?
1
Not at all
2
3
4
5
Perfectly
95
Q

A deceased owned the following assets among their personal possessions:

  • Dining table (worth approximately £200 if sold but would cost £1,500 to replace)
  • Painting worth approximately £300 (sale and replacement value are similar)
  • Diamond ring worth approximately £6,000 (sale and replacement value are similar)

Regarding the value of the assets, which of the following would be the most appropriate?

The PRs should obtain a professional valuation for the dining table and ring but may record an estimated value for the painting.

The PRs should obtain a professional valuation for all of the assets.

The PRs should obtain a professional valuation for the ring but may record estimated values for the dining table and the painting.

The PRs may record estimated values for all of the assets.

The PRs should obtain a professional valuation for the dining table and the painting but may record an estimated value for the ring.

A

The PRs should obtain a professional valuation for the ring but may record estimated values for the dining table and the painting.

Correct
Correct. The sale value of the assets at the date of death is relevant. Estimated figures may be used for ‘low value’ chattels, which would include the dining table (£200 sale value) and the painting (£300). The ring is a single item worth more than £500 so a professional valuation should be obtained.

How well did you know this?
1
Not at all
2
3
4
5
Perfectly
96
Q

You have been appointed as an executor of an estate jointly with the deceased’s adult son. The deceased’s assets comprised a car, a bank account and a small partnership interest in their sole name. The deceased owned his home and personal chattels as joint tenants with his spouse.

The deceased’s surviving spouse has already registered the death and is currently arranging the funeral.

Which of one of the following does not need to be carried out by the executors?

Transferring the house and chattels into the name of the executors.

Valuing the deceased’s debts.

Locating the original will.

Notifying assets holders of the deceased’s death.

Arranging a professional valuation of the business.

A

Transferring the house and chattels into the name of the executors.

Correct
Correct. The joint tenant property passes automatically to the survivor under the survivorship rules and does not form part of the succession estate for administration purposes and would not be transferred into the names of the executors.

incorrect
Notifying assets holders of the deceased’s death.

Incorrect
Incorrect. Refer to your materials regarding the obligation of the PRs in respect of assets passing outside of the succession estate.

How well did you know this?
1
Not at all
2
3
4
5
Perfectly
97
Q

You have been instructed by the executors of an estate to assist with the administration.

The deceased owned a property jointly with his brother and held a joint bank account with his son.

When preparing a schedule of assets in respect of the deceased’s estate which of the following is correct?

The deceased will be deemed to own 50% of the property. Further enquiries are needed to establish whether or not the deceased held a 50% share of the bank account.

Further enquiries are needed to establish whether or not the deceased held a 50% share of the property and joint bank account.

The deceased will be deemed to own 50% of the property and 100% of the bank account (because this passes by survivorship).

The deceased will be deemed to own 50% of the bank account. Further enquiries are needed to establish whether or not the deceased held a 50% share of the property.

The deceased will be deemed to own 50% of both the property and the joint bank account.

A

Further enquiries are needed to establish whether or not the deceased held a 50% share of the property and joint bank account.

Correct
Correct. Where an asset is owned jointly (whether a property or bank account) enquiries are needed to establish what the terms of that co-ownership are. You cannot assume that the deceased held a 50% share or that all joint property is owned as joint tenants.

The other options were incorrect because it was assumed that the deceased owned 50% of one or both of the assets.

incorrect
The deceased will be deemed to own 50% of both the property and the joint bank account.

Incorrect
Incorrect. Review your materials regarding the valuation of the estate. Is it always the case that 50% ownership can be presumed where the deceased owned assets with another person?

How well did you know this?
1
Not at all
2
3
4
5
Perfectly
98
Q

A testator died leaving a will that appoints their civil partner, sister and brother jointly as executors.

The testator’s civil partnership was subsequently dissolved and the testator’s sister has pre-deceased.

Which of the following correctly states who is entitled to apply for the grant of representation?

The brother

The brother and the sister’s own executor

The brother or civil partner

The brother and civil partner

The brother or the sister’s own executor

A

The brother

Correct
Correct. The civil partner cannot act because of the effect of s.18C Wills Act 1837. The sister’s executor cannot act because the chain of representation does not apply in this scenario. The brother may apply as the sole surviving executor.

How well did you know this?
1
Not at all
2
3
4
5
Perfectly
99
Q

A testator died leaving a will that appoints A, B, C, D and E as joint executors.

A is aged 17. B, C, D and E are adults.

Which of the following most accurately explains who will take out the grant of probate?

A, B, C, D and E

B, C , D and E. Power cannot be reserved to A because they are a minor.

A, B, C, D as the first named executors to the maximum of four.

B, C, D and E. Power may be reserved to A who could apply once they reach the age of 18.

B, C, D and E. A cannot act because an appointment of a minor is invalid.

A

B, C, D and E. Power may be reserved to A who could apply once they reach the age of 18.

Correct
Correct. A minor may be appointed by will but cannot act until they reach the age of 18. Power may be reserved to them until this time. A maximum of four executors may apply, so B, C, D and E can be named on the grant.

How well did you know this?
1
Not at all
2
3
4
5
Perfectly
100
Q

For which of the following estates might the chain of representation apply?

  • Testator A appoints a sole executor who survives A but dies before taking out the grant in A’s estate.
  • Testator B appoints two executors, one of whom dies after taking out the grant in B’s estate. The other wishes to act.
  • Testator C appoints an executor who has died before C.
  • Testator D appoints an executor who dies intestate after taking out the grant in D’s estate.

B’s estate

None of them

A’s estate

C’s estate

D’s estate

A

None of them

Correct
Correct. The chain of representation only applies where a sole surviving executor, who has taken out a grant of probate, dies before administering the estate and appoints their own executor in turn.

The others were incorrect because:

Estate A: the executor died before taking the grant. Estate B: there is a surviving executor who will act Estate C: the executor did not survive the testator Estate D: the sole surviving executor died intestate so cannot have appointed an executor for their own estate.

How well did you know this?
1
Not at all
2
3
4
5
Perfectly
101
Q

Your client has died intestate and is survived by their spouse and two children, one aged 20 and the other aged 15. The deceased’s spouse and children are all entitled to inherit from the estate under the intestacy rules.

Which one of the following statements is correct in relation to the administration of the estate?

The testator’s eldest child can apply on their own to act as sole administrator.

The testator’s spouse can apply on their own to act as sole administrator because they will be the primary beneficiary under the deceased’s intestacy.

The testator’s spouse can apply with either of the children to act as joint administrators.

The testator’s spouse cannot act as sole administrator because one of the two children is a minor.

The testator’s spouse cannot act as sole administrator because the role of administrator is a fiduciary one and they will be in a position of conflict with the children.

A

The testator’s spouse cannot act as sole administrator because one of the two children is a minor.
Correct
Correct. Where administrators are needed for an estate a minimum of two must be appointed where any minor interest arises (s.114 Senior Courts Act 1981). There will be a minor interest under the deceased’s intestacy because one of the children is only 15. The other options were wrong because: - Two administrators are required - A minor cannot act - There is no conflict where a PR is also one of the beneficiaries

incorrect
The testator’s spouse can apply on their own to act as sole administrator because they will be the primary beneficiary under the deceased’s intestacy.

Incorrect. Refer to your materials regarding the number of PRs required where a minor interest arises.

How well did you know this?
1
Not at all
2
3
4
5
Perfectly
102
Q

A deceased died leaving a valid will that appoints their adult brother as executor. The deceased’s will leaves a cash lump sum to the deceased’s two adult children and the residue of the estate to their spouse.

The deceased is survived by their spouse and children. The deceased’s brother has pre-deceased.

There are no trustees of the residuary estate.

Who should apply for the grant of representation?

The deceased’s spouse under NCPR 22 because they were married to the deceased.

The deceased’s spouse under NCPR 20 because they are the residuary beneficiary.

The PR of the deceased’s brother.

The deceased’s spouse and at least one of the children – 2 PRs are required.

The deceased’s spouse and both children because they are all estate beneficiaries.

A

The deceased’s spouse under NCPR 20 because they are the residuary beneficiary.

Correct
Correct. The deceased left a will that does not appoint an executor who is able to act (the appointed brother has died before the testator). Therefore, an administrator will need to be appointed under NCPR 20. Only one person is required as there are no life or minor interests and the surviving spouse has the best right to apply, as the residuary beneficiary.

How well did you know this?
1
Not at all
2
3
4
5
Perfectly
103
Q

A man died intestate. The man never married or entered a civil partnership.

He has two sons (20 and 18) and a daughter (15) from a long-term relationship with a woman.

The children are the only beneficiaries of the man’s estate under his intestacy.

Which of the following most accurately explains who may apply for a grant?

Both sons and the woman (on behalf of the daughter) although applications made by the sons will be preferred over the woman’s.

Both sons and the woman (on behalf of the daughter). All three have equal entitlement.

Any of the children.

The woman on her own behalf.

Either son.

A

Both sons and the woman (on behalf of the daughter) although applications made by the sons will be preferred over the woman’s.

Correct
Correct. Under Rule 22 (1) (b) all three children have equal entitlement to apply. However, the daughter is a minor and cannot apply on her own behalf. The woman can apply on behalf of the daughter but the court will prefer an application by an adult in their own capacity (i.e. the sons) over an application made on behalf of a minor.

incorrect
Both sons and the woman (on behalf of the daughter). All three have equal entitlement.

Incorrect. Where an application is made on behalf of a minor, does the court treat this equally to one made by an adult in their own capacity within the same category of entitlement?

How well did you know this?
1
Not at all
2
3
4
5
Perfectly
104
Q

A testator made a valid will appointing their civil partner, mother and son (17) jointly as executors.

The testator’s civil partnership was dissolved after the will was executed.

The testator’s mother does not have mental capacity.

There are no relevant express provisions in the will.

Who may apply for a grant of probate?

The civil partner only

Civil partner and the mother only

The mother and the son

All three executors named in the will

None of the executors named in the will

A

None of the executors named in the will

Correct
Correct. The testator’s former civil partner cannot act (s.18C Wills Act 1837) unless the will expressly states otherwise (which it does not). A minor and someone who lacks capacity are both unable to act.

How well did you know this?
1
Not at all
2
3
4
5
Perfectly
105
Q

A deceased died intestate and their estate passes to their spouse and two children (one of whom is a minor).

The spouse has not applied for a grant but has been dealing with the estate administration and has arranged for the sale of the deceased’s property. However, the spouse no longer wishes to be involved with the administration.

Which of the following would you advise?

Renunciation

Acting alone

Reserving power

Doing nothing

A

Renunciation

Correct
Correct. The spouse with an entitlement to apply as administrator may renounce that right even if she has intermeddled. The other options were incorrect because: - Reserving power is not an option for administrators - She cannot act alone where there is a minor interest - Doing nothing is not a viable option

How well did you know this?
1
Not at all
2
3
4
5
Perfectly
106
Q

A testator’s will appointed their spouse and sister as joint executors.

The testator’s sister reserved power and a grant of probate was issued to the deceased’s spouse. The testator’s spouse has not yet completed the administration.

Which of the following most accurately describes the sister’s entitlement to act in the administration?

The sister can act as PR under the original grant issued in the spouse’s name because the sister is named in the will.

The sister can apply for a grant of probate.

The sister can act under the original grant provided it mentions that notice of the reservation was given to the testator’s spouse at the time the grant was issued.

The sister can apply for a grant of double probate.

The sister cannot apply for a grant.

A

The sister can apply for a grant of double probate.

Correct. The 2nd grant issued in the estate is a grant of double probate. The other options were incorrect because;

  • the new grant was incorrectly stated

-the sister has no entitlement to act under the grant originally issued in the sole name of the spouse

  • reserving power does not preclude a later application when the administration is still ongoing

-the original grant will have referred to notice having been given to the sister who reserved power

How well did you know this?
1
Not at all
2
3
4
5
Perfectly
107
Q

A deceased died without leaving a will. The estate is shared between their spouse and three children (two of whom are minors). No IHT is payable. A paper form will be used for this estate.

Which one of the following statements most accurately describes the requirements when applying for the grant?

The PA1P is required and an administrator will be appointed under NCPR 20.

The PA1A is required and at least two administrators will be appointed under NCPR 22.

The PA1P is required and at least two administrators will be appointed under NCPR 22.

The PA1A is required and an administrator will be appointed under NCPR 22.

The PA1A is required and at least two administrators will be appointed under NCPR 20.

A

The PA1A is required and at least two administrators will be appointed under NCPR 22.

Correct
Correct. Where the deceased died without making a will the correct form is the PA1A. Two administrators are required because there will be at minor interests.

How well did you know this?
1
Not at all
2
3
4
5
Perfectly
108
Q

Which of the following best describes what an applicant under NCPR 22 will always need to do when applying for a grant?

Confirm their familial relationship with the deceased

Confirm their beneficial entitlement to the estate

Clear-off other applicants

Confirm the validity of the deceased’s will

Confirm their familiar relationship with the deceased and their beneficial entitlement to the estate

A

Confirm their familiar relationship with the deceased and their beneficial entitlement to the estate

Correct
Correct. If NCPR 22 applies the deceased died intestate. The entitlement to apply derives from the relationship with the deceased and their right to receive a share of the estate under intestacy, so both must be stated. The other options were incorrect because:

  • they were either incomplete or: - It is not always necessary to clear-off anyone else e.g. when the person with the greatest entitlement will be applying. - If NCPR 22 applies the deceased did not leave a valid will
How well did you know this?
1
Not at all
2
3
4
5
Perfectly
109
Q

Which of the following is correct with regards an application for a grant of probate?

The application cannot be made online.

The grant can only be issued in the name of an executor as it appears in the will.

The value of the deceased’s taxable estate will be noted on the grant.

The application only needs to refer to those executors named in the will who are applying.

The grant can be issued in the deceased’s true name and also any common name in which they held assets.

A

The grant can be issued in the deceased’s true name and also any common name in which they held assets.

Correct
Correct. The full name of the deceased should be included in the application and where the deceased owned assets in a common name this should also be stated so the grant can be issued in both names. The other options were incorrect because:

  • The grant should be issued in the true name of the executors- if this is not how their name appears in the will, the discrepancy can be explained
  • The application should refer to all executors named in the will- and if any are not applying the reason for this should be explained
  • Most probate applications can be made online
  • It is the value of the deceased’s succession (not taxable) estate which appears on the grant
How well did you know this?
1
Not at all
2
3
4
5
Perfectly
110
Q

A testator died leaving a valid will appointing two executors. One of them will renounce. Inheritance tax has been paid. Which of the following most accurately states what needs to be submitted to the probate registry when making an application for the grant?

Certified copy of the will

Certified copy of the will and form of renunciation

Original will and form of renunciation

Original will, form of renunciation and IHT 400

Certified copy of the will and IHT 400

A

Original will and form of renunciation

Correct
Correct. When an executor renounces power they will completed a form of renunciation and this must be submitted along with the application for probate. The original not a copy will must also be provided. The other options were incorrect because: - The original will is needed - The IHT 400 is sent to HMRC not probate registry

incorrect
Original will, form of renunciation and IHT 400

Incorrect
Incorrect. The IHT 400 is sent to HMRC not to the probate registry.

How well did you know this?
1
Not at all
2
3
4
5
Perfectly
111
Q

A deceased died intestate. The deceased’s surviving spouse inherits the whole of the estate free of inheritance tax (this estate is excepted) and is applying for the grant of representation alone.

Which of the following most accurately states what needs to be submitted to the probate registry when making an application for the grant?

Certified copy of the will and IHT 400

Nothing

Original will and the probate registry fee

The probate registry fee

IHT 400

A

The probate registry fee

Correct. This is sent to the probate registry as part of the grant application. The other options were incorrect because there was no will (the deceased died intestate) and the IHT 400 is not relevant.

How well did you know this?
1
Not at all
2
3
4
5
Perfectly
112
Q

An executor takes out a grant of probate and then decides to appoint an attorney to assist with the administration.

Will the power of attorney be sent to the probate registry?

A

No
Correct
Correct. The executor has already taken out the grant in their own name. The probate registry would only be sent the power of attorney if the grant was being issued in the name of the attorney i.e. the executor was delegating their right to apply.

incorrect
yes
Incorrect
Incorrect. The executor has already taken out the grant in their own name. The probate registry would only be sent the power of attorney if the grant was being issued in the name of the attorney i.e. the executor was delegating their right to apply.

How well did you know this?
1
Not at all
2
3
4
5
Perfectly
113
Q

A woman died leaving a will appointing her brother as her executor. The woman’s will contained the following standard attestation clause:

“Signed by the above named [woman’s name] in our joint presence and then by us in hers”

The will was executed in accordance with s 9 Will Act 1837.

The woman was illiterate and unable to read her will so it was read out loud to her before she signed it.

Which of the following affidavits may be required to support the application for a grant of probate?

An affidavit of due execution confirming compliance with s 9 Wills Act 1837.

None – the will is invalid and cannot be submitted to the probate registry.

An affidavit of plight and condition.

None - no affidavit evidence is required on these facts.

An affidavit of due execution confirming knowledge and approval.

A

An affidavit of due execution confirming knowledge and approval.

Correct
Correct: On the facts the will was correctly executed. However, as the woman could not read her will there is no presumption that she knew and approved of its contents. This presumption could have been raised by a specially worded attestation clause – however, this was not done and an affidavit confirming the will was read out loud to her and that she understood it may be needed.

How well did you know this?
1
Not at all
2
3
4
5
Perfectly
114
Q

A woman died leaving a will appointing her spouse as her executor. The woman’s will contained the following standard attestation clause:

“Signed by the above named [woman’s name] in our joint presence and then by us in hers”

The date and two signatures appear as part of the attestation.

Which of the following affidavits may be required to support the application for a grant of probate?

None – the will is not valid so cannot be submitted to the probate registry

None - no affidavit evidence is required and the application can proceed as standard

An affidavit of plight and condition

An affidavit of due execution confirming knowledge and approval

An affidavit of due execution confirming compliance with s 9 Wills Act 1837

A

None – the will is not valid so cannot be submitted to the probate registry

Correct
Correct. The will has not been executed in accordance with the requirements of s 9 WA as there are only two signatures on it not three (a will must be signed by the testator and two witnesses). If a will is not valid it cannot be admitted to the probate registry.

Incorrect. The will has only been signed by two people. An affidavit cannot be submitted to suggest otherwise.

How well did you know this?
1
Not at all
2
3
4
5
Perfectly
115
Q

A client brings you a home-made will made by her mother. The client’s mother has recently died.

The client’s mother kept the fact that she made a will secret and the only people who knew about it were the two witnesses.

The will is well drafted and contains a standard attestation clause. You are confident that the will was signed in accordance with the execution requirements in s 9 Wills Act 1837. However, the will does not contain a date.

The client tells you that both witnesses to the will died before the client’s mother.

Which of the following is correct?

Affidavit of due execution may be required.

An affidavit as to alterations may be required.

An affidavit of due execution cannot be sworn.

No affidavit evidence is required and the probate application can proceed as standard.

An affidavit of plight and condition may be required.

A

An affidavit of due execution cannot be sworn.

Correct
Correct. An affidavit of due execution can be used to confirm the date of a will. However, it must be signed by someone who can swear under oath what this date in fact was. If the only people who knew about this will have pre-deceased there is no suitable deponent.

incorrect
Affidavit of due execution may be required.

Incorrect. Refer to your materials on affidavits and consider who would sign the affidavit.

How well did you know this?
1
Not at all
2
3
4
5
Perfectly
116
Q

A testator dies leaving a will. In his will he leaves his house (worth £600,000), which is subject to a mortgage (£50,000) to his brother, a legacy of £3,000 to his friend and the residue of his estate to his mother. The testator’s assets are worth £50,000 (excluding the house) and he had unsecured debts of £2,000.

The will effectively distributes the whole of T’s estate and is silent with regards the payment of debts.

Which of the following statements is correct?

The unsecured debts will be paid out of the £3,000 set aside for the legacy to T’s friend.

Only the unsecured debts will be repaid from the residue.

The mortgage and unsecured debts will be paid from the sale proceeds of the house.

The whole of the estate will bear the burden of the mortgage and unsecured debts and each beneficiary’s entitlement will abate proportionately.

The mortgage and unsecured debts will be repaid from residue.

A

Only the unsecured debts will be repaid from the residue.

Correct
Correct. T’s brother will inherit the house subject to the mortgage (the mortgage will not be repaid using other assets in the estate) – s.35 AEA 1925.

The unsecured debts will be paid in the statutory order prescribed in Sch 1 Part II AEA – the application of which directs the repayment of debts from the residue (as there is no undisposed of part of the estate). The other options were incorrect because:

  • The legacy to T’s friend is unaffected
  • The house is inherited subject to the mortgage charge and the mortgage debt is not repaid from other estate assets – the value of the house is sufficient to meet the whole of the secured loan
  • The house is only subject to the secured debts
  • The burden of debts is not shared proportionately between the whole of the estate
How well did you know this?
1
Not at all
2
3
4
5
Perfectly
117
Q

A woman dies leaving an estate worth £50,000. By her will the woman leaves £20,000 to her mother and the residue to her sister.

When she died the woman has secured debts of £10,000 and unsecured debts of £20,000.

Which one of the following is correct?

The woman’s estate is insolvent because there are insufficient funds to pay all of her debts and legacies.

The woman’s estate is solvent because there are sufficient funds to pay her unsecured debts and legacies. The secured debts do not need to be considered as these are tied to specific estate assets.

The woman’s estate is solvent because there are sufficient funds to pay her secured debts. It does not matter if the unsecured debts or legacies cannot be paid in full.

The woman’s estate is insolvent because after the debts and cash legacy are paid there is nothing left in the residue to pay to her sister.

The woman’s estate is solvent because there are sufficient funds to pay all of her debts. It does not matter if the legacies cannot be paid in full.

A

The woman’s estate is solvent because there are sufficient funds to pay all of her debts. It does not matter if the legacies cannot be paid in full.
Correct
Correct. An estate is solvent if all the debts, liabilities and expenses can be paid. It is irrelevant whether or not the legacies (pecuniary, residuary or otherwise) can be paid.

The other options were incorrect because to be a solvent estate all debts must be payable from the estate in full not just secured/unsecured debts.

incorrect
The woman’s estate is solvent because there are sufficient funds to pay her unsecured debts and legacies. The secured debts do not need to be considered as these are tied to specific estate assets.
Incorrect
Incorrect. Refer to your materials regarding the payment of the deceased’s debts and review the definition of a solvent estate. Although the burden of secured debts falls primarily to the charged assets, all debts must be payable from the estate assets to be solvent.

How well did you know this?
1
Not at all
2
3
4
5
Perfectly
118
Q

The executors of an estate have sought your advice. They need to sell an item from the residuary estate to raise a cash sum of £10,000 to repay a unsecured debt. They have identified the following residuary assets as potentially suitable:

  • Painting (probate value £15,000, current value £30,000)
  • Car (probate value £10,000, current value £10,000)
  • Collection of jewellery (probate value £12,000, current value £12,000)
  • Motorbike (probate value £4,000, current value £4,000)

One of the residuary beneficiaries has asked the executors whether they can have the car as part of their entitlement.

Which of the following would be the best advice?

It doesn’t matter which item they sell provided the executors fulfil their legal duty to the estate creditors.

The executors should sell the car.

The executors should sell the painting.

The executors should sell the jewellery.

The executors should sell the motorbike.

A

The executors should sell the jewellery.

Correct
Correct. This would be the best option. While the executors have a legal duty to repay the deceased’s debts, and provided they identify items in accordance with the statuary order, they are not legally obliged to sell one item in preference to another, there are practical considerations which they should take into account.

The other items would be less suitable because:

  • the painting has increased significantly in value and may result in a capital gains tax charge
  • a beneficiary has specifically asked for the car, and although the executors don’t have to agree with the request, in this case there is no reason not to because alternative assets can be sold
  • the motorbike would not provide the cash sum that they need.
How well did you know this?
1
Not at all
2
3
4
5
Perfectly
119
Q

The deceased acquired an asset when it was worth £10,000. The asset increased in value during the deceased’s lifetime and was worth £40,000 when the deceased died. The PRs needed to sell the asset a few months after the deceased died and have just sold the item for £45,000.

What is the value of the gain made by the PRs?

None – the gain is treated as being made by the beneficiary who will inherit the asset.

£5,000

£35,000

£30,000

£45,000

A

£5,000

Correct
Correct. The PRs are treated as having made a gain equal to the difference in value between the date of death value (£40,000) and the sale proceeds (£45,000).

The other options were incorrect because:

  • Gains accrued during the deceased’s lifetime were included
  • Sale proceeds are not equivalent to the ‘gain’
  • The PRs made the sale not the beneficiary – so the gain is attributed to the PRs
How well did you know this?
1
Not at all
2
3
4
5
Perfectly
120
Q

In the tax year of death a testator received rental income of £2,000 in respect of which income tax was due but had not been paid.

During the administration the estate received savings income of £50 as bank interest but no other income.

Which of the following is correct?

The PRs should complete a tax return for the deceased for the tax year of death (to include £2,000 rental income) but do not need to account to HMRC for the estate savings income.

The PRs should account to HMRC for tax due in respect of the estate savings income but do not need to worry about the deceased’s untaxed rental income as this does not need to be paid following death.

The PRs should complete a tax return for the deceased for the tax year of death (to include £2,000 rental income) and should also account to HMRC for tax due in respect of the estate savings income.

The PRs should complete a tax return for the deceased for the tax year of death (to include both £2,000 rental income and £50 savings income).

The PRs are not required to account to HMRC for either the deceased’s rental income or the estate savings income.

A

The PRs should complete a tax return for the deceased for the tax year of death (to include £2,000 rental income) but do not need to account to HMRC for the estate savings income.
Correct
Correct. The deceased’s PR must account to HMRC in respect of the deceased’s untaxed rental income for the tax year of death. They are not however required to account to HMRC for the estate savings income as this was bank interest of under the threshold of £100, and there is no other estate income.

incorrect
The PRs should complete a tax return for the deceased for the tax year of death (to include £2,000 rental income) and should also account to HMRC for tax due in respect of the estate savings income.

Incorrect
Incorrect. Refer to your notes on PR responsibilities for IT and CGT. There is no requirement to account to HMRC for the estate income if this comprises solely bank interest and is less than £100.

How well did you know this?
1
Not at all
2
3
4
5
Perfectly
121
Q

During the administration of an estate the PRs transfer a piano to the deceased’s mother and some shares to the deceased’s brother.

The probate value of the piano was £1,200 and the value at the date of transfer was £2,000. The deceased’s mother has just sold the piano for £1,800.

The probate value of the shares was £3,500 and their value at the date of transfer was £3,000. The deceased’s brother has just sold them for £2,500.

Which of the following is correct?

The PRs and both the deceased’s mother and brother have made a loss.

The PRs have made neither a gain nor a loss. The deceased’s mother and brother have made a loss.

The PRs have made a gain. The deceased’s mother and brother have made a loss.

The PRs and the deceased’s mother has made a gain. The deceased’s brother has made a loss.

The PRs have made neither a gain nor a loss. The deceased’s mother has made a gain and the deceased’s brother has made a loss.

A

The PRs have made neither a gain nor a loss. The deceased’s mother has made a gain and the deceased’s brother has made a loss.

Correct
Correct. Where assets are transferred to beneficiaries there is no disposal by the PRs, so they make neither a loss nor a gain. The values at the date of transfer are not relevant.

A beneficiary is deemed to acquire the asset at probate value so the deceased’s:

  • mother acquired the piano with a value of £1,200
  • brother acquired the shares with a value of £3,500.

When the assets were sold by the beneficiaries, the deceased’s:

  • mother sold the piano for £1,800 and would have made a gain of £600
  • brother sold the shares for £2,500 and would have made a loss of £1,000.

The PRs and the deceased’s mother has made a gain. The deceased’s brother has made a loss.

Incorrect
Incorrect. Refer to your notes on PR responsibilities for IT and CGT. Where assets are transferred to beneficiaries there is no disposal by the PRs, so the PRs do not make either a loss or a gain.

How well did you know this?
1
Not at all
2
3
4
5
Perfectly
122
Q

The PRs of an estate are ready to distribute assets to the residuary beneficiaries. The residue comprises cash (£2,000) and shares in a quoted company (£1,000). There are two residuary beneficiaries who are entitled to share equally in the residue.

Which of the following is correct?

The PRs should split the assets in half so that each beneficiary receives one of the two items.

The PRs must sell the shares and then distribute £1,500 in cash to each beneficiary.

The PRs may appropriate the cash and shares in whatever proportions they see fit, provided that both beneficiaries receive assets with a total value of £1,500.

The PRs must obtain the agreement of the beneficiaries about how the appropriate the assets.

The PRs must transfer £1,000 cash and £500 of shares to each beneficiary.

A

The PRs may appropriate the cash and shares in whatever proportions they see fit, provided that both beneficiaries receive assets with a total value of £1,500.

Correct
Correct. The PR may appropriate assets in or towards a beneficiary’s entitlement. Each beneficiary should receive half the value of the residue (£1,500), but neither is entitled to any particular asset. The other options were incorrect because the PRs:

  • do not have to divide each asset in ½
  • do not have to convert the residue into cash in order to distribute identical sums to the beneficiaries
  • do not have to divide the number of items in the estate
  • may give effect to a beneficiary’s wishes but are not obliged to do so.
How well did you know this?
1
Not at all
2
3
4
5
Perfectly
123
Q

An estate is valued at £50,000. The deceased’s will leaves a specific gift of an asset worth £10,000 to A, a general cash legacy of £38,000 to B, and the residue to C. The deceased’s debts total £4,000.

Which of the following is correct?

The creditors are paid in full. A, B and C share the remainder of the estate equally between them.

A receives the specific asset and the creditors are paid in full. B receives £36,000. C receives nothing.

A receives the specific asset and the creditors are paid in full. B and C share the remainder of the estate equally between them.

A receives the specific asset and B receives £38,000. C receives nothing. The creditors receive £2,000.

The estate is insolvent so the statutory order of payment will apply.

A

A receives the specific asset and the creditors are paid in full. B receives £36,000. C receives nothing.

Correct
Correct. The estate is solvent as there are sufficient assets to meet the debts. However, there are insufficient funds to pay the debts and all of the legacies. The beneficiaries receive their inheritance from the assets that remain after creditors are paid in full.

As between the beneficiaries the specific legacies are paid first, then the general legacies and to the extent anything remains this would go to the residuary beneficiary.

Here, the specific legacy will be paid in full and the beneficiary of the general legacy will receive what is left over (but will inherit less than the legacy amount). The residuary beneficiary receives nothing.

How well did you know this?
1
Not at all
2
3
4
5
Perfectly
124
Q

Your firm has been instructed by the PRs of an estate. One of the solicitors has just prepared a set of estate accounts.

Who should approve and sign these?

The PRs and residuary beneficiaries.

The residuary beneficiaries.

The solicitor who prepared them.

The PRs.

The PRs and all of the beneficiaries.

A

The PRs and residuary beneficiaries.

Correct
Correct. The estate accounts should be approved and signed by the PRs and residuary beneficiaries.

How well did you know this?
1
Not at all
2
3
4
5
Perfectly
125
Q

1 – Having obtained the grant of probate Desmond and Tara (the ‘Executors’) are keen to progress the administration of Robert’s estate. Tara has made arrangements to transfer Meadow View and the joint bank account into her sole name. The executors have already closed Robert’s sole current account (and did this without the grant as the figure was small) but the Executors now need advice regarding Robert’s savings account.

Which of the following sets out the most appropriate course of action the Executors should take?

[A] Send a sealed copy of the grant of probate to the bank, arrange for the account to be closed and ask for the money to be transferred to Price Prior’s client account.

[B] Send an official copy of Robert’s will to the bank, arrange for the account to be closed and ask for the money to be divided equally so half is paid to Tara’s personal account and the other half to Desmond’s personal account.

[C] Send an official copy of Robert’s will to the bank, arrange for the account to be closed and ask for the money to be transferred to Price Prior’s client account.

[D] Send a sealed copy of the grant of probate to the bank, arrange for the account to be closed and ask for £150,000 to be paid to Andrew, £10,000 to Crisis and the balance transferred to Tara’s personal account.

[E] Send Robert’s original will to the bank, arrange for the account to be closed and ask for the money to be transferred to Price Prior’s client account.

A

[A] Send a sealed copy of the grant of probate to the bank, arrange for the account to be closed and ask for the money to be transferred to Price Prior’s client account.

  • the bank’s job is just to transfer the balance to PR and not make the payments to the beneficiaries
  • in this scenario, Tara and Desmond have instructed Price Prior

Solution available for in class MCQs (one drive)
Correct [A]:
A sealed copy of the grant of probate (rather than the will) is used to confirm that the Executors have authority to act (B, C and E are incorrect).
The money in the bank account should be sent to Price Prior’s designated client account or a specific Executors account. It should not be paid into the personal account of either Tara or Desmond (Option B, D and E are incorrect); this would mix estate money with their personal money making it more difficult to keep track of funds, accurately account for bank interest and prepare estate accounts and risks a breach of duty. The money should not be paid by the bank to the beneficiaries at this point – these funds will be needed to pay creditors and meet the estate administration costs (options D and E are incorrect).

How well did you know this?
1
Not at all
2
3
4
5
Perfectly
126
Q

2 – The Executors discover the closing balance for Robert’s savings account is greater than the date of death balance because interest has accrued since Robert died. Total interest received is less than £100.

Which of the following most accurately describes the inheritance tax (‘IHT’) and income tax (‘IT’) liabilities?

[A] The bank interest is excluded from Robert’s IHT estate. The interest is income payable to the estate. No IT is payable because the Executors can use their Personal Allowance.

[B] The bank interest is included in Robert’s IHT estate. The Executors must notify HMRC about the increase in value but no IHT is payable because the taxable value of Robert’s estate remains below the nil rate band.

[C] The bank interest is included in Robert’s IHT estate. The Executors do not need to notify HMRC because the taxable value of Robert’s estate remains below the nil rate band.

[D] The bank interest is excluded from Robert’s IHT estate. The interest is income payable to the estate. No IT is payable because the only income is savings income and the amount is below the estate IT threshold.

[E] The bank interest is excluded from Robert’s IHT estate. The interest is income payable to the estate. The Executors are required to pay IT.

A

[D] The bank interest is excluded from Robert’s IHT estate. The interest is income payable to the estate. No IT is payable because the only income is savings income and the amount is below the estate IT threshold.

  • he did not have this interest when he died so it should not have been included at all! no retrospective adjustment for IHT unless claiming a specific loss relief – so not subject to IHT
  • subject to income tax (IT)
    – income tax applies but the amount is so low that HMRC is not interested
  • D and E could in theory be correct

Solution available for in class MCQs (one drive)
Correct [D]:
It is the value of assets on the date of death which are relevant for IHT purposes and any interest accrued following Robert’s death cannot form part of his estate (so options B and C are incorrect).
Post death bank interest is income for the estate and would ordinarily be subject to income tax (an estate expense payable by the PRs from the estate funds). However, where the only income received is saving interest, and the amount is below the de-minimus threshold at which IT would be due (as it is here), no IT is payable (option E is incorrect). The Executors are not entitled to a Personal Allowance (option A is incorrect).

How well did you know this?
1
Not at all
2
3
4
5
Perfectly
127
Q

3 – The Executors are considering selling Robert’s paintings. Robert bought the paintings for £30,000. They were accurately valued at £40,000 on the date Robert died. Their current market value is £45,000. Tara has used all of her capital gains tax allowance this tax year.

Which of the following correctly describes the capital gains tax (‘CGT’) position for the Executors?

[A] They would make a £5,000 gain. The Executors’ tax free allowance would minimise any CGT liability.

[B] They would make a gain of £15,000. The Executors’ tax free allowance would minimise any CGT liability.

[C] They would make a gain of £10,000. The Executors’ tax free allowance would minimise any CGT liability.

[D] They would make a gain of £10,000. The Executors do not qualify for a tax free allowance. CGT would be payable.

[E] They would make a gain of £10,000. No CGT is payable because gains accrued during the deceased’s lifetime are not subject to CGT following their death.

A

[A] They would make a £5,000 gain. The Executors’ tax free allowance would minimise any CGT liability.

  • you always want the value of the asset on the date of death!
  • death is not a disposal so increase before death is irrelevant here!

capital gains tax starting point is date of death

Solution available for in class MCQs (one drive)
Correct [A]:
The Executors have a power of sale in respect of all of the estate assets. Gains accrued during the deceased’s lifetime (the increase in value from £30,000 to £40,000) are effectively cancelled out on a person’s death, and option E was correct in this respect only.
Therefore, only the post death gains (the increase from £40,000 to £45,000) are relevant - the figure for gains stated in all options except A were incorrect.
Whether or not the gain would give rise to any CGT liability depends on how much of the Executors’ annual exempt amount is available. Personal Representatives do qualify for a CGT tax free allowance (so option D was incorrect in this respect). If you thought they did not you may have confused this with the Income Tax rule where there is no personal allowance for PRs.

How well did you know this?
1
Not at all
2
3
4
5
Perfectly
128
Q

4 – The Executors will be personally liable if a creditor or beneficiary suffers loss as a result of their breach of fiduciary/ statutory duties. Having collected Robert’s assets, the Executors must pay his creditors and beneficiaries, but do not know whether they are required to make these payments in a particular order.

Which one of the following most accurately describes the Executors’ obligations?

[A] Robert’s estate is solvent - there are sufficient funds to pay his debts, administration expenses and legacies. The order in which these payments are made does not matter.

[B] Robert’s estate is solvent - there are sufficient funds to pay his debts and administration expenses. The order in which these payments are made does not matter.

[C] Robert’s estate is solvent - there are sufficient funds to pay his debts. The order in which these payments are made does not matter.

[D] Robert’s estate is solvent - there are sufficient funds to pay his debts, administration expenses and legacies. The Executors must make these payments in accordance with a statutory order.

[E] Robert’s estate is solvent - there are sufficient funds to pay his debts. The Executors must make these payments in accordance with a statutory order.

A

[B] Robert’s estate is solvent - there are sufficient funds to pay his debts and administration expenses. The order in which these payments are made does not matter.

solvent: expenses and liabilities
does not matter if you can pay legacies

where it is solvent there is no statutory order of priority since all the creditors are going to be paid!

if the estate was insolvent then there is legislation for which creditors have priority

Solution available for in class MCQs (one drive)
Correct [B]:
An estate is solvent for these purposes if there are sufficient funds to pay the creditors and meet the administration expenses. It is irrelevant whether or not all of the legacies can be paid (therefore options A, C and D were incorrect).
Where the estate is solvent, but there are insufficient funds to pay the legacies, special rules apply to determine how the legacies are met and in what order. This does not apply to Robert’s estate.
Where an estate is solvent the order in which unsecured debts are paid is not important, the statutory order of payment only applies where the estate is not solvent (therefore options D and E were incorrect).

How well did you know this?
1
Not at all
2
3
4
5
Perfectly
129
Q

5 – The Executors have paid the credit card and gas bills and want to make payments to beneficiaries. However, they are concerned about their liability if they distribute all of Robert’s estate and subsequently discover he had other creditors of whom they were unaware.

Which of the following most accurately sets out the steps the Executors should take now to protect themselves from a possible claim later by unknown creditors?

[A] Place a notice of their intention to distribute Robert’s estate in the Gazette in accordance with s.27 Trustee Act 1925. The Executors should delay distributing the estate until two months from the date of the notice.

[B] Place a notice of their intention to distribute Robert’s estate in the Gazette and a newspaper local to Robert’s address in accordance with s.27 Trustee Act 1925. The Executors must wait at least two months from Robert’s death before placing the notice.

[C] Place a notice of their intention to distribute Robert’s estate in the Gazette and a newspaper local to Robert’s address in accordance with s.27 Trustee Act 1925. The Executors must wait at least six months from Robert’s death before distributing the assets.

[D] Place a notice of their intention to distribute Robert’s estate in the Gazette in accordance with s.27 Trustee Act 1925. The Executors should wait at least two months from Robert’s death before placing the notice.

[E] Place a notice of their intention to distribute Robert’s estate in the Gazette and a newspaper local to Robert’s address in accordance with s.27 Trustee Act 1925. The executors should delay distributing the estate until two months from the date of the notice.

A

[E] Place a notice of their intention to distribute Robert’s estate in the Gazette and a newspaper local to Robert’s address in accordance with s.27 Trustee Act 1925. The executors should delay distributing the estate until two months from the date of the notice.

Solution available for in class MCQs (one drive)
Correct [E]:
To comply with the requirements of s.27 Trustee Act 1925 (and thus be protected by claims from unknown creditors) the Executors must place a notice of their intention to distribute the estate in the Gazette and a local paper, giving creditors 2 months from the date of the notice to get in touch. Once this deadline has passed the Executors would be protected by claims from unknown creditors and may make distributions.
A: is incomplete - an advert is also needed in a local paper.
B and D: there is no deadline for when the notice should be placed.
C: the deadline is incorrect. You may have been thinking about claims under the IPFDA 1975.

-s27 notice procedure: simply the way of notifying the general public that you are about to distribute somebody’s estate

and time limit is lined to notice (and not date of death) and it is 2 months!

local newspaper only relevant for land and business in the area

How well did you know this?
1
Not at all
2
3
4
5
Perfectly
130
Q

6 – The Executors followed the correct procedure for placing the s.27 notice and are now ready to pay beneficiaries. They decided not to sell the paintings and Crisis have asked the Executors for a painting (valued at £8,000).

Which one of the following is an accurate summary of the Executors’ powers?

[A] The Executors must pay Crisis £10,000 because this is what was stated in Robert’s will.

[B] The Executors must give Crisis the painting instead of a cash lump sum because Crisis have made a formal request and the paintings have not been specifically given to anyone else.

[C] The Executors only have the power to give Crisis a painting instead of a cash sum if they choose a painting with a value of £10,000.

[D] The Executors have the power to give Crisis the painting provided that in addition they also transfer £2,000.

[E] The Executors cannot give the painting to Crisis because Tara, as Robert’s spouse, is entitled to his personal chattels.

A

[D] The Executors have the power to give Crisis the painting provided that in addition they also transfer £2,000.

Solution available for in class MCQs (one drive)

they can but they don’t have to
and if they do they have to give the balance
PR cannot underpay
power of appropriation
- ability to decide which assets satisfy the legacy

need consent of beneficiary (here crisis)
need to make sure it is a balancing payment
and calculate balancing payment properly FROM THE DATE OF APPROPRIATION

valuation = date of appropriation (NOT THE DATE OF DEATH)

Correct [D]:
The Executors have the power under s.41 Administration of Estates Act 1925 to transfer assets in satisfaction of the charity’s cash entitlement under the will. Therefore, Option A is incorrect.
There is no obligation to exercise this power, although most PRs will comply with a reasonable request. Therefore, Option B is incorrect.
If a beneficiary requests assets in or towards the satisfaction of a cash legacy the Executors must ensure that the beneficiary receives their full entitlement. As the painting is worth £8,000 an additional £2,000 must be paid to Crisis so that they receive the full value of their inheritance. Option D is correct as the asset does not have to be exactly the same value as the cash legacy.
Option E is incorrect. Tara is not entitled to the personal chattels. You may have been confused with the spouse’s entitlement under the intestacy rules, which do not apply in this case.

How well did you know this?
1
Not at all
2
3
4
5
Perfectly
131
Q

A man died last month leaving a valid will appointing his spouse and brother as executors and giving the whole of his estate to his minor children.

The man’s gross taxable estate was valued at £800,000 and did not include any residential property.

The man’s spouse died two years ago. The man’s brother does has not taken any action so far in the administration of the estate and intends to renounce probate.

Assuming a paper application is required, which of the following most accurately states the items that will comprise the application for the grant of representation?

PA1P.

PA1P and a form of renunciation.

PA1P, IHT 421 and a form of renunciation.

PA1A, IHT 421. Provided the brother has not intermeddled no form of renunciation is required.

PA1A, IHT 400 and a form of renunciation.

A

PA1P and a form of renunciation.
Correct
Correct. The man left a valid will so form PA1P is required, even though the named executors (his spouse and brother) will not be making the application.

Inheritance tax will be payable following the man’s death. The value of his taxable estate is above the NRB and will pass to non-exempt beneficiaries. The man does not have any residential property so the RNRB will not apply. We do not know whether the TNRB may be claimed or not, but even if it did, the value of his estate is greater than 2 x NRB. As such the IHT 400 would be completed and sent to HMRC (not the Probate Registry). HMRC (not the executors) forward an IHT421 to the probate registry.

Renunciation is only possible if the executor has not intermeddled; the man’s brother has not intermeddled and renunciation is possible. A form of renunciation is always required where an executor is renouncing their role.

incorrect
PA1A, IHT 400 and a form of renunciation.
Incorrect
Incorrect. Review the materials which explain when a PA1P / PA1A would be used. Ensure you can determine what IHT form is required for an estate and which documents are included in an application for probate of both excepted and non-excepted estates. The IHT 400 is not sent to the Probate Registry.

PA1P, IHT 421 and a form of renunciation.
Incorrect
Incorrect. Ensure you can determine what IHT form is required for an estate and which documents are included in an application for probate of both excepted and non-excepted estates. The IHT 421 is not sent by the executors to the probate registry.

How well did you know this?
1
Not at all
2
3
4
5
Perfectly
132
Q

A woman dies leaving a valid will that appoints her spouse as executor and gives all of her assets to him.

The woman’s spouse takes out the grant of probate but he dies before completing the administration of the woman’s estate. He leaves a will appointing their adult son as his executor and the only beneficiary of his will.

Their son has just obtained the grant of probate for his father’s estate.

Which of the following is correct regarding who is able to administer the woman’s estate?

The son is able to complete the administration of his mother’s estate under the authority conferred by the grant of probate of his father’s estate.

The son is able to complete the administration of his mother’s estate because he is the sole beneficiary of his father’s estate.

The son is not able to complete the administration of his mother’s estate without making a further application to the probate registry.

The son is not able to complete the administration of his mother’s estate because he is not named as her executor.

The son is able to complete the administration of his mother’s estate because he is the ultimate beneficiary of her estate.

A

The son is able to complete the administration of his mother’s estate under the authority conferred by the grant of probate of his father’s estate.

Correct
Correct. The chain of representation under s.7 Administration of Estates Act 1925 applies. As the woman’s spouse had taken out the grant of probate of her estate before he died, and their son had taken out the grant of probate of his father’s estate, the chain of representation is complete. The son can complete the administration of both estates under authority conferred by the grant in his father’s estate. No further application is required.

How well did you know this?
1
Not at all
2
3
4
5
Perfectly
133
Q

A woman left a valid will which appoints her civil partner as her executor and gives the whole of the woman’s estate to her adult sister.

The woman was survived by her mother, sister and adult son. The woman’s civil partner died before the woman.

Which of the following correctly states who would apply for the grant of representation of the woman’s estate?

The woman’s mother

The woman’s sister and mother (two administrators are required)

The personal representative of the woman’s civil partner (under the chain of representation)

The woman’s son

The woman’s sister

A

The woman’s sister

Correct
Correct. The woman left a will which does not validly appoint any executors. Therefore NCPR 20 applies.

Under NCPR 20 the person with the greatest entitlement to apply is the beneficiary of the woman’s residuary estate, here her sister.

The other options were incorrect because neither the mother nor son are beneficiaries and do not rank in priority above the sister. Only one PR is required (there are no life or minor interests) and the chain of representation does not apply.

How well did you know this?
1
Not at all
2
3
4
5
Perfectly
134
Q

A woman died leaving a valid will which shares her estate equally between her adult children. The woman’s sister has been appointed as executor of the woman’s estate.

One of the woman’s three children cannot be located and has been missing since going travelling two years ago. Despite carrying out extensive searches no one can find the missing beneficiary.

The woman’s sister wants to complete the administration of the woman’s estate and distribute the assets between the two other children. However, she is concerned about her personal liability should the missing child return later and claim their 1/3 share.

Which one of the following would not adequately protect the woman’s sister?

Distributing the estate to the two other children and obtaining insurance to protect against a subsequent claim by the missing beneficiary.

Making a payment into court of the missing beneficiary’s 1/3 share.

Placing a notice in accordance with s.27 Trustee Act 1925.

Obtaining a Benjamin Order.

Retaining the missing beneficiary’s 1/3 share and delaying the administration of the woman’s estate.

A

Placing a notice in accordance with s.27 Trustee Act 1925.

Correct
Correct. All of the other options are feasible where there is a known but missing beneficiary. The protection afforded by a s.27 Trustee Act 1925 notice is in relation to unknown creditors and beneficiaries, not those who are known about but cannot be located.

How well did you know this?
1
Not at all
2
3
4
5
Perfectly
135
Q

An unmarried man died recently. The only lifetime transfer he made was an LCT (value £550,000) four and a half years before he died. IHT of £43,800 was paid on the LCT at the time of the transfer.

Calculate the IHT due on the LCT at the date of the man’s death.

£87,600

£10,200

£43,800

£8,760

£52,560

A

£8,760

Correct
Correct: The man had a cumulative total of £0 (Step A) and was able to use his AE from the year of the LCT and the previous year to reduce the chargeable value to £544,000 (Step C). The NRB is deducted from the chargeable value, leaving £219,000 to be taxed at the death rate of 40% (Step D). As the LCT was 4 years before death, taper relief is applied at 40% (Step E) and then credit is given for the IHT paid during his lifetime (Step F). The other answers were wrong because they failed to apply one or more of the AE at Step C, taper relief at Step E or credit for the lifetime tax at Step F.

How well did you know this?
1
Not at all
2
3
4
5
Perfectly
136
Q

A woman settles £650,000 on trust for her nieces and nephews (an LCT). She has made no previous lifetime transfers.

Calculate the IHT due on the LCT at the time it is made.

£63,800

£65,000

£130,000

£127,000

£64,400

A

£63,800

Correct: The woman has a cumulative total of £0 (Step A) and is able to use her AE from the year of the LCT and the previous year to reduce the value of the LCT to £644,000 (Step C). The NRB is deducted from the chargeable value, leaving £319,000 to be taxed at the lifetime rate of 20% Step D). The other answers were wrong because they missed one or both lots of AE and/or applied IHT at the death rate of 40%.

How well did you know this?
1
Not at all
2
3
4
5
Perfectly
137
Q

A woman died leaving a will that appoints her daughter as executrix. There are no provisions within the will which affect the incidence of payment of inheritance tax (‘IHT’).

The woman’s taxable estate contains the following items:

Bank accounts

Property owned tenants in common with her mother

A life interest in a trust

Who will be liable for paying IHT following the woman’s death?

The woman’s daughter and mother

The woman’s daughter and the trustees of the trust

The woman’s daughter, mother and trustees of the trust

The woman’s daughter only

The woman’s mother and trustees of the trust

A

The woman’s daughter and the trustees of the trust

Correct
Correct: The liability for paying tax on the free-estate (here the bank accounts and share of the property owned as tenants in common) lies with the woman’s daughter as her executor under the general rule. The liability for paying tax on the trust interest lies with the trustees of the trust.

138
Q

A man dies leaving a will that makes no express provision relating to the payment of inheritance tax (‘IHT’). By his will the man gives his:

Quoted shares to his father (no business property relief applies) (£40,000)

Personal possessions to his mother (£30,000)

£50,000 to his niece (this gift is not expressed to be free of tax)

The remaining assets in his estate are worth £220,000 and pass to the man’s nephew under the gift of residue.

Which of three assets noted above should the man’s PRs use to pay the IHT due following his death?

Quoted shares

Personal possessions

None of them

Any of them – the PRs may choose which items are used to meet general testamentary expenses

£10,000

A

None of them

Correct
Correct: The general rule is that the residuary estate bears the burden of the IHT due in respect of the assets passing to the PRs and that other gifts in the will are made free of tax.

The other options were incorrect because they involved using assets given away specifically in the will and did not form part of the residuary estate.

139
Q

A woman dies 9 years after making an LCT and 2 years after making a large PET to her daughter.

Inheritance tax is payable following her death estate.

By her will the woman appoints her spouse as her executor, gives her house to her brother and leaves the remainder of her estate “after all taxes have been paid” to her grandson.

Which of the following is correct with regards the payment of IHT due following the woman’s death?

The burden of the IHT will be shared between the woman’s brother and grandson

The woman’s daughter cannot be liable to pay IHT

The value of the LCT must be grossed-up before the IHT liability can be determined

The trust assets cannot be used to pay any of the IHT due

Any IHT payable in respect of the lifetime transfers will be met from the residuary estate (the general rule has been varied)

A

The trust assets cannot be used to pay any of the IHT due
Correct
Correct: The LCT was made more than 7 years before her death and is therefore not assessed to IHT at this time. As such, none of the trust assets can be used to meet the IHT bill. The other options were incorrect because:

The PET failed and if IHT is due in respect of the lifetime transfer, the woman’s daughter would be liable to pay this.

Grossing up occurs in respect of the IHT due on creation of the trust, not following death.

The burden of IHT would not be shared between the beneficiary of a specific gift and the residuary beneficiary unless the testator had given a direction in her will to vary the general rule.

The expression “all taxes” does not vary the general rule.

incorrect
The value of the LCT must be grossed-up before the IHT liability can be determined
Incorrect: Review your materials relating to the liability and burden of inheritance tax. Grossing up occurs in respect of the IHT due on creation of the trust, not following death.

Any IHT payable in respect of the lifetime transfers will be met from the residuary estate (the general rule has been varied)
Incorrect: Review your materials relating to the liability and burden of inheritance tax. The expression “all taxes” does not vary the general rule.

The burden of the IHT will be shared between the woman’s brother and grandson
Incorrect: Review your materials relating to the liability and burden of inheritance tax. The burden of IHT would not be shared between the beneficiary of a specific gift and the residuary beneficiary unless the testator had given a direction in her will to vary the general rule.

140
Q

A man makes a gift into trust (LCT) worth £15,000. The man has made no other lifetime transfers.

Which one the following exemptions/reliefs applies?

Small gifts

Annual exemption of £6,000

Marriage

Annual exemption of £3,000

Woodlands relief

A

Annual exemption of £6,000

Correct
Correct: The man can claim the annual exemption (‘AE’) of £3,000 for the tax year of the gift. As he has made no other lifetime transfers we know he has not used his AE from the previous year, so this may be claimed as well, which gives a total of £6,000.

The other options were incorrect because:

AE of more than £3,000 can be claimed

Small gifts allowance cannot apply to a gift worth more than £250, nor to a gift into a trust

For marriage exemption to apply the gift must be made to a party of the marriage (not a trust)

Woodlands relief is not available in respect of lifetime transfers.

141
Q

A woman made a gift of £500,000 to her daughter 3.5 years before she died.

Which of the following is correct in relation to the woman’s lifetime gift?

The gift is a PET which means it is not subject to IHT and therefore taper relief is irrelevant.

The gift is chargeable to IHT following the woman’s death. Taper relief applies and only 20% of the IHT due is now payable.

The gift is a PET, which means it was not chargeable when it was made. Therefore, there is no IHT liability which can be tapered.

The gift is chargeable to IHT following the woman’s death. Taper relief applies and an 80% reduction in the IHT due can be claimed.

The gift is chargeable to IHT following the woman’s death. Taper relief applies and a 20% reduction in the IHT due can be claimed.

A

The gift is chargeable to IHT following the woman’s death. Taper relief applies and a 20% reduction in the IHT due can be claimed.

Correct
Correct: The woman has made a gift to another person during her lifetime (a PET) and then died within 7 years after making the gift (the PET has failed). The failed PET is chargeable following the woman’s death. As the woman survived more than 3 years after making the gift, taper relief will apply. The rate of taper is a 20% reduction in the IHT due (i.e. 80% of the IHT due is payable).

The other options were incorrect because:

-The PET is chargeable - it failed as the woman died less than 7 years after making the gift

  • Taper relief applies as the woman survived more than three years after making the gift
  • The rate of taper is 20% and therefore 80% remains payable
142
Q

A couple are getting married. What is the maximum amount of IHT marriage relief that can be claimed in relation to a gift to the couple by the groom’s aunt?

£2,000

£1,000

None

£2,500

£5,000

A

£1,000

Correct
Correct. The aunt is not a parent or grandparent so the other figures do not apply. She cannot give £1,000 to each party - the relief applies “per marriage”

143
Q

A client comes to see you to discuss IHT planning. The client owns a valuable family home, a rental property and significant cash savings.

Which of the following represents poor IHT planning advice?

The client could transfer the rental property to his children and pay them rent for any use he later makes of the property.

The client could give away £3,000 each tax year.

The client could take out a life insurance policy and write the benefit of the policy in trust for his children.

The client could transfer the family home to his adult children and continue to live there rent free provided the children agree.

The client could ensure his spouse has sufficient cash to give away £3,000 each tax year.

A

The client could transfer the family home to his adult children and continue to live there rent free provided the children agree.

Correct
Correct – this option would not be good IHT planning. If a person gives away an asset that they continue to benefit from they make a gift with reservation of benefit (GROB). For IHT purposes the client would continue to be treated as the owner of the property and their estate for IHT purposes would be no smaller than if they had simply done nothing.

144
Q

A client comes to see you to discuss IHT planning. The client’s estate is taxable. The client is not married and is not close with any of their family aside from their only child (21) who is studying full time at university. The client spends roughly 90% of their income each year, leaving 10% unused. The client has savings of about £60,000.

On these facts, which of the following IHT exemptions are likely to be the most useful for this client?

Annual exemption and small gifts allowance.

Marriage exemption, family maintenance and the annual exemption.

Spouse exemption, small gifts allowance and normal expenditure from income.

Family maintenance, annual exemption and normal expenditure from income.

Family maintenance and spouse exemption.

A

Family maintenance, annual exemption and normal expenditure from income.

Correct
Correct. The client could maintain their adult child while in full time education and make use of the family maintenance exemption. The client could also make regular gifts of their spare income and claim normal expenditure from income. The annual exemption would be useful for gifts that would not otherwise be covered by another exemption. The other options were wrong because:

The client was not married

There was nothing to suggest that the client would make multiple gifts of small amounts to other family members

There is no marriage taking place

145
Q

A client comes to see you to discuss IHT planning as they have just won the sum of £400,000 from playing the national lottery. The client can afford to give away most of this amount. There is no specific person that needs a large sum of money up front but the client would like to benefit a number of different family members.

Is the following true or false?

The client should make a single PET to the value of £400,000 and purchase insurance to cover the risk that IHT becomes payable if the client dies in the 7 years that follow the gift.

True

False

A

False

Correct
Correct. A single PET would be a gift to one person which the client has indicated is not suitable. It would be better to set up a trust to benefit a wider group of people i.e. make an LCT instead of a PET.

146
Q

Tax avoidance is a criminal offence.

True or false?

False

True

A

False

Correct: Tax evasion is a criminal offence. Tax avoidance, however aggressive, is not.

147
Q

A woman dies. Her estate consists of her house (valued at £500,000), £200,000 in a bank account and shares worth £100,000 (which qualify for BPR at 100%). She acquired the shares with a loan of £100,000. There is £50,000 outstanding on the loan at the date of her death. Her funeral expenses are £10,000.

What is the value of the woman’s taxable estate once debts and reliefs have been deducted?

£690,000

£600,000

£640,000

£590,000

£700,000

A

£690,000

Correct: The woman’s assets are worth £800,000. The funeral expenses and loan are deducted to reach a figure of £740,000. As the loan is deducted from the value of the shares (reducing them to £50,000) only £50,000 can be deducted for BPR. This results in a figure of £690,000.

148
Q

A woman buys a holiday cottage in Cornwall (England) for £200,000. She stays in the cottage for several weeks every year and rents it out to holidaymakers for the rest of the year. Eight years before the woman dies, she makes a gift of the cottage to her daughter. At the time of the gift, the cottage is worth £300,000. The woman continues to stay in the cottage for several weeks every year and receives the rental income until her death. At the date of the woman’s death, the cottage is valued at £500,000.

How is the cottage treated for inheritance tax (‘IHT’) and capital gains tax (‘CGT’) purposes in the hands of the woman and her daughter respectively?

For inheritance tax purposes, the cottage is treated as part of the woman’s taxable estate and is valued at £300,000. For capital gains tax purposes, the woman disposes of the cottage for £300,000, resulting in a gain of £100,000. Her daughter acquires the cottage at its market value of £300,000

For IHT purposes, the disposal of the cottage is a potentially exempt transfer which takes place more than seven years before the woman’s death and is therefore not taxable. For CGT purposes the cottage is treated as passing on the woman’s death, meaning no CGT is payable on the transfer and the woman’s daughter acquires it at its market value of £500,000.

For IHT purposes, the cottage is treated as part of the woman’s taxable estate and is valued at £500,000. For CGT purposes the cottage is also treated as passing on the woman’s death, meaning no CGT is payable on the transfer and the woman’s daughter acquires it at its market value of £500,000.

For IHT purposes, the disposal of the cottage is a potentially exempt transfer which takes place more than seven years before the woman’s death and is therefore not taxable. For capital gains tax purposes, the woman disposes of the cottage for £300,000, resulting in a gain of £100,000. Her daughter acquires the cottage at its market value of £300,000.

For IHT purposes, the cottage is treated as part of the woman’s taxable estate and is valued at £500,000. For CGT purposes, the woman disposes of the cottage for £300,000, resulting in a gain of £100,000. Her daughter acquires the cottage at its market value of £300,000.

A

For IHT purposes, the cottage is treated as part of the woman’s taxable estate and is valued at £500,000. For CGT purposes, the woman disposes of the cottage for £300,000, resulting in a gain of £100,000. Her daughter acquires the cottage at its market value of £300,000.

Correct
Correct: The gift is a GROB which does not cease before the woman’s death so it is treated as part of her taxable estate for IHT purposes. It is also a potentially exempt transfer but it is not reassessed because it was made more than 7 years before the woman’s death, so it is not reassessed when the woman dies. The lifetime gift is treated as a disposal for CGT purposes, meaning the woman has made a gain (which will be chargeable because it’s not her main residence) and her daughter does not benefit from the free CGT uplift.

149
Q

Each of the following testators made a will leaving their whole estate to their surviving spouse.

Testator A is married and owns the following: a rental property worth £350,000, cash savings of £40,000 and personal possessions worth £15,000. Testator A has a full nil rate band available.

Testator B is married and owns the following: a residential property owned jointly with their spouse which is worth £250,000, cash savings of £7,000 and personal possessions worth £10,000. Testator B used part of their nil rate band and only £280,000 remains.

Testator C was married when their will was drafted but subsequently divorced. Testator C owns property valued at £500,000, has investments worth £30,000 and personal possessions worth £25,000. Testator C has no available nil rate band.

For which of these testators will spouse exemption reduce or eliminate the charge to inheritance tax following their death?

Testator A only.

Testator B only.

All of them.

Testators B and C only.

Testators A and B only.

A

Testator A only.

Correct
Correct. Spouse exemption is useful where the deceased’s estate would otherwise be taxable. Testator A has assets worth more than the nil rate band and therefore IHT would be payable if spouse exemption did not apply.

The other options were incorrect because:

Testator B does not have a taxable estate. The value of the assets is less than the nil rate band so no tax saving is achieved.

Testator C is not married when they died so spouse exemption does not apply.

incorrect
Testator B only.

Incorrect
Incorrect. Testator B does not have a taxable estate. The value of the assets is less than the nil rate band so no tax saving is achieved.

150
Q

A client comes to see you for advice on making a new will. The client wants to share their assets between their nieces and nephews and a local charity. The clients owns property worth £210,000, has cash savings of £100,000 and personal possessions worth £30,000. The client has a full nil rate band available. You may assume there are no estate liabilities.

The client proposes making a cash gift to the charity of £34,000.

Which of the following is correct?

The gift to charity is exempt and the remainder of his estate would be taxed at 40%.

The gift to charity is taxed at 36% and the remainder of the estate is taxed at 40%.

All of the estate will be taxed at 36%.

The gift to charity is exempt and the remainder of the estate would be taxed at 36%.

No IHT would be payable on the man’s estate.

A

No IHT would be payable on the man’s estate.

Correct. The man’s estate is worth £340,000. The gift to charity of £34,000 qualifies for 100% relief. The remainder of his estate worth £306,000 is within the nil rate band so no tax is due.

The other options were incorrect because:

Although the charitable gift is worth at least 10% of the estate, after the nil rate band is applied, there is no part of the estate chargeable to tax to which the reduced rate can apply.

If tax were payable, the reduced rate of IHT would apply.

The reduced rate of tax does not apply to charitable gift

incorrect
All of the estate will be taxed at 36%.
Incorrect. The gift to charity is fully exempt and is not taxed.

The gift to charity is exempt and the remainder of the estate would be taxed at 36%.
Incorrect
Incorrect. Although the charitable gift is worth at least 10% of the estate, after the nil rate band is applied, there is no part of the estate chargeable to tax to which the reduced rate can apply.

151
Q

A client owns shares in a ltd company set up by their friend. The shares are currently qualifying business assets and are worth £20,000. The testator’s other assets comprise his savings (£300,000), car (£15,0000), personal possessions (£50,000).

The client wishes to make a will that provides for his daughter and leaves the residue of his estate to his civil partner.

The client has no nil rate band available.

If the client wishes to make efficient use of business property relief (BPR), which of the following would be appropriate advice with regards their will?

For BPR to apply to the legacy the shares must qualify on the date the client signs their will.

The testator should ensure the shares form part of the residue of his estate.

If the testator is worried that the shares might fall in value he should sell them and leave a specific cash legacy of the sale proceeds to his daughter.

The testator could make a specific gift of the shares to his daughter.

The testator should give the shares to his civil partner who will then claim BPR when they die.

A

The testator could make a specific gift of the shares to his daughter.

Correct
Correct. The client’s estate is taxable to the extent it does not pass to his civil partner. As such, it is efficient to give exempt assets to the chargeable beneficiary. If the shares were given to the client’s civil partner BPR would be lost. The other options were incorrect because:

  • The shares must qualify for BPR when the testator dies (not executes his will).
  • It is not efficient to make a specific gift of exempt assets to an exempt beneficiary, and there is no guarantee the assets will still qualify for BPR on the death of the civil partner.
  • It is not efficient for the exempt assets to form part of residue where the residue is given to an exempt beneficiary.
  • A gift of cash will not qualify for BPR.

incorrect
If the testator is worried that the shares might fall in value he should sell them and leave a specific cash legacy of the sale proceeds to his daughter.
Incorrect. A gift of cash will not qualify for BPR.

The testator should ensure the shares form part of the residue of his estate.
Incorrect
Incorrect. It is not efficient for the exempt assets to form part of residue where the residue is given to an exempt beneficiary.

152
Q

A client comes to see you to discuss IHT planning by their will.

The client wants to make a will that leaves everything they have to their spouse (S) if the client pre-deceases S. If the client survives S, they want to leave their estate to their daughter (D).

You discuss two options:

Option 1: your client could leave everything to S.

Option 2: your client could leave a gift of the nil rate band to D and the balance of their estate to S.

Assuming your client dies before their spouse, which of the following is correct?

Option 1 is preferable if your client wishes to guarantee that D will inherit a significant portion of their estate.

Option 2 is preferable if your client would prefer the family to benefit from an increase in the nil rate band amount after your client dies.

Option 2 is preferable if S has a much smaller estate with only limited assets of their own.

Whether your client chooses Option 1 or Option 2, it makes no difference to the nil rate band available when S dies.

Whether your client chooses Option 1 or Option 2, it makes no difference to the IHT payable when your client dies.

A

Whether your client chooses Option 1 or Option 2, it makes no difference to the IHT payable when your client dies.

Correct. No IHT is payable when the client dies, whether Option 1 or 2 is chosen. With Option 1, the whole estate is spouse exempt. With Option 2, the gift to D is taxed at 0% and the balance is spouse exempt. The other options are incorrect because:

It does make a difference to the NRB available for S. With Option 2, the client uses their own NRB and none will be transferred to S. With Option 1, the client’s NRB is not used and so can be transferred to S.

For the family to benefit from an increase in the NRB after the client’s death, the client should avoid using their NRB (i.e. choose Option 1). When S later claims both NRBs, the amount is calculated with reference to the NRB at the time of S’s death.

To guarantee that D will inherit, Option 2 (which includes a direct gift to D) should be chosen. If Option 1 is chosen everything falls within S’s control, and S may decide not to benefit D.

If S has a smaller estate with only limited assets and will require the use /benefit of your client’s estate to maintain an acceptable standard of living, it may not be sensible to make a significant gift to D.

incorrect
Whether your client chooses Option 1 or Option 2, it makes no difference to the nil rate band available when S dies.
Incorrect. Refer to your materials relating to tax planning by will and use of the NRB. It does make a difference to the NRB available for S. With Option 2, the client uses their own NRB and none will be transferred to S. With Option 1, the client’s NRB is not used and so can be transferred to S.

Option 2 is preferable if S has a much smaller estate with only limited assets of their own.
Incorrect. Refer to your materials relating to tax planning by will and use of the NRB. If S has a smaller estate with only limited assets and will require the use /benefit of your client’s estate to maintain an acceptable standard of living, it may not be sensible to make a significant gift to D.

Option 1 is preferable if your client wishes to guarantee that D will inherit a significant portion of their estate.
Incorrect. Refer to your materials relating to tax planning by will and use of the NRB. -To guarantee that D will inherit, Option 2 (which includes a direct gift to D) should be chosen. If Option 1 is chosen everything falls within S’s control, and S may decide not to benefit D.

153
Q

You are drafting a will for a client who wishes to make a tax-free gift to their son of the full nil rate band at the date of their death.

Your client does not want the gift to include the amount of any transferrable NRB or any residence NRB.

True or false: The following clause the best way of achieving your client’s objectives:

“I give the amount of £325,000 to my son”.

True

False

A

False

Correct
Correct. The amount of the NRB might change over time, and the client may use some of their NRB during their lifetime, so you cannot know for certain what the exact amount should be. It is preferable to use a formula clause that refers generically to the nil rate band at the date of death. However, care should be taken to ensure a formula clause is not interpreted to also include any transferred NRB or residence NRB that might be available when the client dies.

154
Q

A client comes to see you to discuss IHT planning by their will.

Your client is not married but lives with their long-term partner (P). The couple have two children together.

Your client has assets to the value of £500,000, which includes a residential property.

P has assets worth £30,000 and does not have any interest in a residential property.

Your client and P both make wills that leave the whole of their estate to each other, and on the survivor’s death their combined estates pass to their children. Assume the couple live in the property until the death of the survivor of them.

Which of the following is correct?

The residence NRB could be claimed on the death of the survivor of your client and P.

Both NRBs will be used in full irrespective of the order of their deaths.

The residence NRB could only be claimed on your client’s death if your client died first.

Both NRBs will be used in full only if P dies first.

The estate of the survivor of your client and P would be eligible for the unused proportion of the NRB of the first to die.

A

The residence NRB could be claimed on the death of the survivor of your client and P.

Correct. The residence nil rate band could be claimed by the survivor of your client and P; at this point the two estates would be combined into the hands of the survivor and therefore contain the residential property interest, and the whole amount will pass to their children. The other options were incorrect because:

If P dies first, their NRB will only be used to the extent they have assets available. Their estate is well below the amount of the NRB. To the extent the NRB is unused it is wasted.

No transferrable NRB applies because your client and P are not married.

Although your client currently owns the residential interest, if they died first, the property passes to P (as the survivor), not the children and therefore the residence NRB would not be available at this time.

155
Q

A man was married and has three adult children from that marriage. After the man divorced his first wife he later remarried. The man has no children with his 2nd wife but she has two children of her own from her previous marriage.

The man wants to make a will is tax efficient and benefits his second wife while she is alive. He wants his wife to have a guaranteed right to his estate but open to suggestions about what form that should take. After she dies, he would like his estate to pass to his own children. The man does not want to leave any of his assets to the children of his second wife.

The man is concerned that if he gives his estate to his second wife outright she might make a will leaving everything (including the assets she inherited from him) to her own children.

The value of the man’s estate is greater than the nil rate band available.

Which of the following would you advise?

The man should create a discretionary trust and name his spouse and children as beneficiaries. Spouse exemption would not apply on the man’s death.

The man should create a discretionary trust and name his spouse and children as beneficiaries. Spouse exemption would apply on the man’s death.

The man should create a life interest will trust naming his spouse as the life tenant and his children as the remainder beneficiaries. Spouse exemption would not apply on the man’s death.

The man should create a life interest will trust naming his spouse as the life tenant and his children as the remainder beneficiaries. Spouse exemption would apply on the man’s death.

The man should create a life interest will trust naming his spouse as the remainder beneficiary and his children as life tenants. Spouse exemption would apply on the man’s death.

A

The man should create a life interest will trust naming his spouse as the life tenant and his children as the remainder beneficiaries. Spouse exemption would apply on the man’s death.

Correct
Correct. Setting up a life interest will trust with his spouse as the life tenant and his children as remainder beneficiaries would achieve the clients aims. His spouse would have a guaranteed right to income and the capital of his estate would remain available for his children following her death. Spouse exemption will apply provided his spouse is named as the life tenant so this is a tax efficient structure to use in his will. The other options were incorrect because i) a discretionary trust is not suitable because his wife would have no guaranteed rights to the trust fund and ii) for spouse exemption to apply his spouse needs to be the life tenant not the remainder beneficiary.

156
Q

A man was married and has three adult children from that marriage. After the man divorced his first wife he later remarried. The man has no children with his 2nd wife but she has two children of her own from her previous marriage.

The man wants to make a will is tax efficient and benefits his second wife while she is alive. He wants his wife to have a guaranteed right to his estate but open to suggestions about what form that should take. After she dies, he would like his estate to pass to his own children. The man does not want to leave any of his assets to the children of his second wife.

The man is concerned that if he gives his estate to his second wife outright she might make a will leaving everything (including the assets she inherited from him) to her own children.

The value of the man’s estate is greater than the nil rate band available.

Which of the following would you advise?

The man should create a discretionary trust and name his spouse and children as beneficiaries. Spouse exemption would not apply on the man’s death.

The man should create a discretionary trust and name his spouse and children as beneficiaries. Spouse exemption would apply on the man’s death.

The man should create a life interest will trust naming his spouse as the life tenant and his children as the remainder beneficiaries. Spouse exemption would not apply on the man’s death.

The man should create a life interest will trust naming his spouse as the life tenant and his children as the remainder beneficiaries. Spouse exemption would apply on the man’s death.

The man should create a life interest will trust naming his spouse as the remainder beneficiary and his children as life tenants. Spouse exemption would apply on the man’s death.

A

The man should create a life interest will trust naming his spouse as the life tenant and his children as the remainder beneficiaries. Spouse exemption would apply on the man’s death.

Correct
Correct. Setting up a life interest will trust with his spouse as the life tenant and his children as remainder beneficiaries would achieve the clients aims. His spouse would have a guaranteed right to income and the capital of his estate would remain available for his children following her death. Spouse exemption will apply provided his spouse is named as the life tenant so this is a tax efficient structure to use in his will. The other options were incorrect because i) a discretionary trust is not suitable because his wife would have no guaranteed rights to the trust fund and ii) for spouse exemption to apply his spouse needs to be the life tenant not the remainder beneficiary.

157
Q

A woman left the whole of her estate to a two-year discretionary trust under her will. Her civil partner and children are the trust beneficiaries. When she died, the woman’s estate was worth more than the nil rate band.

The 10 months after her death and after the administration of her estate was finalised the trustees appointed half of the trust fund to the woman’s civil partner and the other half to the woman’s children.

Which of the following is correct?

IHT would have been payable when the woman died. The trust distributions do not give rise to a refund of IHT because they were not made during the administration.

IHT would have been payable when the woman died. However, following the trust distributions a refund of inheritance tax can be claimed.

No IHT was payable when the woman died as her estate was left to a discretionary trust rather than an individual person.

IHT would have been payable when the woman died. The trust distributions do not give rise to a refund of IHT because the transfers were made after her death.

No IHT was payable when the woman died as her civil partner was a beneficiary of the will trust.

A

IHT would have been payable when the woman died. However, following the trust distributions a refund of inheritance tax can be claimed.

Correct
Correct. We know that IHT would have been payable following the woman’s death. Under s.144 IHTA distributions made from a discretionary will trust in the two years following death are treated as transfers made by the deceased under their will. In this case, the woman’s will is therefore treated as giving half of her estate to her civil partner and half to her children –in which case civil partner exemption will be available. A refund of some or all of the IHT paid at the time can be made.

The other options were incorrect because:

the effect of s.144 was not considered

the effect of s.144 applies in the 2 years following death and is not limited to the administration period

IHT would have been paid at the time – a discretionary trust is not an exempt beneficiary, and civil partner exemption would not have been available just because the civil partner was one of the beneficiaries

158
Q

A testator wishes to set up a trust under their will to benefit their children and grandchildren. The testator is uncertain what the best option is. The testator has three children and seven grandchildren.

One of the testator’s children is known to have a gambling addiction and another child is currently attending couples counselling due to marital difficulties.

All of the testator’s grandchildren are under the age of 18.

What is the most appropriate advice?

The testator should not leave any assets to their children by will, whether directly or into trust, because there is a risk these assets would become subject to a claim in the event of a child becoming bankrupt or getting divorced.

The testator should set up a discretionary trust under the will and name all of his children and grandchildren as beneficiaries.

The testator should make an outright gift to each of the children and grandchildren to ensure that each receives a specified amount.

The testator should set up a life interest trust under the will and appoint the grandchildren as life tenants and his three children as remaindermen.

The testator should set up a life interest trust under the will appoint the children as life tenants and his grandchildren as remaindermen.

A

The testator should set up a discretionary trust under the will and name all of his children and grandchildren as beneficiaries.

Correct
Correct. A discretionary trust seems most suitable. A life interest trust and outright gifts would be possible but less appropriate given the children’s circumstances and that the grandchildren are minors. A discretionary trust offers flexibility for trustees to take individual circumstances into account before distributing money (no beneficiary is entitled to anything) and a discretionary trust may offer protection from claims by creditors or on divorce.

159
Q

Can a solicitor take instructions on drafting a will from someone other than the testator?

Yes, but only if the instructions are provided by the client’s spouse or civil partner.

No, a solicitor can only take instructions from the client directly.

Yes, but only if the client is present when the third party gives the instructions.

Yes, but only if the third party is appropriately authorised and the solicitor has no reason to suspect that the instructions do not reflect their client’s wishes.

Yes, but only if the client provides written confirmation that the third party is authorised to act.

A

Yes, but only if the third party is appropriately authorised and the solicitor has no reason to suspect that the instructions do not reflect their client’s wishes.

Correct
Correct: See CCS 3.1

160
Q

What should a solicitor do if a client’s testamentary capacity is in doubt?

Refuse to act for the client and advise them to seek medical advice.

Take the client’s instructions but make a note in the will highlighting concerns about testamentary capacity. It is advisable for the solicitor to act as a witness to the will.

Interview a family member of the client in order to find out more about their circumstances and make a note of findings. The solicitor should only prepare a will if they believe, following the interview, that the client has capacity. It is advisable for the family member to act as a witness.

Consult an independent medical professional and provide them with information about the client. The solicitor should only prepare the will if, based on the advice provided by the medical professional, they believe it likely that the client has capacity.

Obtain the client’s consent to approach their medical practitioner for confirmation of capacity and make a record of the findings. The solicitor should only prepare a will if capacity is confirmed. It is advisable for the doctor to act as a witness and record their findings.

A

Obtain the client’s consent to approach their medical practitioner for confirmation of capacity and make a record of the findings. The solicitor should only prepare a will if capacity is confirmed. It is advisable for the doctor to act as a witness and record their findings.

Correct
Correct: This reflects the golden rule and best practice.

161
Q

Should a solicitor agree to draft a will for a client who is proposing to leave a gift to the solicitor in their will?

A solicitor is not absolutely prohibited from acting in these circumstances but there is a potential conflict of interest. They should not act where the client is proposing to make a gift of more than £500 to the solicitor unless the client takes independent advice.

A solicitor is absolutely prohibited from acting for a client in these circumstances.

A solicitor is prohibited from acting in these circumstances but can encourage the client to leave the property to the firm or to a member of the solicitor’s family instead.

A solicitor is permitted to act in these circumstances but should ensure that the will is witnessed by another solicitor who can confirm the absence of undue influence.

A solicitor is not absolutely prohibited from acting in these circumstances but there is a potential conflict of interest and it is inadvisable to act.

A

A solicitor is not absolutely prohibited from acting in these circumstances but there is a potential conflict of interest. They should not act where the client is proposing to make a gift of more than £500 to the solicitor unless the client takes independent advice.

Correct: Solicitors should not act where there is a conflict or significant risk of conflict. The SRA considers a gift of £500 to be significant.

162
Q

Which of the following best describes the position regarding testamentary capacity?

A – Testamentary capacity must exist at the time the will is executed

B – Testamentary capacity must exist at the time the testator gives instructions for his or her will and at the time the will is executed

C – Testamentary capacity need not exist at the time the will is executed if the testator had capacity when they gave instructions

D – Testamentary capacity need not exist at the time the will is executed if the testator had capacity when they gave instructions and the testator understands they are signing a will for which they gave instructions

A

D – Testamentary capacity need not exist at the time the will is executed if the testator had capacity when they gave instructions and the testator understands they are signing a will for which they gave instructions

A/B - be wary of must

in an ideal world, TC at both instruction and execution

C/D - Parker v Felgate !! cf timing of capacity

163
Q

. Which of the following is the most accurate statement?

A – The personal representatives must prove a testator had testamentary capacity when applying for the grant of probate.

B – Testamentary capacity is presumed where the will appears rational and has been properly executed.

C – Testamentary capacity is presumed where the will appears rational and has been properly executed unless there is evidence that raises doubt in which case the burden of proof passes to the personal representatives.

D –Testamentary capacity is presumed where the will appears rational and has been properly executed unless there is evidence that raises doubt in which case the burden of proof passes to the person seeking to have the will declared invalid.

A

C – Testamentary capacity is presumed where the will appears rational and has been properly executed unless there is evidence that raises doubt in which case the burden of proof passes to the personal representatives.

164
Q

A man wishes to make his will tax efficient and is seeking advice on the exemptions and reliefs available on his death. He is unmarried and has a son aged 7. He owns the house in which he lives. He would like to distribute his will in the following manner:

· His share of the family business to his brother. He has owned this for three years.

· A lump sum to his mother to cover her nursing home fees in the event she is still alive.

· His house and residuary estate to his son.

Which of the following can be used to reduce the man’s liability to inheritance tax on his death?

Family maintenance for the gift to his mother. Business property relief for his share in the business. Residence nil rate band.

Business property relief for his share in the business.

Business property relief for his share of the family business. Residence nil rate band.

The annual exemption for the year of death if unused. Business property relief for his share of the family business. Family maintenance exemption for the gift to his mother.

The annual exemption for the year of death if unused. Business property relief for his share of the family business. Family maintenance exemption for the gift to his mother. Residence nil rate band.

Learning Material
Choose your material

Assessment level MCQs
Slideshow
Assessment MCQs
Question

A

Business property relief for his share of the family business. Residence nil rate band.

Correct
Correct: BPR will be available as he has owned the business interest for more than 2 years.

incorrect
Family maintenance for the gift to his mother. Business property relief for his share in the business. Residence nil rate band.
Incorrect: The family maintenance relief is available for lifetime gifts only.

165
Q

You are advising a client on inheritance tax planning. The client’s most valuable asset is their home (worth £850,000). They want to leave it to their son but have asked you whether it would be preferable to transfer it into the son’s name now to avoid it being subject to inheritance tax.

Which of the following is the best advice to the client should they decide to make a lifetime gift of the house to their son?

The client can continue to live in the house after making the gift but should pay market rent to their son for the seven year period following the gift to ensure that the house ceases to be part of their estate for inheritance tax purposes. The client must also survive for more than seven years after the gift to ensure no inheritance tax is payable in respect of the lifetime gift.

The client cannot continue to live in the house after making the gift if he wants to ensure that the house remains outside of his taxable estate. The client must also survive for more than seven years after the gift to ensure no inheritance tax is payable in respect of the lifetime gift.

The client can continue to live in the house after making the gift but should continue to pay market rent to their son to ensure that the house ceases to be part of the client’s estate for inheritance tax purposes. This will also ensure that no inheritance tax is payable in respect of the lifetime gift.

The client can continue to live in the house after making the gift but the house will be treated as part of their taxable estate until they move out. Provided the client survives for more than seven years after the gift, and moves out of the house at some point before they die, the house will not be part of their estate for inheritance tax and no inheritance tax would be payable in respect of the lifetime gift.

The client can continue to live in the house after making the gift but should continue to pay market rent to their son to ensure the house ceases to be part of the client’s estate for inheritance tax purposes. The client must also survive for more than seven years after the gift to ensure no inheritance tax is payable in respect of the lifetime gift.

A

The client can continue to live in the house after making the gift but should continue to pay market rent to their son to ensure the house ceases to be part of the client’s estate for inheritance tax purposes. The client must also survive for more than seven years after the gift to ensure no inheritance tax is payable in respect of the lifetime gift.

Correct
Correct: The gift of the house will be a gift with reservation of benefit (GROB) unless the client pays market rent during his period of occupation. If he pays market rent then he has made a valid PET and the house will not be included in his taxable estate. For the PET itself to be fully exempt the man also has to survive 7 years from making the gift.

The other options were incorrect because:

  • The man must pay market value rent during the whole period of occupation to avoid the GROB rules
  • If the man lives in the property rent free, but moves out of the house and ceases to derive benefit from the property before he dies, this is will ensure there is no GROB. However, there would still be a PET by the man of the property on the date he moves out (i.e. the date the gift becomes fully effective), and to avoid inheritance tax consequences the man must then survive 7 years from moving out.

incorrect
The client can continue to live in the house after making the gift but should pay market rent to their son for the seven year period following the gift to ensure that the house ceases to be part of their estate for inheritance tax purposes. The client must also survive for more than seven years after the gift to ensure no inheritance tax is payable in respect of the lifetime gift.
Incorrect: The relevant period is the seven years before death, not the seven years after the gift.

The client can continue to live in the house after making the gift but should continue to pay market rent to their son to ensure that the house ceases to be part of the client’s estate for inheritance tax purposes. This will also ensure that no inheritance tax is payable in respect of the lifetime gift.
Incorrect: Although this would prevent an IHT charge under the GROB rules, note that the gift would be a potentially exempt transfer.

166
Q

You attend a meeting with a client to discuss making a new will. Your client has prepared a list of assets which include two company shareholdings (worth approximately £20,000 in total) but they cannot recall the exact number of shares that comprise each shareholding.

Your client is single but has three adult children and no other dependants. Following a recent family argument your client is not currently in contact with their youngest child. Your client is vague about the reason behind the argument.

You are instructed to draft a will that leaves 40% of your client’s estate to each of the two eldest children and the remaining 20% to the youngest child.

Does your client have testamentary capacity? Select the most accurate answer.

Your client does not satisfy the test in Banks v Goodfellow because they do not understand the extent of their property.

Your client does not satisfy the test in Banks v Goodfellow because they have not provided a justifiable reason for preferring the two eldest children.

It is likely your client satisfies the test in Banks v Goodfellow and a will can be prepared which follows their instructions.

Your client does not satisfy the test in Banks v Goodfellow because they do not understand the extent of their property, and their instructions do not adequately reflect the moral obligation to benefit their children.

Your client does not satisfy the test in Banks v Goodfellow because their instructions do not adequately reflect the moral obligation to benefit their children.

A

It is likely your client satisfies the test in Banks v Goodfellow and a will can be prepared which follows their instructions.

Correct.

To satisfy the test in Banks v Goodfellow a testator must:

  • Understand the nature of the will and its effect: nothing in the facts suggest this is in doubt
  • Appreciate the extent of their property. This does not require an exact recollection of every item. A general understanding of what is owned, and an approximate value of the item is sufficient. On the facts provided this appears to be the case.
  • Be aware of any moral claims upon them. This does not require fairness nor oblige the testator to actually leave anything to those with any moral claim. Your client is aware that all three children may expect to inherit, but is entitled to leave the estate in unequal shares and to do so without having an objectively good reason.

The incorrect options were wrong because:

  • An exact knowledge of the number of shares owned is not required
  • There is no obligation to divide your estate equally between your children
167
Q

A man is registered blind. He has given instructions to his solicitor to draft a will and this has been prepared for him to sign. The man’s solicitor is satisfied the man has testamentary capacity. The man is able to sign the will but he is unable to read the will himself.

Which of the following best describes the legal requirements for knowledge and approval (intention) to make a will?

There is no presumption of knowledge and approval in this case.

When the man signs the will there is a presumption of knowledge and approval because the will has been drafted in accordance with his instructions, unless there is evidence to rebut this.

When the man signs the will there is a presumption of knowledge and approval because there is a presumption of testamentary capacity.

When the man signs the will there is a presumption of knowledge and approval because there is a presumption of testamentary capacity unless there is evidence to rebut this.

When the man signs the will there is a presumption of knowledge and approval because the will has been drafted in accordance with his instructions.

A

There is no presumption of knowledge and approval in this case.

Correct
Correct. There is no presumption of knowledge and approval in this case. The will should be read over to the man and his express approval to the contents of the will must be given. The attestation clause should include wording to reflect that this took place.

168
Q

A woman and her husband wish to make gifts to their son and his partner on the occasion of their marriage. Earlier this tax year the woman alone gave £5,000 to her daughter from a previous marriage on the occasion of her daughter’s marriage.

What is the maximum combined amount of marriage exemption that the woman and her husband can utilise to make wedding gifts to their son and his partner?

£10,000

£6,000

£5,000

£12,000

£2,500

A

£10,000

Correct
Correct: Both the man and the woman can give £5,000 to their son. It does not matter that the woman has used the marriage relief as the exemption is per marriage, nor per tax year. They cannot also give £1,000 to his partner as the exemption applies per marriage, not per recipient.

169
Q

A woman leaves the residue of her estate in the following shares:

· 50% equally between my three children

· 50% equally between my brother and sister

One of the woman’s children has predeceased her leaving a daughter, the woman’s granddaughter. The woman’s brother has also predeceased her leaving a son, the woman’s nephew.

Which of the following best describes how the residue will be distributed?

The daughter’s share will pass to the granddaughter. The sister will take the whole of the other 50% share in the residue.

The 50% share to the children will be divided between the surviving two children and the sister will take the whole of the other 50% in the residue.

The daughter’s share will pass to the granddaughter and the brother’s share will pass to the nephew.

The daughter’s share will pass to the granddaughter. The gift to the brother will fail and there will be a partial intestacy of his share.

The estate will be split equally between the three surviving named beneficiaries.

A

The daughter’s share will pass to the granddaughter. The gift to the brother will fail and there will be a partial intestacy of his share.

Correct
Correct: S 33 Wills Act 1837 applies so that issue (children, grandchildren, great grandchildren etc) take the share their parent would have taken. The section does not apply to any other beneficiary so not to the brother’s share. This contrasts with the statutory trusts which apply on intestacy (s 47 AEA) where the substitution provision applies to all blood relatives. It is not the case that the sister takes her brother’s share unless it is expressly stated that the gift is to such of the named beneficiaries as survive and if more than one equally. This is not implied.

170
Q

A man instructs his solicitor to draw up his will. He wishes to include a gift to his solicitor of 20,000. He says that this is in appreciation of the many years of excellent service he has received from his solicitor. The man estimates his estate is worth around £100,000.

Which of the following best describes the position of the solicitor?

The solicitor must refuse to act for the client in this situation.

The solicitor may draft the will and accept the gift provided the man first takes independent legal advice.

The solicitor may draft the will but should refuse the gift as this is a conflict of interest under the solicitor’s code of conduct.

The solicitor may draft the will but should refuse the gift as it is of significant value.

The solicitor may draft the will but should refuse the gift as he has already been paid for the work that he has carried out for the man.

A

The solicitor may draft the will and accept the gift provided the man first takes independent legal advice.

Correct
Correct: There is no rule which prohibits a client making a gift by will to his or her solicitor. However, paragraph 6.1 of the Code of Conduct for Solicitors requires a solicitor not to act if there is an own interest conflict or a significant risk of an own interest conflict.

If a solicitor drafts a will where the client wishes to make a gift of significant value to the solicitor, a member of their family, or an another employee of the firm, the solicitor should be satisfied that the client has first taken independent legal advice with regard to making the gift. If the client does not agree to take independent advice the solicitor should cease to act on the instructions.

This includes situations where the intended gift is of significant value in relation to the size of the client’s overall estate, but also where the gift is of significant value in itself. In practice however, most firms will have their own rules which prohibit this.

incorrect
The solicitor may draft the will but should refuse the gift as this is a conflict of interest under the solicitor’s code of conduct.

Incorrect: Although there is a conflict of interest, there is not an absolute prohibition on the solicitor drafting the will or accepting the gift. What must they do before they can act?

171
Q

A woman leaves the following gifts in her will:

· My house to my daughter Louise

· The residue of my estate to my son Theo.

Which of the following best describes where the legal burden of inheritance tax and the costs of transfer fall?

The residue bears the whole of the inheritance tax and the house will bear the cost of transfer.

The residue bears the whole of the inheritance tax and the costs of transfer of the house.

The house and the residue bear the burden of inheritance tax proportionately according to their value and the residue will bear the cost of transfer.

The house and the residue bear the burden of inheritance tax proportionately according to their value and the house will bear the cost of transfer.

The house and the residue bear the burden of inheritance tax equally and the house will bear the cost of transfer.

A

The residue bears the whole of the inheritance tax and the house will bear the cost of transfer.

Correct
Correct: In the absence of any alternative express provision in the will the burden of inheritance tax falls upon the residue of the estate and is a testamentary expense. By contrast, costs of transfer of any specific gift will fall upon the beneficiary.

incorrect
The house and the residue bear the burden of inheritance tax proportionately according to their value and the residue will bear the cost of transfer.

Incorrect: Revisit your materials on allocation of exemptions and reliefs. If there is no express provision in the will, do the recipients of specific gifts bear the cost of inheritance tax? And does the residue bear the transfer costs of specific gifts?

The residue bears the whole of the inheritance tax and the costs of transfer of the house.

Incorrect: Revisit your materials on allocation of exemptions and reliefs. If there is no express provision in the will, does the residue bear all costs?

172
Q

A woman dies, leaving a will which provides that her property is to be held upon trust for her husband for his life and after his death for such of her son and daughter as survive her husband and attain the age of 25 in equal shares.

The woman’s son is 23 and her daughter is 25.

Which of the following correctly describes the respective nature of the husband, son and daughter’s interests?

The husband and children all have vested interests.

The husband and children have contingent interests.

The husband and daughter have vested interests and the son has a contingent interest.

The husband and daughter have contingent interests and the son has a vested interest.

The husband has a vested interest and the children have contingent interests.

A

The husband has a vested interest and the children have contingent interests.

Correct: The husband has a vested interest in the capital. The children both have contingent interests. Whilst the daughter has satisfied the age contingency her interest is still contingent on surviving her father which is not yet satisfied. The son’s interest is contingent on reaching the age of 25 and surviving his father.

173
Q

A man leaves the following gifts in his will:

· My Rolex Watch to John

· My BT shares to Althea

· £5,000 to be paid from my Sussex Building Society account to Hayley.

At the time of his death the man no longer owned the original Rolex watch but had replaced this with a new Rolex Watch, his BT holding had increased to 200 shares from the 100 shares he had when he made the will and the Sussex Building Society account had a balance of £4,000.

Which of the following best describes what, if anything, each beneficiary will receive? (You do not need to consider issues of certainty regarding the identity of the beneficiaries.)

John will receive the Rolex watch the man had when he died, Althea will receive 100 BT shares and Hayley the £4,000 in the building Society.

John will receive nothing, Althea will receive the 200 BT shares and Hayley will the receive £4,000 from the Building Society Account together with £1,000 from the general estate, provided there are sufficient funds.

John will receive nothing. Althea will receive the 200 BT shares and Hayley will the receive the £4,000 from the Building Society Account.

John will receive the Rolex watch the man had when he died, Althea will receive the 200 BT shares and Hayley the £4,000 in the building Society.

John will receive nothing. Althea will receive 100 BT shares and Hayley will receive £4,000 from the Building Society Account together with £1,000 from the general estate, provided there are sufficient funds.

A

John will receive nothing, Althea will receive the 200 BT shares and Hayley will the receive £4,000 from the Building Society Account together with £1,000 from the general estate, provided there are sufficient funds.

Correct: The watch is a specific legacy. Under s.24 Wills Act the general rule is that the will speaks from the date of death. However, use of the word ‘my’ indicates a contrary intention to the general rule indicating the man was referring specifically to the watch he owned when he made the will rather than the watch he owned at the date he died.

As regards a holding of shares, or a collection of items that is capable of growing, the use of the word ‘my’ does not alter the general position and the gift is construed as the holding or contents of the collection at the date of death.

The gift of the money in the building society is a demonstrative legacy. if there are insufficient funds in the designated account these are made up from other assets in the estate in the same way as a general legacy.

174
Q

True or False:

A letter of wishes is usually stored with a will with the intention that its terms are to be incorporated into the will.

A

false

Correct. The letter is intended as guidance only to the estate trustees, not to be a legally binding document.

175
Q

Does the following clause successfully incorporate the document mentioned into the will?

“I leave my collection of watches to my grandchildren as per my note to them”

Yes

No

A

No
Correct. The note may or may not exist at the time of execution (it’s not clear) but, in any event, the will does not expressly refer to the note being in existence at the date of execution.

incorrect
yes
Incorrect. The note may or may not exist at the time of execution (it’s not clear) but, in any event, the will does not expressly refer to the note being in existence at the date of execution.

176
Q

A testator wants to make a gift of their paintings to their children by will. However, the client does not yet know who should get which painting.

You advise the client to make a will now and include a clause referring to a list confirming the intended distribution. You tell the client that provided they make this list before they die, it will be effective. Are you correct?

Yes

No

A

No

Correct
Correct. To successfully incorporate an unexecuted document into a will it must exist at the time the will is made, not just before death.

177
Q

A valid will contains the following clauses:

I give £100 to my son, Michael
I £500 to my daughter, Mary
After execution, the testator made the following amendments:

Michael’s gift: The testator crossed out the ‘£100’ and inserted the replacement figure of ‘£200’.

Mary’s gift: The testator obliterated the ‘£500’, intending Mary to receive nothing. The original figure is no longer visible.

Applying s 21 Wills Act 1837, which of the following is the most accurate description of what Michael and Mary are entitled to receive?

Mary is entitled to £500. Michael is entitled to £200.

Mary is entitled to £500. Michael is entitled to £100.

Mary is not entitled to anything. Michael is entitled to £100.

Neither Mary nor Michael are entitled to anything.

Mary is not entitled to anything. Michael is entitled to £200.

A

Mary is not entitled to anything. Michael is entitled to £100.

Correct:

Applying s 21 Wills Act 1837, unattested alterations made after execution are invalid.

This means Michael cannot receive £200 (the new amount is an invalid alteration). However, as the original amount of £100 remains visible so Michael is entitled to this.

Although the alteration to Mary’s gift (the obliteration) is also unattested, as the original amount cannot be deciphered by natural means Mary receives nothing and the alteration is effective.

The other options were incorrect because:

  • Mary cannot receive £500 because the amount is not visible through natural means
  • Michael cannot receive £200 because the alteration has not been attested and is therefore ineffective
  • Michael is entitled to receive £100 because the original clause hasn’t been amended and the amount is legible
178
Q

A testator made a valid will two years ago that contains the following clause:

“I give [£800] to my son”. The will is typed but the sum of £800 has been inserted into the blank space by hand.

The testator died yesterday and has only one son.

What is the testator’s son entitled to receive?

£800, but only if the testator inserted the sum before he executed the will.

£800, but only if the attestation clause in the will specifically refers to the alteration.

£800, as it is not necessary for the alteration to be attested.

£800, but only if the person who prepared the will signs an affidavit confirming the amendment reflected the true intention of the testator.

Nothing, as the alteration itself has not been attested.

A

£800, as it is not necessary for the alteration to be attested.

Correct
Where a blank space has been completed (inserting £800) there is a presumption that the blank was completed before execution. In which case the alteration would be valid under the general rule in s 21 Wills Act 1837.

The other options were incorrect because:

  • There is a presumption the blank space was completed before execution so this does not have to be proved in order for the gift to be valid
  • Attestation of the alteration is not required where a blank space is completed
  • The attestation of the will as a whole does not need to refer to the alteration (completing a blank space is deemed to have occurred before execution)

Affidavit evidence is not required

179
Q

A testator made a valid will two years ago that contains the following clause:

“I give £100 to my nephew”

The ‘£100’ has been crossed out by hand but remains visible underneath.The attestation clause in the will makes no reference to the amendment. The testator died yesterday and has only one nephew. You do not know when the testator made the alteration to his will.

What is the testator’s nephew entitled to receive?

£100, under the general rule in s 21 Wills Act 1837.

£100, but only if the witnesses to the will sign an affidavit confirming the alteration was made before execution.

£100, but only if the person who prepared the will signs an affidavit confirming the alteration was made before execution.

Nothing, under the general rule in s 21 Wills Act 1837.

Nothing, on the face of it the amendment demonstrates a clear intention to revoke the gift.

A

£100, under the general rule in s 21 Wills Act 1837.

Correct
Correct: The general rule in s.21 Wills Act 1837 is that unattested alterations are deemed to have been made after execution and are therefore invalid. In the absence of evidence to the contrary, the alteration is ineffective. The original gift is visible through natural means and so the nephew will inherit this.

The other options were incorrect because:

  • There is a presumption the alteration was made after execution, so no affidavit evidence is required either by witnesses or the person who prepared the will.
  • The general rule does not confirm the validity of the amendment

Crossing a line through a number does not of itself demonstrate intention to revoke

180
Q

A testator made a valid will five years ago, which includes the following clause:

“3. I give £400 to my youngest daughter”

The testator made a valid codicil to this will six months ago. The codicil did not expressly refer to or amend clause 3 of the will. When the will was executed the testator had two daughters. When the codicil was executed, the testator had just given birth to her third daughter.

The testator died last month. What is the effect of clause 3 of the will?

The testator’s second daughter will receive £400 because she was the youngest daughter when the will was made and the codicil did not amend this clause of the will.

The testator’s second daughter will receive £400 because she was the youngest daughter when the will was made.

The testator’s second daughter will receive £400 because it is clear this must have been the intention of the testator.

Clause 3 does not take effect because there is ambiguity regarding which of the daughters it refers to.

The testator’s third daughter will receive £400 because she was the youngest daughter when the codicil was made.

A

The testator’s third daughter will receive £400 because she was the youngest daughter when the codicil was made.

Correct
Correct: The codicil re-publishes the will which is interpreted with reference to the date of the codicil. On the date the codicil was executed the testator had three daughters and the youngest of them will receive £400. The other options were incorrect because:

  • The effective date of execution of the will is the date of the codicil
  • The codicil republishes the will as a whole not just the clauses it amends

There is no reference in the codicil to how the clause should take effect the general legal principles apply

181
Q

A testator made a valid will two years ago. The will was witnessed by his son and his neighbour and included the following clauses:

“3. I give £30,000 to my son

  1. I give £30,000 to my daughter”

The testator made a valid codicil to his will a few months later which was witnessed by his two neighbours.

After executing the will, but before executing the codicil, the testator made a manuscript amendment to clause 4 of his will by drawing a line through the amount of £30,000 and replacing it with £50,000. No express reference to the amendment to the will was made in the codicil. The original amount of £30,000 remains visible.

What will the testator’s son and daughter receive?

The testator’s son will receive nothing as he witnessed the will. The testator’s daughter will receive £50,000, but only if evidence that the amendment was made before the codicil was executed is provided.

The testator’s son will receive £30,000. The testator’s daughter will receive nothing.

The testator’s son will receive nothing as he witnessed the will. The testator’s daughter will receive £50,000. No additional evidence proving when the amendment was made is needed because the date of the codicil is after the date of the will.

The testator’s son will receive £30,000. The testator’s daughter will receive £50,000, but only if evidence that the amendment was made before the codicil was executed is provided.

The testator’s son will receive nothing as he witnessed the will. The testator’s daughter is only entitled to £30,000.

A

The testator’s son will receive £30,000. The testator’s daughter will receive £50,000, but only if evidence that the amendment was made before the codicil was executed is provided.

Correct: The testator’s son witnessed the will. According to s.15 Wills Act 1837 he would not be entitled to the gift. However, the later codicil, which he did not witness, re-executes the will and therefore avoids the effect of s.15.

The amendment to the daughter’s gift was made after the will was executed so would be ineffective. However, the later codicil re-executes the will (by which time it includes the amended clause) and so the amendment is confirmed. As the codicil does not expressly confirm the amendment was made before the codicil was signed affidavit evidence to this effect would be required.

The other options were incorrect because the:

  • effect of s.15 is avoided - the codicil was not witnessed by the son and this ‘corrects’ the issue.
  • testator’s daughter will receive something – either the original amount or the increased amount if the alteration is proven to be effective
  • date of the codicil is, of itself, insufficient to validate the amendment. Evidence is needed to prove the amendment was made before the codicil was signed as the codicil itself does not expressly refer to the amendment to the will.
182
Q

You have received three sets of instructions from the following testators:

A testator who revoked their will by destroying it. Testator 1 regrets their actions and wants to revive the will again and include some additional clauses.
A testator who made a will two years ago and subsequently married has just found out that by getting married his will was automatically revoked. Testator 2 wants to revive their previous will and include some additional clauses.
A testator who made a will last year now wishes to amend it. Testator 3 wants to revoke part of the will. There are no additional clauses to add.
Which of the testators could achieve their aim by making a codicil?

Testators 1, 2 and 3.

Testator 2 only.

Testator 2 and 3 only.

Testator 1 and 2 only.

Testator 1 and 3 only.

A

Testator 2 and 3 only.

Correct:

A codicil can revive a will that was previously revoked, but not if it has been destroyed. This means Testator 1 could not use a codicil to revive his previous will but Testator 2 could.

A codicil can be used to revoke a will in whole or in part. Testator 3 can therefore use a codicil. It does not matter that there are no additional clauses to add.

183
Q

A testator made a valid will four years ago. The following year the testator executed another valid will. The second will contains the following clause: “I hereby declare this to be my last will”.

The testator died yesterday. There are no other relevant clauses within either will.

What is the effect of the second will?

To be effective the second will should have been drafted to take effect only on the condition that the first will was revoked.

The second will impliedly revokes the whole of the first will as it is dated later.

The second will expressly revokes the whole of the first will.

The second will does not expressly revoke the first and to the extent there is inconsistency the first will is given effect to.

The second will impliedly revokes the first will, but only to the extent they are inconsistent.

A

The second will impliedly revokes the first will, but only to the extent they are inconsistent.

Correct
Correct. The clause in the question does not revoke the previous will. Where there are no express words of revocation both wills remain active. The later will revokes the earlier, but only to the extent they are inconsistent.

The other options were incorrect because:

  • the clause included in the second will would not amount to express revocation
  • it is possible for a later will to revoke an earlier will by implication
  • the later will revokes the earlier where there are discrepancies (the earlier will does not take priority)

the statement relating to conditional revocation is not standard practice.

184
Q

A testator has argued with his daughter and wants to a make a new will excluding her from any benefit. The testator brings a copy of his current valid will into the office and tears it up in front of you saying “I want to revoke this will and make a new will”.

What is the effect of the testator’s actions?

The testator’s will is revoked because he expressly states that he wants to make a new will.

The testator’s will is revoked because he carried out an act of revocation in front of a solicitor.

The testator’s will is revoked only to the extent his daughter benefits from it.

The testator’s will remains valid.

The testator’s will is revoked because he carried out an act of revocation with intention to revoke.

A

The testator’s will remains valid.
Correct: Revocation by destruction requires physically destruction by the testator and intention to revoke. Although this testator intends to revoke their will revocation is only effective if the original will is destroyed. Here only a copy has been destroyed.

The other options were incorrect because:

  • the original will not a copy needs to be destroyed
  • there is no requirement to destroy the will in front of a solicitor
  • the testator’s words do not demonstrate an intention to partially revoke

an intention to make a new will is irrelevant. An intention to revoke the existing will is required.

incorrect
The testator’s will is revoked because he carried out an act of revocation with intention to revoke.
Incorrect: Review your notes on revocation. Make sure you understand all of the requirements for destruction to effectively revoke a will.

The testator’s will is revoked only to the extent his daughter benefits from it.
Incorrect: Review your notes on revocation. Make sure you understand all of the requirements for destruction to effectively revoke a will.

185
Q

A testator brings you their existing valid will, on the front of which they have written “revoked”. The testator explains they wanted to revoke their existing will, but only if their son’s marriage next month goes ahead. If the wedding does not take place the testator wishes to retain the existing will.

Which of the following is correct?

The testator’s will has not been revoked. However, it will be possible for the testator to revoke it now conditionally upon their son’s subsequent marriage.

The testator’s will has been revoked and the testator should therefore revive this will or make a new will as soon as possible.

The testator’s will has been revoked conditionally upon on their son getting married.

The testator’s will has not been revoked. The testator should wait until after their son gets married before making a new will.

The testator’s will has not been revoked. The testator needs to wait until after their son gets married before taking steps to revoke the existing will.

A

The testator’s will has not been revoked. However, it will be possible for the testator to revoke it now conditionally upon their son’s subsequent marriage.

Correct: By writing “revoked” on his will the testator has not yet effectively revoked it (conditionally or otherwise). However, it would be possible for the testator to take steps now to revoke the will conditional upon his son’s subsequent marriage.

The other options were incorrect because:

  • the will has not yet been revoked (conditionally or otherwise)
  • it is not necessary to wait until after the marriage

incorrect
The testator’s will has not been revoked. The testator should wait until after their son gets married before making a new will.
Incorrect: Review your materials on revocation. Make sure you understand how a testator is able to revoke their will. Is writing “revoked” sufficient?

186
Q

A testator made a valid will four years ago which contained the following clause:

“This will is made in contemplation of any civil partnership I subsequently enter”.

The testator entered into a civil partnership one year later. Two months ago the testator separated from their civil partner and started proceedings to dissolve the civil partnership.

The testator died yesterday before any court order was finalised. There are no other relevant clauses in the will.

Which of the following is correct in respect of the testator’s will?

The will was automatically revoked when the testator separated from their civil partner.

The will was partially revoked when the testator initiated formal proceedings to dissolve the civil partnership.

The will was automatically revoked when the testator entered the civil partnership and the testator died intestate.

The testator’s will has not been revoked as no court order confirming the dissolution of the civil partnership was issued before they died.

The will was automatically revoked when the testator entered the civil partnership. If the testator has made a previous will, their estate will be distributed according to the terms of the previous will.

A

The will was automatically revoked when the testator entered the civil partnership and the testator died intestate.
Correct: By virtue of s 18 B Wills Act 1837 (‘WA 1837’) entering into a civil partnership automatically revokes any previous will made by the testator. The express clause in the testator’s will concerns a hypothetic civil partnership and does not identify a named civil partner – it is therefore ineffective.

The other options were incorrect because:

S 18C WA 1837 does not take effect until a court order confirming dissolution has been issued

S 18B WA 1837- entering a civil partnership revokes all previous wills in full, not just the last one

incorrect
The will was automatically revoked when the testator entered the civil partnership. If the testator has made a previous will, their estate will be distributed according to the terms of the previous will.
Incorrect: Review your materials regarding the effect on a testator’s will of entering a civil partnership and any subsequent dissolution of this. S 18B Wills Act 1837 applies to all previous wills, not just the will made immediately before the civil partnership.

The testator’s will has not been revoked as no court order confirming the dissolution of the civil partnership was issued before they died.
Incorrect: Review your materials regarding the effect on a testator’s will of entering a civil partnership and any subsequent dissolution of this. Focus on the effect of s 18B.

187
Q

A testator died last week. The testator made her first valid will three years ago.

The will appoints the testator’s husband to be joint executor of her estate with her brother. The will contains one gift of £300 to her nephew and the rest of her assets are shared equally between such of her children and husband that survive her.

The testator and her husband finalised their divorce last year. The testator’s husband is still alive.

Which of the following best describes the effect of the will?

The will was automatically revoked in full when the testator divorced. The testator died intestate.

The will is not affected by the divorce as it was executed beforehand. The clauses take effect as stated.

The will was partially revoked when the testator divorced. The testator’s brother will act as sole executor, the testator’s nephew will inherit £300 and the rest of her estate passes to her children only.

The will was partially revoked when the testator divorced. The testator’s former spouse and brother will be appointed executors, the testator’s nephew will inherit £300 and the rest of her estate is shared between her children only.

The will was partially revoked when the testator divorced. The testator’s brother will act as sole executor, the testator’s nephew will inherit £300 and the rest of her estate is shared between her former spouse and children.

A

The will was partially revoked when the testator divorced. The testator’s brother will act as sole executor, the testator’s nephew will inherit £300 and the rest of her estate passes to her children only.

Correct: By virtue of s 18A Wills Act 1837 (‘WA’) the will is interpreted as though the testator’s former spouse has pre-deceased her (a partial revocation). This means only the brother can act as executor and the former spouse cannot take a share of her estate; the rest of her assets (after the gift to her nephew is paid) will be shared between her children only.

The other options were incorrect because:

  • Divorce does not fully revoke the will – the gift to the testator’s nephew would be unaffected
  • S 18A WA prevents the appointment of the former spouse as executor and trustee as well as rendering their beneficial entitlements void
  • S 18A WA affects all wills/codicils made prior to the divorce
188
Q

A testator plans to get married next month. The testator made a valid will (Will 1) years before meeting their fiancé (F) and leaving all of their assets to their sister.

The testator now wants to make a new will (Will 2) leaving most of their assets to their future spouse.

If the testator executes Will 2 before the wedding, which of the following is correct?

Will 2 should not have been executed before the wedding and can only remain effective if the wedding does not take place.

Will 2 will not be revoked by the testator’s marriage, provided it is made expressly in contemplation of the testator’s marriage to F.

Will 2 will not take effect until the testator’s marriage, provided it is made in contemplation of the testator’s marriage to F.

Will 2 will not be revoked by the testator’s marriage, provided it is made expressly in contemplation of the testator’s marriage. No reference to F is required.

Will 1 is unaffected by the testator’s marriage.

A

Will 2 will not be revoked by the testator’s marriage, provided it is made expressly in contemplation of the testator’s marriage to F.

Correct: By virtue of s 18 Wills Act 1837 marriage automatically revokes any will or codicil made beforehand. This can only be avoided if the will includes an express clause stating it is made in contemplation of the marriage to F and should not be revoked by it.

The other options were incorrect because:

  • Whether or not a will is made in contemplation of marriage, the will is effective on execution unless there is express wording that it should only take effect on the occurrence of particular event
  • Will 1 is revoked by the marriage.

Will 2 can be made before the wedding provided it is made expressly in contemplation of that wedding.

189
Q

A woman died last month. In her latest will, which she made five years ago, the woman appointed her partner as her executor and her residuary beneficiary. The woman and her partner entered a civil partnership 18 months ago.

Which of the following is correct?

The woman has died intestate.

The woman has died partially intestate. The civil partner may act as executor but the gift of residue to her lapses.

The woman has died testate. The civil partner may act as executor and the gift of residue to her will be effective, provided there is no extrinsic evidence suggestion the woman wanted someone else to inherit instead.

The woman has died testate. The civil partner may not act as executor but the gift of residue to her will be effective, provided there is no extrinsic evidence suggestion the woman wanted someone else to inherit instead.

The woman has died partially intestate. The civil partner may not act as executor and the gift of residue to her lapses.

A

The woman has died intestate.

Correct. Entering a civil partnership has the effect of revoking any previous will – s 18B Wills Act 1837. There is nothing to suggest the will was made in contemplation of the woman’s subsequent civil partnership – the woman has died intestate.

190
Q

A woman died last month. In her latest will, which she made five years ago, the woman appointed her husband as one her executors and named him as one of her residuary beneficiaries. The woman and her husband divorced 18 months ago.

Which of the following is correct?

The husband may act as executor and the gift of residue to him will be effective, provided there is no extrinsic evidence suggesting the woman wanted someone else to inherit instead.

The husband may act as executor but the gift of residue to him lapses.

The husband may not act as executor and the gift of residue to him lapses.

The husband may not act as executor but the gift of residue to him will be effective, provided there is no extrinsic evidence suggesting the woman wanted someone else to inherit instead.

The woman has died intestate.

A

The husband may not act as executor and the gift of residue to him lapses.

Correct: The effect of s18A Wills Act 1837 treats the ex-spouse named in the will as if they had pre-deceased the testator. This means that ex-spouse cannot be appointed as executor and cannot inherit under the will.

191
Q

A man died recently. The man’s will, which he executed last year, included the following gift:

“I give £500 to my goddaughter “.

The man altered the will a week before he died, by striking through the figure of £500 with a single line.

He had not made any other alteration, carried out any other formality, or taken any other step, and the amount of the gift to the goddaughter is still visible.

Which of the following is correct?

The alteration is valid and the man’s goddaughter will receive nothing.

The alteration is invalid and the man’s goddaughter will receive £500.

The original gift is apparent so the executors can decide whether the man’s goddaughter receives £500 or whether she will receive nothing.

As the alteration was made after the will was executed the will is invalid.

The alteration was made after the will was executed so the will is admitted to probate with a blank space. The man’s goddaughter will receive nothing.

A

The alteration is invalid and the man’s goddaughter will receive £500.

Correct. Any alteration (strike through of the original gift) made after the will is executed is invalid, unless the effect of the alteration is an obliteration (s 21 Wills Act 1837). This amendment is not valid and as the original amount can be identified by natural means the gift remains effective and the man’s goddaughter receives £500.

192
Q

A woman died recently. The woman’s will, which she executed last year, included the following gift, which she then altered with correction fluid a month after execution:

“I GIVE £ to my brother”

Prior to the alteration the gift read “£2,500” but, afterwards, the amount could not be seen at all.

Which of the following is correct?

If the executors submit extrinsic evidence of the original amount the man’s brother will receive £2,500.

The will is valid but the man’s brother receives nothing.

As the alteration was made after the will was executed the will is invalid.

If the executors submit an affidavit of due execution of the will the man’s brother will receive £2,500.

The executors can decide how much the man’s brother should receive.

A

The will is valid but the man’s brother receives nothing.

Correct. Any alteration (the use of correction fluid to cover up the amount of the gift) made after the will is executed is invalid unless the effect of the alteration is an “obliteration” (s 21 Wills Act 1837). As this amendment has completely covered up (obliterated) the number and it is not possible to read what the original gift was, the will is submitted to probate with a blank space and the man’s brother receives nothing.

193
Q

A woman died yesterday and by her will leaves the whole of her estate (except for a £10,000 legacy to a registered charity) to her children. Her assets include a car (£3,000), household possessions (£15,000) and bank accounts (£145,000). A year before she died the woman transferred a property (now worth £200,000) to her niece. The woman continued to live alone in the house until she died and did not pay her niece any rent.

In the tax year of the woman’s death, the basic nil rate band is £325,000 and the main residence nil-rate band is £175,000.

The woman made no lifetime gifts. Her debts and funeral costs are £12,000.

How much inheritance tax will be payable on the woman’s estate?

£10,400

£6,400

£0

£136,400

£11,200

A

£6,400
Correct: The woman’s taxable estate includes her car, personal possessions, bank accounts and also the current value of the property. Although she was not the legal owner of the property when she died, its value is included in her taxable estate because she made a gift with reservation of benefit. The total value of her taxable estate is: £363,000. Her debts and funeral costs of £12,000 are deducted to leave £351,000. After applying charity exemption of £10,000, £341,000 remains.

No RNRB applies as she did not leave the residence to a lineal descendent on her death. After the basic NRB of £325,000 is applied, £16,000 remains chargeable at 40%, which gives £6,400 of IHT payable.

The other options were incorrect because:

  • The value of the property was excluded (a gift with reservation of benefit) but should not have been
  • No deduction was made for debts and funeral expenses
  • No charity exemption was applied
  • The RNRB should not have been applied
  • The basic NRB was not applied but should have been

incorrect
£0
Incorrect: Refer to your materials relating to the calculation of IHT in respect of the death estate. You did not consider the gift with reservation of benefit rules in respect of the property. Or, you have applied a RNRB when the woman’s estate does not qualify.

£10,400
Incorrect: Refer to your materials relating to the calculation of IHT in respect of the death estate. You did not apply charity exemption.

194
Q

A woman died recently. At the date of her death she owned a residential property, a commercial property and cash in a savings account.

The woman had a pension which paid a discretionary lump sum following her death. The woman had nominated her children as the beneficiaries.

The woman had a remainder interest in a trust set up following the death of her brother. The woman’s mother is the life tenant and she is still alive.

Which of the following best describes the assets included in the woman’s taxable estate?

Both properties, her savings account and the discretionary pension lump sum.

The commercial property and her savings account.

The commercial property, her savings account, the discretionary pension lump sum and her remainder interest in her brother’s will trust.

Both properties, her savings account and her remainder interest in her brother’s will trust.

Both properties and her savings account.

A

Both properties and her savings account.

Correct
Correct. Residential and commercial properties are included, as is the cash balance of any bank account. The pension lump sum is excluded because it is discretionary in nature and has been nominated for her children - no money is payable to her estate. Her remainder interest in the trust is excluded property for inheritance tax purposes (the life tenant is still alive so the trust capital had not been paid to her before she died). The incorrect options either excluded an item that should have been included, or included an item that should not have been.

195
Q

A woman died recently leaving an estate comprising her home (£350,000), bank accounts (£2,000) and private company shares, which were purchased one year before she died (£30,000). The cost of her funeral was £7,000.

The woman had never married or entered a civil partnership and had made no lifetime gifts.

The whole of the woman’s estate will pass to her nephew.

In the tax year of her death the NRB is £325,000 and the main residence NRB is £175,000.

How much inheritance tax is due following the woman’s death?

£0

£8,000

£14,000

£20,000

£150,000

A

£20,000
Correct. Step 1: She made no lifetime gifts so has a cumulative total of 0. Step 2/3: The woman’s estate is worth £382,000 (home + bank account + shares). Step 4: Funeral costs (£7,000) can be deducted leaving £375,000. Step 5: No BPR applies because she did not own the company shares for at least 2 years before she died. Step 6: No residence NRB applies as the house passes to her nephew. Step 7: The woman’s estate benefits from her own full NRB (there is no transferred amount as she never married). £375,000 less £325,000 leaves balance of £50,000 @ 40% = £20,000.

incorrect
£8,000
Incorrect. Your answer is too low. Review each step of the calculation and in particular think about whether the estate qualifies or not for a Residence or Transferred Nil Rate Band.

196
Q

A man inherits his wife’s business which she had owned for 10 years. One year later he makes a potentially exempt transfer (PET) of the business to his son. Six months later the son sells the business with the man’s approval. The man dies a further six months later.

Which of the following best describes how business property relief (BPR) applies to the man’s lifetime gift of the business (failed PET) following the man’s death?

BPR is available as the man was still alive when his son sold the business.

BPR is not available on the failed PET as the son no longer owned the business when his father died.

BPR is not available on the failed PET as the man had not owned the interest in the business for two years when he made the gift.

BPR is available on the failed PET and reduces the value of the gift.

BPR is available on the failed PET and reduces the tax payable on the gift.

A

BPR is not available on the failed PET as the son no longer owned the business when his father died.

Correct: The man’s lifetime gift of the business to his son is a failed PET as the man died within 7 years of making the gift. When assessing the failed PET, BPR can be claimed only if i) the assets would have qualified for relief when the gift was made by the man, and, ii) they still qualify for relief in the hands of the son. Here, the gift would have qualified for BPR when it was made because the man can include his spouse’s period of ownership (he inherited the assets following her death) and so satisfy the two-year ownership requirement. However, when the man died the son no longer owned the property and so BPR does not apply. BPR cannot be claimed in respect of the man’s lifetime gift.

incorrect

BPR is not available on the failed PET as the man had not owned the interest in the business for two years when he made the gift.
Incorrect: Revise your materials on BPR. Note that the man inherited the business from his wife. What effect does this have on the qualifying period of ownership?

BPR is available on the failed PET and reduces the tax payable on the gift.
Incorrect: Revise your materials on BPR. Note that the son no longer owned the business when the man died. You should also revisit the effect of BPR. It reduces the value of the gift, rather than the amount of tax payable

BPR is available on the failed PET and reduces the value of the gift.
Incorrect: Revise your materials on BPR. Note that the son no longer owned the business when the man died.

BPR is available as the man was still alive when his son sold the business.
Incorrect: Revise your materials on BPR. Is it relevant that the man was still alive when his son sold the business? At this point it is still a PET and not chargeable.

.

197
Q

A man who had owned his farm for 30 years ceased farming 5 years ago when he let the farmland on a long agricultural tenancy. The man continued to live in the farmhouse as his home. The man has just died.

Which of the following best describes how agricultural property relief (APR) will apply to the man’s estate?

APR will not be available as the tenant had not occupied the property for 7 years.

APR will be available on the farmland at 50% but will not be available on the farmhouse.

APR at 50% will be available on the farmland and farmhouse.

APR will be available on the farmland at 100% but will not be available on the farmhouse.

APR at 100% will be available on the farmland and the farmhouse.

A

APR will be available on the farmland at 100% but will not be available on the farmhouse.

Correct: Where the land is not farmed by the owner at the date of the transfer (here being death) it must have been owned by the transferor and occupied for agricultural purposes by the owner or another for 7 years. That requirement is satisfied as the occupation for agricultural purposes by the farmer is added to the 5 years of occupation by the tenant. As the tenancy although a long tenancy was a granted after 2005 100% relief is available. APR is not available on the farmhouse as it is no longer occupied for agricultural purposes.

incorrect
APR at 100% will be available on the farmland and the farmhouse.
Incorrect: APR at 100% is available on the farmland for the reason set out above. APR will not be available on the farmhouse as it is no longer occupied for agricultural purposes.

198
Q

A testator made a will that contained the following gift: “I give all of myshares inX ltd to any of my niecesalive at the date of my death and if more than one in equal shares”.

The testator owned 12,000 shares in X ltd, and had three nieces,when the will was executed.

The testator owned 24,000 shares in X ltd, and was survived by four nieces (aged 2, 10, 15 and19), when he died.

Which of the following is correct?

A – Each of his four nieces will receive 6,000 of the 24,000 shares he owned on the date he died.

B – The three nieces alive at the date the will was executed will each receive 8,000 of the 24,000 shares he owned on the date he died. It is not possible to leave a legacy to a beneficiary who does not exist when the will is made.

C – Each of his four nieces will receive 3,000 of the 12,000 shares he owned when the will was executed.

D – The class closing rules apply and it is necessary to see if the youngest niece reaches the age of 18 before it is possible to determine who is entitled to inherit the shares.

E – The class closing rules apply and because the eldest niece is the only beneficiary to reach the age of 18 on the date of the testator’s death, she will inherit all of the shares.

A

A – Each of his four nieces will receive 6,000 of the 24,000 shares he owned on the date he died.

correct

consolidation recording w11 goes through answers

199
Q

A woman died intestate and her estate will be distributed to her spouse and children under the intestacy rules.
The woman’s spouse does not want to inherit anything from the woman and would prefer to redirect all of her inheritance to her own children from a previous marriage.

You are not required to consider tax consequences.

Which of the following is correct?

A – The woman’s spouse could only achieve her objectives by disclaiming her inheritance.

B – The woman’s spouse could achieve her objectives by entering a deed of variation or by disclaiming her inheritance.

C – The woman’s spouse cannot achieve her objectives by either a variation or by disclaiming her inheritance.

D – The woman’s spouse cannot vary her inheritance because the woman died intestate.

E – The woman’s spouse cannot disclaim her inheritance because she wants her share to pass to specific beneficiaries.

A

E – The woman’s spouse cannot disclaim her inheritance because she wants her share to pass to specific beneficiaries.

correct

consolidation recording w11 goes through answers

200
Q

A man died three months ago and by his will left his entire estate to his girlfriend. The following family members want to make a claim against the man’s estate under the Inheritance (Provision for Family and Dependants) Act 1975 (the ‘Act’):
The man’s former wife. The man had paid his ex-wife maintenance on an ad hoc basis after they got divorced but stopped when she remarried three years ago.
The man’s elderly grandfather who received a weekly allowance of £100 from the man until the date the man died.
The man’s son from his marriage. The man provided his son with financial support while he was at university but stopped these payments six months ago when his son started working.

Who is able to bring a claim under the Act against the man’s estate?

A – The man’s son only.

B – The man’s grandfather and son.

C – The man’s former spouse and grandfather.

D – The man’s grandfather only.

E – The man’s former spouse, grandfather and son.

A

B – The man’s grandfather and son.

correct

former spouses can bring claims only if they have not remarried!

consolidation recording w11 goes through answers

201
Q

A woman makes a will that gives cash legacies to her friends and the residue of her estate to her sister.

The woman’s estate comprises a house, cash savings and household personal possessions.

IHT will be payable following the woman’s death. The woman had made no lifetime transfers in the tax year of her death or the previous tax year.

What is the total value of the available exemptions and reliefs that will apply in respect of the woman’s death estate?

A – £6,250
B – £6,000
C – £0
D – £3,000
E – £3,250

A

C – £0

correct

not D as the 3000 exemption is only for lifetime transfers!!

consolidation recording w11 goes through answers

202
Q

A woman died yesterday and by her will left £10,000 to charity and the remainder of her estate to her daughter. The woman had never married or entered a civil partnership and did not make any lifetime gifts.

Her estate comprised her home (£450,000), bank accounts (£40,000), car (£13,000) and household possessions (£5,000). The woman’s debts and funeral expenses total £1,000.

In the tax year of her death the nil-rate band is £325,000 and the main residence nil-rate band is £175,000.

How much inheritance tax will be payable on the woman’s estate?

A – £0

B – £72,800

C – £2,800

D – £1,400

E – £68,800

A

A – £0

correct

3 Taxable estate: 450 + 40 + 13 + 5 + = 508,000
4 Deduct expenses: 508 - 1 = 507
5 Exemptions: 507 - 10 = 497
6 RNRB: 497 - 175 = 322
7 RNB + IHT = 322- 325 = <0 ! No tax

consolidation recording w11 goes through answers

203
Q

By his will, a testator appointed his spouse, his friend and his adult son to be his executors. The testator has now died.
The testator and his wife divorced after the will was executed.
The son predeceased the testator and a grant of probate to the son’s estate was obtained by his nephew.
The testator left all of his estate to his niece who is 20 years of age.

Who has the best right to apply for a grant of representation to the testator’s estate?

A. The testator’s ex-spouse, the testator’s friend and the nephew of the son only.

B. The testator’s friend and the testator’s niece only.

C. The testator’s friend, the nephew of the son and the testator’s niece only.

D. The testator’s friend only.

E. The testator’s friend and the nephew of the son only

A

D. The testator’s friend only.

consolidation recording w11 goes through answers

204
Q

A man died intestate a month ago. He had never been married or in a civil partnership. At the time of his death, the man was living with his partner, with whom he had been cohabiting for 20 years, and her daughter (aged 23 years).
The partner’s daughter has lived with the man and his partner throughout their relationship.
The man had a son (aged 25 years) from a previous relationship and a daughter (aged 19 years) whom he and his partner adopted ten years ago.

Who is entitled to share in the distribution of the man’s estate?

A. The partner, the son and the man’s daughter only.

B. The son, the man’s daughter and the partner’s biological daughter only.

C. The son and the man’s daughter only.

D. The son only.

E. The man’s daughter only.

A

C. The son and the man’s daughter only.

consolidation recording w11 goes through answers

205
Q

A man dies intestate a month ago. The man’s estate comprises only:
Cash £294,000
Chattels £6,000
The man is survived by a spouse and adult daughter. His son died two years ago leaving children of his own (the man’s grandchildren) a daughter who is aged 18 and married and a son aged 12.

Which of the following statements best explains how the man’s estate will be distributed?

A – The spouse is entitled to the whole of the man’s estate.

B – The spouse is entitled to the man’s personal chattels and £270,000. The daughter, granddaughter and grandson are each entitled to £8,000. The daughter and granddaughter will have a vested interest and the grandson will have a contingent interest.

C – The spouse is entitled to the man’s personal chattels and £282,000. The daughter and granddaughter will each have a vested interest in £6,000.

D – The spouse is entitled to the man’s personal chattels and £282,000. The daughter will have a vested interest in £6,000, the granddaughter will have a vested interest in £3,000 and the grandson will have a contingent interest in £3,000.

E – The spouse, daughter, granddaughter and grandson are each entitled to £75,000. The spouse and daughter will have vested interests and the granddaughter and grandson will have contingent interests.

A

D – The spouse is entitled to the man’s personal chattels and £282,000. The daughter will have a vested interest in £6,000, the granddaughter will have a vested interest in £3,000 and the grandson will have a contingent interest in £3,000.

correct

the spouse is entitled to 270k and half of what is left over

the issues are entitled the the other half in equal shares

the predeceased issue’s own issue (ie the grandchildren) have equal shares

consolidation recording w11 goes through answers

206
Q

Under the terms of a man’s will, the whole of his estate was given to his daughter. The man had a serious disagreement with his daughter and decided that he no longer wanted her to be the beneficiary of his will. He tore his original will into four pieces.
The man and his daughter are now reconciled and the man regretted his earlier decision. He wants his daughter to inherit the whole of his estate.

Is the man’s will valid?

A – Yes, because his daughter is the beneficiary named in the will and the man intends her to inherit the whole of his estate.

B – Yes, because a will must be revoked by a revocation clause in a new will executed in accordance with s.9 Wills Act 1837.

C – Yes, because the doctrine of dependent relative revocation will apply.

D – No, because the will was revoked by destruction.

E – No, because the will must be read as a whole and this is no longer possible.

A

D – No, because the will was revoked by destruction.

correct

consolidation recording w11 goes through answers

207
Q

A woman died yesterday having made the following lifetime transfers:
Gift 1: £100,000 gift to her son in January 10 years ago
Gift 2: £20,000 gift into trust in March 5.5 years ago
Gift 3: £5,000 to her daughter on the event of her wedding in May 2 years ago
Gift 4: £6,000 to her grandson 2 days after Gift 3

Which of the following is correct in respect of the woman’s lifetime transfers?

A – The annual exemption cannot be claimed for Gift 4 because it was allocated to Gift 3 (made earlier in the same tax year).

B – Taper relief can be claimed in respect of the inheritance tax payable following death in respect of Gift 2.

C – The value of Gift 1 was relevant when calculating the inheritance tax charge for Gift 2 when the transfer was made.

D – Inheritance tax in respect of Gift 2 was paid at the lifetime rate of 20% when the transfer was made.

E – The chargeable value of Gift 4 is £0.

A

E – The chargeable value of Gift 4 is £0.

correct
consolidation recording w11 goes through answers

wrong
A – gift 3 is tax-exempt (5000 for marriage from parent per person) so both annual exemptions can be used for gift 4

B – no taper relief as there is no tax! (same reason as D) taper does not adjust value of transfer - taper relief adjusts the value of the liability once the tax has been calucualted

C – no because earlier PET didn’t fail

D – wrong as no tax was paid as there was no tax liability

LCT
- cumulative total is 0
- Value of transfer: 20k
(-6k: annual exemptions x2)
- chargeable value: 14k
- apply NRB: no tax payable as value within NRB

208
Q

A man dies testate. His will leaves the residue of his estate equally between his children provided they reach the age of 21. There is an express substitution clause in the will which specifies that if any of his children do not survive him, their share passes absolutely (i.e. there is no age contingency) to their own children (the man’s grandchildren).

The man is survived by his spouse and three children A (25), B (19), C (16). The man had another child (D) who died before him. D’s spouse (S) and their two children X (3) and Y (6) survive the man.

Which of the following most accurately states the beneficiaries with a vested interest in the residue of man’s estate?

Select one alternative:

S, A and B

S, A, B, C, X and Y

A and B

A, B and C

A, X and Y

A

A, X and Y

A has a vested interest in the residue as A has reached the contingent age of 21. B and C both only have contingent interests as they are under the age of 21 (that B is over the age of 18 does not matter). D’s share passes by substitution to D’s children X and Y who are both minors. There are no contingent age requirements attached to the substitution under the will so both X and Y will have vested interests in the part of the man’s estate that would have passed to D. Note that a minor beneficiary may have a vested interest although they cannot give good receipt for capital until they reach 18. S does not inherit under the man’s will.

209
Q

A woman died recently. Ten years before she died the woman gave a holiday cottage she owned to her children. The children are the registered owners of the property. After making the gift, the woman continued to stay in the cottage each year up until she died. She did not pay her children rent for her use of the cottage.

The property was worth £100,000 when she made the gift and was worth £150,000 when she died.

Which of the following is correct in relation to the woman’s inheritance tax estate?

Select one alternative:

The holiday cottage is excluded because she gave it away more than 7 years before she died.

The holiday cottage is included at the value of £150,000.

The holiday cottage is included at the value of £50,000.

The holiday cottage is excluded because she was not the legal owner at the date of her death.

The holiday cottage is included at the value of £100,000.

A

The holiday cottage is included at the value of £150,000.

The woman has made a gift with reservation of benefit. Although she is no longer the legal owner of the holiday cottage, for inheritance tax purposes she is treated as never having given the cottage away because she continued to benefit from it without paying rent. Therefore, the value of the cottage at the date of her death is included in her estate. The other options were wrong because: i) the cottage did not fall outside of her taxable estate, ii) the value at the date of death is relevant, not the increase in value since the gift was made and not the value of the property when it was given away. Note that the date of death value is used whether the value of the asset has increased or decreased since the date of the gift.

210
Q

A woman died intestate. She was survived by her civil partner (CP), her parents and her children, A (20) and B (15). The woman’s civil partner died two weeks after the woman.

The woman’s net estate available for distribution is £400,000 (which includes her chattels worth £50,000)

Which of the following accurately describes the distribution of the woman’s estate under intestacy?

Select one alternative:

CP is entitled to the woman’s chattels and a statutory legacy (together worth £270,000) and half of the remainder. The other half of the remainder is divided equally between A and B absolutely.

The woman’s parents are each entitled to £200,000 absolutely.

A and B are each entitled to £200,000 on statutory trust.

CP is entitled to £400,000 absolutely.

CP is entitled to the woman’s chattels (£50,000), and a statutory legacy of £270,000, and half of the remainder. The other half of the remainder is divided equally between A and B on statutory trust.

A

A and B are each entitled to £200,000 on statutory trust.

The woman’s civil partner did not survive her by 28 days so the woman’s estate is distributed under intestacy on the basis that she had issue but no surviving civil partner. In this situation the whole of the woman’s estate is divided equally between her children on statutory trusts. The other options were incorrect because neither the woman’s civil partner nor parents are entitled to take a share.

211
Q

A man died recently leaving a will that gives the whole of his estate to charity. The man is survived by his spouse and her daughter (the man’s step-daughter) who is aged 15. Neither the man’s spouse nor his step-daughter were living with him at the date he died.

When he died, the man was living in a flat that he owed as joint tenants with his brother.

Which of the following is correct with regards a claim against the man’s estate under the Inheritance Provision for Family and Dependants Act 1975 (the ‘Act’)?

Select one alternative:

The man’s spouse cannot make a claim under the Act because she did not live with the man at the date he died.

The court does not have the power to make an award under the Act in respect of the man’s flat because this passes to his brother by virtue of the survivorship rules.

The man’s step-daughter cannot make a claim under the Act because she is not his biological child.

An award made to the man’s surviving spouse would be limited to the amount she required for her maintenance.

The man’s brother is only entitled to bring a claim under the Act if he was being financially maintained by the man at the date of his death.

A

The man’s brother is only entitled to bring a claim under the Act if he was being financially maintained by the man at the date of his death.

A person can only make a claim under the Act if they fall within one of the specific categories of applicant. The deceased’s siblings have no automatic entitlement to apply by virtue of their familial relationship. The only category that might apply to the man’s brother is a ‘person maintained’ by the deceased. The other options were incorrect because i) a claim by the man’s spouse would be assessed with reference to what was reasonable in the circumstances (the surviving spouse standard) and is not limited to what she needs for maintenance. ii) the man’s step-daughter could apply as someone ‘treated as a child’ of the deceased (or perhaps as someone maintained). i.e. that she is not the deceased’s biological child does not preclude her from making an application, iii) there is no requirement for a spouse to be living with the deceased to make an application under the Act and iv) there are circumstances were the court does have the power to make an order in respect of jointly owned property passing outside of the succession estate.

212
Q

An elderly testator recently executed his will. The testator suffers with arthritis in his hands and, although there is no doubt regarding his capacity, on the day of execution, he was unable to hold a pen and instructed his solicitor to sign on his behalf.

The will was signed by the solicitor on behalf of the testator, and witnessed by two independent adults. The testator, solicitor, and both witnesses were all present throughout the execution process.

The following attestation clause was included in the testator’s will:

“Signed by the above named Testator in our joint presence and then by us in his”

Select one alternative:

There is no presumption that the testator had knowledge and approval.

There is a presumption that the testator had knowledge and approval because the will was executed in accordance with the requirements of s 9 Wills Act 1837.

The will is invalid because the attestation clause does not specify that the solicitor signed on behalf of the testator.

The will is invalid because the testator did not sign it himself.

There is a presumption that the testator had knowledge and approval because the testator had testamentary capacity.

A

There is no presumption that the testator had knowledge and approval.

The presumption of knowledge and approval does not apply because the will was signed by someone else on behalf of the testator and this was not expressly confirmed in the attestation clause. The other options were incorrect because although a presumption of knowledge and approval would usually apply where the testator has capacity and has executed the will in accordance with s 9 requirements, this does not apply where the testator themselves does not sign (and the attestation clause does not reflect what occurred). A will can be validly executed even if not signed by the testator, provided that is it signed by someone else at the testator’s direction and in his presence (as occurred in this case). The inclusion of an attestation clause (correctly drafted or not) is not a requirement for a will to be valid.

213
Q

A man (M) and his unmarried adult daughter (D) both die intestate as a result of injuries sustained following a car accident. The man dies 2 days before his daughter. The man and his daughter are survived by the man’s adult son (S).

Both D and S have their own children (the man’s grandchildren), all of whom are alive and are under the age of 18.

Which of the following is correct?

Select one alternative:

D’s half share of M’s estate passes under M’s intestacy and is divided equally between S and any of D’s children who reach 18 or marry beforehand.

D’s half share of M’s estate passes under D’s intestacy to her children absolutely.

D’s half share of M’s estate passes under M’s intestacy to S absolutely.

D’s half share of M’s estate passes under D’s intestacy to any of her children who reach 18 or marry beforehand.

D’s half share of M’s estate passes under M’s intestacy to D’s children by virtue of the substitution limb of the statutory trusts.

A

D’s half share of M’s estate passes under D’s intestacy to any of her children who reach 18 or marry beforehand.

The man’s estate will be divided equally between his son and daughter under the terms of his intestacy. Each of D and S will have a vested interest as they are adults and there is no minimum period of survivorship required. When D later dies intestate, her estate (which includes the half share of her father’s estate) passes under her own intestacy to her children. Her children inherit on the terms of the statutory trusts, i.e. contingent on reaching the age of 18 or marrying beforehand. The other options were incorrect because: i) D has a vested interest in the half-share of M’s estate so these assets pass under her intestacy not his, and ii) Under D’s intestacy, her children inherit contingent on reaching the age of 18 (or marrying beforehand). As they are minors they do not inherit absolutely.

214
Q

A client wants advice on the inheritance tax position following their death.

The client currently owns a runs a small organic farm which includes the farm house in which the client lives with his wife. The client purchased the farm in his sole name three years ago and the current market value is £1M. The agricultural value is £500,000.

Which of the following correctly states the position with regards agricultural property relief (APR) assuming the client dies tomorrow?

Select one alternative:

APR of £1M can be claimed.

No APR can be claimed because the client has not owned and occupied the farm for at least 7 years.

If the client left his estate to his spouse, the amount of spouse exemption available would be greater than the value of any APR that could otherwise be claimed.

No APR can be claimed in relation to the value of the farm house because it is used for residential purposes.

APR of £500,000 can be claimed.

A

If the client left his estate to his spouse, the amount of spouse exemption available would be greater than the value of any APR that could otherwise be claimed.

There is no limit to the amount of spouse exemption that can apply (the exemption is 100% of the value of the items inherited by the deceased’s spouse). The amount of APR available is limited to the agricultural value of the exempt assets. The other options were incorrect because: i) it is possible for APR to apply to a farm house provided it is of a character appropriate to the farm, ii) as the client owns and occupies the land for farming purposes, the minimum period of ownership is 2 years not 7, iii) APR is claimed in respect of the agricultural value of the land, not the market value.

215
Q

A man died yesterday. He was survived by his mother (M), sister (S), his long-term partner (P) and their three children, A (21), B (19) and C (18). The man had not made a will.

Who has the best entitlement to apply for a grant of representation in the man’s estate?

Select one alternative:

M

At least two of A, B and C (two applicants are required).

S

P

Any one of A, B or C

A

Any one of A, B or C

The testator died intestate and his three children will share his estate between them. No minority interest arises so only one PR is required. Rule 22 NCPR 1987 applies as the man died without making a will. Under NCPR 22 the beneficiaries of the estate (his children) have the best right to apply. They are all adult so have an equal entitlement. The other options were incorrect because M, S and P have no right to apply under NCPR 22.

216
Q

A house is owned in equal shares by a man and a woman who lived together as an unmarried couple. The man has just died. A local estate agent has valued the house at £600,000.

What is the value of the man’s half share of the property for inheritance tax purposes?

Select one alternative:

£300,000 - if the man and woman owned the house as joint tenants no discount is available.

£300,000 - no discount is available because of the application of the related property rules between cohabitees.

£300,000 less a percentage discount to reflect the fact that the house was co-owned at the man’s death with someone who was not his spouse.

£300,000 - no discount is available because the man and woman were not married when they purchased the house.

£300,000 - if the man and woman owned the house as tenants in common no discount is available.

A

£300,000 less a percentage discount to reflect the fact that the house was co-owned at the man’s death with someone who was not his spouse.

The value of the man’s half share can be discounted because he owned the property jointly with someone who is not his spouse/civil partner. The other options were incorrect because the related property rules do not apply in this case as the couple were not married. It is irrelevant whether the property was owned as joint tenants or tenants in common for these purposes.

217
Q

A man died leaving a valid will that includes the following gifts:

I give my chattels to my nephew
I give £30,000 to my niece
There are no other gifts in the will.

The man’s estate includes his chattels, cash of £50,000 and a house in his sole name (worth £500,000).

The man is survived by his niece, nephew, spouse and adult son. The man’s daughter had pre-deceased him leaving her own daughter (the man’s granddaughter) who also survives the man.

Which of the following best describes who will inherit the man’s estate?

Select one alternative:

The man’s nephew, niece and spouse.

The man’s nephew, niece, spouse, son and grand-daughter.

The man’s nephew and niece.

The man’s spouse, son and grand-daughter.

The man’s nephew, niece, spouse and son.

A

The man’s nephew, niece, spouse, son and grand-daughter.

The man has died partially intestate because the will does not deal with all of his estate. The cash legacy and chattels are given to the man’s niece and nephew under the terms of his will. The other assets (the cash balance and the house) pass in accordance with the intestacy rules. Under intestacy, the man’s spouse and children will receive a share of these assets. The man’s daughter cannot inherit as she has pre-deceased the man, but her daughter (the man’s granddaughter) will receive the share of the estate that would have passed to her parent under the substitution limb of the statutory trusts. The other options were incorrect because they did not include all of the beneficiaries.

218
Q

A collection of rare books was owned by a woman. Ten years ago, when the collection was worth £9,000, she gave one third of the collection to her sister.

The woman’s sister has recently died leaving all of her assets to her nephew, including the share of the book collection the woman gifted to her. Inheritance tax will be payable on the sister’s estate.

The value of the sister’s share of the collection when she died was £2,000. The market value of the whole collection was £12,000.

What is the value of the sister’s share of the book collection for inheritance tax purposes?

Select one alternative:

£3,000

£0

£2,000

£6,000

£4,000

A

£2,000

The related property rules do not apply in this scenario because the woman and her sister were not married or in a civil partnership. The market value of the 1/3 share owned by the sister on the date she died is the correct value for inheritance tax purposes. The other options were incorrect because they: i) applied the related property rules ii) considered the gift from the perspective of the donor not the donee or iii) considered the value at the date of the gift not the date of death.

219
Q

A testator died leaving a will by which all of his estate was shared equally between his adult grandchildren, A and B. The estate assets included quoted company shares worth £200,000 on the date of the testator’s death.

A and B were each transferred half of the testator’s shares, which had increased in value to £240,000 by the date of the transfer, so A and B each received shares with a value of £120,000.

Eighteen months later A sold their shares for £130,000. A few months afterwards B sold their shares for £100,000.

Which of the following statements best describes the capital gains tax position?

Select one alternative:

A makes a gain of £30,000. B makes neither a loss nor a gain.

The PRs of the estate have made a loss of £10,000.

The PRs of the estate have made a gain of £40,000.

A and B both make a gain of £20,000.

A makes a gain of £10,000. B makes a loss of £20,000.

A

A makes a gain of £30,000. B makes neither a loss nor a gain.

Where assets in an estate are transferred to a beneficiary by the PRs there is no capital gains tax disposal. The beneficiaries are treated as receiving the assets at their date of death value. So here, the shares were worth £200,000 in total on the date of death (with a half share being worth £100,000). This means A and B are both treated as receiving the shares with an acquisition cost of £100,000. When A sells the shares later for £130,000, A makes a gain of £30,000, and when B sells the shares later for £100,000, B has made neither a loss nor a gain. The other options were incorrect because: i) the gain/loss made by A and B was incorrectly described with reference to the transfer value of the shares (instead of the date of death value), ii) the gain/loss made by A and B was incorrectly described with reference to the increase in value between the date of death and date of transfer, iii) the PRs have not made a disposal so have not made a gain or loss for capital gains tax purposes.

220
Q

A man’s estate includes the following:

Property owned as joint tenants with his spouse (value of whole £600,000, value of half £300,000, and value of half with 10% discount £270,000)
Life interest in a will trust created by his father (trust fund value at creation £50,000 and value on the date of the man’s death, £75,000)
Bank accounts (£3,000)
What is the value of the man’s gross taxable estate?

Select one alternative:

£378,000

£273,000

£323,000

£348,000

£678,000

A

£378,000

The man’s taxable estate includes the property, the life interest trust and his bank accounts. The man’s half share of the property is valued at £300,000 (there is no discount as related property rules apply - his spouse is the co-owner). The relevant value for the will trust is the value at death (not creation) here £75,000. Adding the bank accounts of £3,000 gives a total of £378,000.

221
Q

A woman died last month leaving a valid will that gives her collection of watercolours to her friend and the residue of her estate to her nephew. In addition to the woman’s house, her estate includes a car (worth £5,000), the collection of watercolours (worth £1,000) and a bank account (£20,000).

The personal representatives (‘PRs’) must raise £7,400 to pay inheritance tax (‘IHT’) and will not be using the instalment option.

Which of the following is the best advice for the PRs?

Select one alternative:

The PRs should use the Direct Payment Scheme in relation to the funds in the woman’s bank account. The bank will pay a sum equal to the IHT bill directly to the beneficiaries.

The PRs should use the Direct Payment Scheme in relation to the funds in the woman’s bank account. The bank will pay a sum equal to the IHT bill directly to HMRC.
Is correct

The PRs should sell the car and watercolours and use the sale proceeds to make part payment. They may defer paying the balance for a further 6 months.

The PRs should use the Direct Payment Scheme in relation to the funds in the woman’s bank account. The bank will pay a sum equal to the IHT due directly to the PRs, which they must then use to pay HMRC.

The PRs should sell the car and the watercolours and use the sale proceeds plus cash from the bank account to fund the IHT.

A

The PRs should use the Direct Payment Scheme in relation to the funds in the woman’s bank account. The bank will pay a sum equal to the IHT bill directly to HMRC.

222
Q

A woman died intestate 6 weeks ago. Her personal representatives (‘PRs’) wish to protect themselves against the possibility that a personal claim is brought against them by one of woman’s creditors of whom they have no notice.

Which of the following best describes the steps the PRs should take before distributing the woman’s estate?

Select one alternative:

They should place an advert in the London Gazette (and a newspaper local to the area in which the woman’s house was situated) and wait 2 months from the date of death.

They should wait until 2 months after the date of death to place an advert in the London Gazette (and a newspaper local to the area in which the woman’s house was situated).

They should place an advert in the London Gazette (and a newspaper local to the area in which the woman’s house was situated) and wait 2 months from the date of the advert.

They should wait until 2 months after the date the grant of representation is issued to place an advert in the London Gazette (and a newspaper local to the area in which the woman’s house was situated).

They should place an advert in the London Gazette (and a newspaper local to the area in which the woman’s house was situated) and wait 2 months from the date the grant of representation is issued.

A

They should place an advert in the London Gazette (and a newspaper local to the area in which the woman’s house was situated) and wait 2 months from the date of the advert.

If the PRs follow the s.27 Trustee Act 1925 notice procedure they are protected from personal liability to pay creditors who were not paid from estate during the administration because the PRs were unaware at the time of their right to any of the estate – i.e. unknown creditors. The notice procedure requires the PRs to place an advert as stated in the question and wait 2 months from the date of the notice, after which they may distribute the estate. The other options were wrong because the time limits were incorrectly stated.

223
Q

A woman made a valid will leaving all of her estate to her brother.

A few day before she died and believing she was unlikely to survive much longer, the woman gave her gold ring to her granddaughter and said “I want you to have this, but only in the event that I die”.

The woman also owned the following assets in her sole name:

House (owned as tenants in common with her spouse)
Life insurance policy (not written in trust)
Chattels
Cash in bank account
Assume that all debts and expenses are paid from the woman’s bank accounts.

Which assets will the woman’s brother be entitled to?

Select one alternative:

The woman’s half share of the house, life insurance proceeds, chattels (excluding the gold ring) and the remaining cash in the bank account.

The woman’s half share of the house, life insurance proceeds and the remaining cash in the bank account.

The woman’s half share of the house, life insurance proceeds, chattels (including the gold ring) and the remaining cash in the bank account.

Chattels (excluding the gold ring) and the remaining cash in the bank account.

Chattels (including the gold ring) and the remaining cash in the bank account.

A

The woman’s half share of the house, life insurance proceeds, chattels (excluding the gold ring) and the remaining cash in the bank account.

The woman’s brother is only entitled to receive the assets passing as part of her succession estate. This includes the share of the house owned as tenants in common (it is joint tenant property that passes outside the succession estate), life policy proceeds (because these have not been written in trust), chattels (her spouse is not automatically entitled to these where there is a will) and the cash balance in her bank account. The woman’s succession estate excludes the gold ring which is the subject of a donatio mortis causa.

224
Q

A man died and by his will left all his estate to his wife. He made no lifetime gifts.

The man’s wife (W) died recently and by her will leaves all her estate to her children. She made no lifetime gifts.

W’s taxable estate (after debts and liabilities have been paid) is worth £800,000 and comprises a single property which has always been let on a commercial basis (£600,000) and her various bank accounts.

In the tax year of W’s death the NRB is £325,000 and the main residence NRB is £175,000.

How much inheritance tax is due following W’s death?

Select one alternative:

£120,000

£30,000

£190,000

£0

£60,000

A

£60,000

Step 1: W made no lifetime gifts so her cumulative total is 0. Step 2/3/4: The value of her estate after debts and liabilities is £800,000. Step 5: No exemptions or reliefs apply on facts given. Step 6: No RNRB applies as she did not own a residential property. Step 7: W’s PRs can claim 2 x basic NRB (W’s and a transferred amount from her pre-deceased spouse). We know the man did not use any of his own (he made no lifetime gifts and spouse exemption applied to the whole of his death estate) so the full amount can be transferred. £800,000 - £325,000 - £325,000 leaves £150,000 chargeable at 40%= £60,000.

225
Q

A testator died four months ago leaving a valid will appointing her brother as her sole executor and leaving her entire estate to her adult nephews. The testator was survived by her spouse and her nephews.

The testator’s brother predeceased her and left a will appointing an executor who took out the grant of probate in respect of the brother’s estate.

Which of the following most accurately states the grant required for the testator’s estate and who will apply?

Select one alternative:

Both nephews (two PRs are required) will apply for a grant of letters of administration with will annexed.

The brother’s executor will apply for a grant of probate.

The spouse will apply for a grant of letters of administration with will annexed.

The brother’s executor will apply for a grant of double probate.

Either nephew (only one PR is required) will apply for a grant of letters of administration with will annexed.

A

Either nephew (only one PR is required) will apply for a grant of letters of administration with will annexed.

The deceased left a valid will but the sole executor appointed under the will pre-deceased the testator the relevant grant is letters of administration with will annexed. The right to apply is determined by NCPR 20, under which those who have a beneficial entitlement under the will (here the nephews) have the best right to apply. The other options were incorrect because; i) the spouse was not a beneficiary of the will and has no right to apply, ii) there were no life or minor interests so only one PR was needed, iii) the chain of representation does not apply in this situation so the brother’s executor has no right to apply.

226
Q

A woman died 6 months ago leaving a valid will that gives the whole of her estate to her brother (B).

B has just entered into a deed of variation of the woman’s will to re-direct a cash sum of £100,000 to the woman’s adult son (S). B made an election under s.142 Inheritance Tax Act 1984 but not under s.62 Taxation of Chargeable Gains Act 1992.

Inheritance tax was payable following the woman’s death and she did not own any exempt assets.

Which of the following is correct?

Select one alternative:

B has made a disposal to S and capital gains tax maybe payable as a result.

B will have to survive 7 years after giving £100,000 to S to avoid any inheritance tax consequences.

HMRC should be notified about the deed of variation.

The woman’s PRs will have to consent to the deed of variation.

For inheritance tax purposes the woman has made a gift to S under her will.

A

For inheritance tax purposes the woman has made a gift to S under her will.

As B made an election under s.142 IHTA the gift of £100,000 from B to S is treated as having been made (for IHT purposes) by the woman’s will. The other options were incorrect because: i) B does not make a PET so does not need to survive 7 years to avoid IHT consequences, ii) although no election is made under s.62 TCGA, cash is exempt so CGT is irrelevant, iii) HMRC do not need to be notified, nor do A’s PRs need to consent, because the variation has no effect on the amount of IHT payable (there are no exempt beneficiaries or exempt assets).

227
Q

Your client and his sister have been jointly appointed as executors of their father’s estate. They have arranged the funeral which is taking place today. Your client has just received a job offer to work in Spain for 12-18 months and wants to know if there is any way that he can let his sister administer the estate without him.

Which of the following is correct advice for your client?

Select one alternative:

He can only renounce probate if his sister agrees.

He can renounce probate because he has not intermeddled with the estate.
Answered and correct

He can renounce probate because he will be leaving the UK.

He cannot renounce probate because if he does there will only be one executor remaining and a minimum of two is required.

He can renounce probate because he will be unavailable for at least 12 months and therefore the administration would not be completed within the “executors’ year”.

A

He can renounce probate because he has not intermeddled with the estate.

A person may only renounce probate if they have not intermeddled. Arranging a funeral is not intermeddling but an act of common humanity so your client can renounce. The other options were incorrect because: i) an executor may continue to act from abroad ii) the consent of a co-executor is not required ii) although being “unavailable” may be the reason why a person wishes to renounce this is not a determining factor in whether they are entitled to and iv) one executor is sufficient.

228
Q

On 14th August 2020, S gives £3,000 to D. On 15th August 2020, S gives £250 to E. On 1 September 2020, S gives £250 to each of E, F and G.

Which of the gifts are chargeable?

Select one alternative:

£500 to E

£3,000 to D and £500 to E

None

£3,000 to D and £250 to E

£250 to E

A

£500 to E
The correct answer is £500 to E. Both gifts to E are chargeable. The annual exemption applies to the gift to D. The small gifts exemption apply to the gifts to F and G.

229
Q

2
Five years ago your client made a will that contained pecuniary legacies to friends and family with the residue of the estate left to their spouse. By the will your client also appointed their spouse to be the sole executor of their estate. Confirmation of your client’s divorce was received yesterday.

Which one of the following statements is correct?

Select one alternative:

Your client’s will is not revoked and all of its terms will take effect.

Your client’s will is not revoked, but the executor appointment clause will fail. The gift of residue and the legacies to friends and family will take effect.

Your client’s will is not revoked, but the gift of residue fails. The executor appointment clause and the legacies to friends and family will take effect.

Your client’s will is revoked and none of its terms will take effect.

Your client’s will is not revoked, but the gift of residue and executor appointment clause both fail. The legacies to friends and family will take effect.

A

Your client’s will is not revoked, but the gift of residue and executor appointment clause both fail. The legacies to friends and family will take effect.

The correct answer is that the client’s will is not revoked, but the gift of residue and executor appointment clause both fail. The legacies to friends and family will take effect. The effect of the divorce is to treat the spouse as having predeceased the testator, meaning the appointment of the spouse as executor is ineffective and any legacies to the spouse will fail.

230
Q

A man has made no previous gifts and wishes to benefit his daughter who is about to be married. He wishes to gift to his daughter as much as possible without a potential liability to Inheritance Tax should he die within seven years.

Ignoring the family maintenance exemption and normal expenditure out of income, what is the maximum amount of cash the man can gift to his daughter without a potential liability to inheritance tax?

Select one alternative:

£325,000

£511,000

£330,000

£333,000

£336,000

A

£336,000

The correct answer is £336,000. The man can use the NRB, marriage exemption and two annual exemptions to make gifts to his daughter without any inheritance tax liability.

231
Q

A man has made no previous gifts and wishes to benefit his daughter who is about to be married. He wishes to gift to his daughter as much as possible without a potential liability to Inheritance Tax should he die within seven years.

Ignoring the family maintenance exemption and normal expenditure out of income, what is the maximum amount of cash the man can gift to his daughter without a potential liability to inheritance tax?

Select one alternative:

£325,000

£511,000

£330,000

£333,000

£336,000

A

£336,000

The correct answer is £336,000. The man can use the NRB, marriage exemption and two annual exemptions to make gifts to his daughter without any inheritance tax liability.

RNRB does not apply to lifetime gifts.

232
Q

A man died recently leaving his estate equally between his children and grandchildren.

Two years ago, the testator’s son (then 21) went travelling and none of the family has seen him since. Despite extensive effort the testator’s other children, who are the man’s personal representatives (‘PRs’), have not been able to trace the son.

Three months ago the PRs placed advertisements complying with s 27 Trustee Act 1925. Neither the son nor anyone claiming to be entitled to a share of the estate has responded to the notice. The PRs propose to distribute the estate and pay only the beneficiaries they know about and can locate.

Which of the following best describes the protection afforded to the PRs as a result of these steps?

Select one alternative:

They will not be protected from claims by unknown beneficiaries, but will be protected from claims by the son.

They will be protected from claims by unknown beneficiaries and the son in their capacity as PR but not in their capacity as estate beneficiaries.

They will not be protected from claims by either unknown beneficiaries or the son.

They will be protected from claims by unknown beneficiaries in their capacity as PR but not in their capacity as estate beneficiaries. They are not protected from a claim by the son.

They will be protected from all claims.

A

They will be protected from claims by unknown beneficiaries in their capacity as PR but not in their capacity as estate beneficiaries. They are not protected from a claim by the son.

The correct answer is that the PRs will be protected from claims by unknown beneficiaries in their capacity as PR but not in their capacity as estate beneficiaries. They are not protected from a claim by the son. S 27 Trustee Act 1925 only provides protection against unknown beneficiaries, not missing beneficiaries. The protection is for the PRs only. If an unknown or missing beneficiary re-emerges, they can make a claim against other estate beneficiaries who have received more than they are entitled to from the estate.

The PRs will be protected against unknown beneficiaries but only in their capacity as PR. They may still be liable to unknown beneficiaries and/or the son if they receive more than they are entitled to from the estate.

S 27 Trustee Act 1925 only provides protection against unknown beneficiaries, not missing beneficiaries. The protection is for the PRs only. If an unknown or missing beneficiary re-emerges, they can make a claim against other estate beneficiaries who have received more than they are entitled to from the estate.

233
Q

A man attends a meeting with his lawyer to discuss making a new will. During the meeting the man produces a photocopy of the will he has made previously and writes “cancelled” on the front page. He then hands his lawyer the copy and says “I want to cancel this will”. The man takes no other action with regards this previous will.

The man subsequently executes a new will which contains the words: “This is the last will and testament of [the man]”.

Assuming there are no other relevant provisions in the new will, which of the following is correct?

Select one alternative:

It is not clear whether the man’s previous will was revoked or not. However, if the client does now have two valid wills, the later will takes priority if there is any discrepancy between the two.

The man did not revoke his previous will because it was not destroyed by him and there are no express words of revocation in the new will. The later will takes priority if there is a discrepancy between the two.

The man revoked his previous will by writing “cancelled” on the photocopy and stating expressly that he wanted to cancel the will. These actions demonstrate the required intention to revoke.

The man revoked his previous will when he executed his new will because the new will contains words which indicate the later will should supersede any earlier will.

The man did not revoke his previous will because he wrote “cancelled” on a copy rather than the original. The earlier will takes priority if there is a discrepancy between the two.

A

The man did not revoke his previous will because it was not destroyed by him and there are no express words of revocation in the new will. The later will takes priority if there is a discrepancy between the two.

The correct answer is that the man did not revoke his previous will because it was not destroyed by him and there are no express words of revocation in the new will. The later will takes priority if there is a discrepancy between the two.

234
Q

You take instructions for a will from a new client just before you leave the office on annual leave for two weeks. Upon your return you prepare a draft will for your client, which they approve saying “it seems fine to me”. You intend to ask the client to come into the office to sign their will but forget to set up an appointment. Three weeks later you arrange for a final version to be posted to your client but when sending the will for signing do not include any instruction about the execution process.

You have just discovered that your client has died unexpectedly. You do not know whether they signed their will before they died.

Which of the following is most accurate with regards your liability?

Select one alternative:

If the will was validly executed but it turns out that you failed to include a gift to one of the beneficiaries in accordance with the client’s instructions, you would not be liable for this mistake as there is no element of dishonesty.

If the will was validly executed but it turns out that you failed to include a gift to one of the beneficiaries in accordance with the client’s instructions, you would not be liable for this mistake because the client (to whom you owe a duty of due diligence) has now died.

If the will was validly executed but it turns out that you failed to include a gift to one of the beneficiaries in accordance with the client’s instructions, you would not be liable for this mistake as the client approved the draft and should have noticed this themselves.

If the will was validly executed but one of the witnesses was a beneficiary, you may be liable for failing to provide adequate instructions to the client.

If the client did not sign their will before their death, although regrettable, this would not give rise to any liability on your behalf.

A

If the will was validly executed but one of the witnesses was a beneficiary, you may be liable for failing to provide adequate instructions to the client.

The correct answer is that If the will was validly executed but one of the witnesses was a beneficiary, you may be liable for failing to provide adequate instructions to the client. A solicitor has a duty to prepare a will in accordance with their instructions and to properly advise the client on execution. They may be liable after the client’s death to an estate beneficiary who suffers loss as a result.


If the will was validly executed but it turns out that you failed to include a gift to one of the beneficiaries in accordance with the client’s instructions, you would not be liable for this mistake as there is no element of dishonesty.
- A solicitor is responsible for drafting a will which gives effect to their client’s intentions. Even if the mistake was honest, the solicitor may be liable for negligence. After the testator’s death, the solicitor may be liable to the intended beneficiary.

If the will was validly executed but it turns out that you failed to include a gift to one of the beneficiaries in accordance with the client’s instructions, you would not be liable for this mistake because the client (to whom you owe a duty of due diligence) has now died.
- A solicitor is responsible for drafting a will which gives effect to their client’s intentions. After the testator’s death, the solicitor may be liable to the intended beneficiary.

If the will was validly executed but it turns out that you failed to include a gift to one of the beneficiaries in accordance with the client’s instructions, you would not be liable for this mistake as the client approved the draft and should have noticed this themselves.
- A solicitor is responsible for drafting a will which gives effect to their client’s intentions. After the testator’s death, the solicitor may be liable to the intended beneficiary.

If the client did not sign their will before their death, although regrettable, this would not give rise to any liability on your behalf.
- It is possible for the solicitor to be liable to an estate beneficiary if they did not properly advise their client and this has resulted in a loss to an estate beneficiary.

235
Q

Four tax years ago, A made chargeable transfers that used up A’s entire nil rate band. Three tax years ago, A gave £4,000 to B. Later in the same tax year, A gave £8,000 to C.

A has just died. How much of the gift to C is chargeable to inheritance tax?

Select one alternative:

£5,000

£8,000

£2,000

£0

£1,000

A

£8,000
The correct answer is £8,000. The full amount is chargeable to tax. The NRB was used in the previous year and the annual exemption was used on the gift to B.

£1,000
Read the question carefully. This is the amount chargeable on the gift to B, not C.

236
Q

A valid will contains the following clause: “I give £40,000 to my sister”.

The amount of “£40,000” has been crossed through by hand, although the original figures remain visible.

Which of the following best describes the effect of the alteration to the will?

Select one alternative:

The sister may receive £40,000. The alteration will not be effective if there is another copy of the will showing the original gift unamended.

The sister will not receive anything. The alteration is effective because it is presumed to have to have been made before the will was executed.

The sister will not receive anything. The alteration is effective because it is clear the testator intended to delete the gift.

The sister may receive £40,000. The alteration will be effective as long as it can be proved that it was made by the testator, not a third party.

The sister will receive £40,000. The alteration is not effective as the original amount can be seen by natural means.

A

The sister will receive £40,000. The alteration is not effective as the original amount can be seen by natural means.

The correct answer is that the alteration is not effective as the original amount can be seen by natural means. It is presumed that that alteration was made after execution, and it has not been attested, rendering it invalid.

237
Q

On 1 October 2020, A makes gifts to B using the annual exemptions. On 2 October 2020 A makes a further gift to B of £455,000. A dies within 3 years of this gift.

What is the amount of tax payable on the gift as a result of A’s death?

Select one alternative:

£52,000

£26,000

£0

£182,000

£41,600

A

£52,000

The correct answer is £52,000. The gift to B is a PET. As A died within seven years of making the PET it becomes chargeable. A has a full NRB of £325,000 available. The remaining £130,000 is taxed at the death rate of 40%. No taper relief is available.

Don’t forget the NRB.

238
Q

A man prepares his will and signs it in the presence of a witness. A second witness then arrives and the man informally acknowledges his signature while asking the second witness to sign the will. The first witness then moves to the other side of the room as the second witness signs, although she does ask whether the will would be invalid. The will contains an attestation clause.

Which of the following statements best describes the position regarding the validity of the will?

Select one alternative:

The will has not been validly executed as the second witness was not present when the testator signed the will.

The will has not been validly executed as the parties are not all present during the execution at the same time.

The will has been validly executed although the Probate Registry would ask for an affidavit of plight and condition of the will.

The will has been validly executed and the attestation clause raises a presumption of due execution.

The will has not been validly executed as the testator should have signed the will in the presence of the witnesses.

A

The will has been validly executed and the attestation clause raises a presumption of due execution.

The correct answer is that the will has been validly executed and the attestation clause raises a presumption of due execution. The requirements of s 9 Wills Act 1837 have been complied with. The attestation clause raises the presumption of due execution.

The second witness did not need to be present when the testator signed the will. It is sufficient that the testator acknowledged his signature.

It is not necessary for all parties to be present at the same time.

The requirements of s 9 Wills Act 1837 have been complied with. The attestation clause raises the presumption of due execution.

239
Q

A testator executes a will containing a clause giving his ring to his nephew. After the will has been signed, the testator puts his pen down on the will and some ink leaks out onto the document and obliterates the gift of the ring.

Which of the following statements best describes the validity of the gift?

Select one alternative:

The nephew will not receive the ring as the gift has been revoked by the obliteration.

The nephew will only be entitled to the ring if he can submit an affidavit of due execution of the will.

The nephew will not be entitled to the ring as the alteration was made after the will was executed.

The nephew will be entitled to the ring if extrinsic evidence can be obtained setting out the original clause in the will.

The nephew will not receive the ring as the obliteration should have been initialled by the testator and the witnesses.

A

The nephew will be entitled to the ring if extrinsic evidence can be obtained setting out the original clause in the will.

The correct answer is that the nephew will be entitled to the ring if extrinsic evidence can be obtained setting out the original clause in the will. This is an obliteration without intention to revoke.

Note that this is an obliteration without intention to revoke. There is no issue with execution of the will. The problem is that there needs to be evidence of the original clause.

240
Q

You have recently prepared a will for a client who is ill in hospital and now wants to execute their will. You are confident your client has testamentary capacity, however, they are physically unable to hold a pen and cannot sign the will in person.

Which of the following is the best advice?

Select one alternative:

The client may direct someone to sign the will on their behalf in accordance with s 9 Wills Act 1837. However, unless the attestation clause is amended to demonstrate the correct process was followed the will would be invalid.

The client may direct someone to sign the will on their behalf in accordance with s 9 Wills Act 1837. Provided the attestation clause is amended to demonstrate the correct process was followed, the client’s knowledge and approval of the will can be presumed.

To comply with s 9 Wills Act 1837 the client must sign their own will and this can be satisfied by the client making any ‘mark’, even if it is not their usual signature.

To comply with s 9 Wills Act 1837 the client must sign their own will with their usual signature.

The client may direct someone to sign the will on their behalf in accordance with s 9 Wills Act 1837. However, the attestation clause cannot be used as evidence of the client’s knowledge and approval unless an affidavit of due execution is signed by the witnesses.

A

The client may direct someone to sign the will on their behalf in accordance with s 9 Wills Act 1837. Provided the attestation clause is amended to demonstrate the correct process was followed, the client’s knowledge and approval of the will can be presumed.

The correct answer is that the client may direct someone to sign the will on their behalf in accordance with s 9 Wills Act 1837. Provided the attestation clause is amended to demonstrate the correct process was followed, the client’s knowledge and approval of the will can be presumed.

Amending the attestation clause raises a presumption of knowledge and approval. In the absence of such a clause, an affadavit of knowledge and approval would be needed when submitting the will to probate.

A client who cannot physically execute a will may direct someone to sign it on their behalf but it is necessary to amend the attestation clause to show that this procedure was followed.

Although any mark may constitute a signature, this is not the preferable approach and is inappropriate in a case where a client cannot hold a pen. The client may direct someone to sign the will on their behalf in accordance with s 9 Wills Act 1837. Provided the attestation clause is amended to demonstrate the correct process was followed, the client’s knowledge and approval of the will can be presumed.

If the attestation clause is amended appropriately, the client’s knowledge and approval of the will can be presumed.

241
Q

F died in 2002 when the nil rate band was £250,000. He had used only half of his nil rate band. His wife J died in 2020, when the nil rate band was £325,000. She had not made any lifetime chargeable transfers.

Ignoring any potential residential nil rate band, how much nil rate band is available to J’s death estate?

Select one alternative:

£650,000

£450,000

£575,000

£487,500

£325,000

A

£487,500

The correct answer is £487,500. J has 100% of her own NRB which is increased by the unused percentage of F’s NRB. This gives J 150% of £325,000.

J has 100% of her own NRB which is increased by the unused percentage of F’s NRB. This gives J 150% of £325,000.

242
Q

A man executed a will five years ago. One of the gifts in the will leaves the man’s brother a cash legacy of £1,000. The man’s brother was a witness to the will.

Last month the man executed a codicil to the will which changed the amount of the cash legacy to his brother to £500. Two of the man’s work colleagues witnessed the codicil. Neither of them is a beneficiary of the will or codicil.

Which of the following statements provides the best advice to the man?

Select one alternative:

The codicil is treated as having been executed on same date as the will. The brother will receive nothing as he was a witness to the will.

The effect of the codicil is to re-publish the will at the date that the codicil was executed. The brother will receive £500.

The codicil is treated as having been executed on same date as the will. The brother will receive £1,000.

The codicil is treated as having been executed on same date as the will. The brother will receive £500.

The effect of the codicil is to re-publish the will at the date that the codicil was executed. The brother will receive £1,500.

A

The effect of the codicil is to re-publish the will at the date that the codicil was executed. The brother will receive £500.

The correct answer is that the effect of the codicil is to re-publish the will at the date that the codicil was executed. The brother will receive £500. The amendment is effective and £500 will pass to the brother because the brother was not a witness to the codicil.

243
Q

A man died last month testate leaving his entire estate to a woman he was living with. The adult son of the man wishes to challenge the validity of the will on the basis of undue influence. The man’s son maintains that the woman encouraged the man to leave everything to her, to the exclusion of himself.

Which of the following statements best describes the advice that you would give the son?

Select one alternative:

There is a presumption of undue influence in relation to testamentary dispositions. The burden of proof lies with the woman. What the woman did is of itself likely to lead to a successful claim for undue influence.

There is no presumption of undue influence in relation to testamentary dispositions. The burden of proof lies with the son. What the woman did is not of itself likely to lead to a successful claim for undue influence.

There is no presumption of undue influence in relation to testamentary dispositions. The burden of proof lies with the woman. What the woman did is not of itself likely to lead to a successful claim for undue influence.

There is no presumption of undue influence in relation to testamentary dispositions. The burden of proof lies with the son. What the woman did is of itself likely to lead to a successful claim for undue influence.

There is a presumption of undue influence in relation to testamentary dispositions. The burden of proof lies with the son. What the woman did is not of itself likely to lead to a successful claim for undue influence.

A

There is no presumption of undue influence in relation to testamentary dispositions. The burden of proof lies with the son. What the woman did is not of itself likely to lead to a successful claim for undue influence.

The correct answer is that There is no presumption of undue influence in relation to testamentary dispositions. The burden of proof lies with the son. What the woman did is not of itself likely to lead to a successful claim for undue influence. Encouraging is not the same as coercing.

244
Q

A man died on 1 May 2020. He was a bachelor and left his entire estate to his nephew. He did not make any lifetime gifts. His estate comprised a house free of mortgage worth £670,000 and bank and building society accounts with balances amounting to £225,000. The man’s debts and funeral expenses amounted to £10,000.

How much inheritance tax will be payable on the man’s estate?

Select one alternative:

£84,000

£154,000

£354,000

£224,000

£151,600

A

£224,000

The correct answer is £224,000. The man’s assets amount to £895,000. Deduct the £10,000 debts and funeral expenses to reach £885,000. The man had a full NRB of £325,000 available. The remaining £560,000 is taxed at the death rate of 40%.

RNRB is not available. Nor is the annual exemption.

245
Q

Under the terms of an unmarried testator’s will the whole of their estate is given to their only child. When the testator was clearing out their office their will was accidentally shredded.

Which of the following most accurately explains whether the testator’s will is still valid?

Select one alternative:

The will is still valid because the testator’s child is the sole beneficiary of the will and would inherit under intestacy in any event.

The will is still valid because although the will was destroyed there was no intention to destroy it.

The will is still valid because the doctrine of dependent relative revocation will apply.

The will is not valid because the will must be read as a whole and this is no longer possible.

The will is still valid because a will must be revoked by a revocation clause in a new will that is executed in accordance with s.9 Wills Act 1837.

A

The will is still valid because although the will was destroyed there was no intention to destroy it.

The correct answer is that the will is still valid because although it was destroyed there was no intention to do so. Destruction must be accompanied by an intention to destroy

The intestacy rules are separate from the will. In any case, the will has not been revoked because the destruction was accidental.

Destruction is not sufficient to revoke a will, unless accompanied by an intention to revoke.

This doctrine is not relevant. There has been no attempt at substituting a legacy. The will has been destroyed. It is possible to revoke a will by destruction but that has not occurred here because the destruction was accidental.

This rule applies to partial destruction. The whole will has been destroyed here but this will not revoke the will because the destruction was accidental.

246
Q

Your client wants to make a will leaving the whole of their estate equally between their two children.

Your client is 93 and living in a hospice where they will require medical care, including the administration of strong pain killers, for the remainder of their life. Your client tells you they feel fine when you visit to take instructions but the client’s daughter tells you that this morning your client did not recognise her.

Which of the following best describes the steps you should follow?

Select one alternative:

There are grounds to doubt your client’s testamentary capacity and therefore a medical professional should be instructed to confirm whether the test in Banks v Goodfellow is satisfied.
Answered and correct

There are grounds to doubt your client’s testatmentary capacity and therefore a medical professional should be instructed to confirm whether the test in Parker v Felgate is satisfied.

As your client’s instructions are rational and your client has expressly confirmed to you they are fine, no formal assessment of capacity is required.

There are grounds to doubt your client’s mental capacity and therefore a medical professional should be instructed to confirm whether the test in the Mental Capacity Act 2005 is satisfied.

As your client’s instructions are rational, no formal assessment of capacity is required provided a detailed file note is made.

A

There are grounds to doubt your client’s testamentary capacity and therefore a medical professional should be instructed to confirm whether the test in Banks v Goodfellow is satisfied.

The correct answer is that there are grounds to doubt your client’s testatmentary capacity and therefore a medical professional should be instructed to confirm whether the test in Banks v Goodfellow is satisfied.

The facts suggest the potential for a lack of capacity given the age, medication and indication they didn’t recognize the daughter. Medical advice should be sought and the correct test is Banks v Goodfellow.

247
Q

A testator leaves a legacy of “my Bernstein piano and my Boots plc shares” in his validly executed will to his nephew. At the time the will is executed, he has 50 Boots Plc shares. After the will is executed, the piano is sold and he purchases a Steinway piano with the sale proceeds and has cash left over of £5,000.

At the date of his death, the testator owns the Steinway piano and 100 Boots plc shares. What legacy does the nephew receive?

Select one alternative:

100 Boots plc shares only.

The Steinway piano and 100 Boots plc shares.

£5000 and 100 Boots plc shares.

The Bernstein piano and 50 Boots plc shares.

The Steinway piano and 50 Boots plc shares.

A

100 Boots plc shares only.

The correct answer is 100 Boots plc shares only. A gift of “my Boots shares” will be taken to mean the number of shares owned at the date of death. The gift of the piano adeems and cannot be replaced by the piano which was purchased later.

A gift of “my Boots shares” will be taken to mean the number of shares owned at the date of death. The gift of the piano adeems and cannot be replaced by the piano which was purchased later.

248
Q

20
A testator’s will leaves legacies of £2,000 to his son and £1,000 to his brother. The son has predeceased the testator leaving two children and the brother has predeceased leaving a son. The will contained no provisions regarding substitutions.

Which of the following is correct?

Select one alternative:

The testator’s grandchildren each receive £1,000 and the nephew £1,000.

The testator’s grandchildren each receive £1,000 and the brother’s legacy of £1,000 passes into the testator’s residuary estate.

Both legacies are distributed under the testator’s residuary estate.

£2,000 passes into the testator’s residuary estate and the nephew receives £1,000.

The testator’s grandchildren each receive £1,500.

A

The testator’s grandchildren each receive £1,000 and the brother’s legacy of £1,000 passes into the testator’s residuary estate.

The correct answer is that the son’s children each receive £1,000 and the brother’s legacy of £1,000 passes into the testator’s residuary estate. S 33 wills Act 1837 applies so that the issue of the son take the share their parent would have taken. S 33 does not apply to the nephew.

S 33 wills Act 1837 applies so that the issue of the son take the share their parent would have taken. S 33 does not apply to the brother’s legacy.

S 33 wills Act 1837 applies so that the issue of the son take the share their parent would have taken. S 33 does not apply to the brother’s legacy but this does not mean that his share is distributed to the grandchildren.

249
Q

A man and his wife visit their solicitor to make wills. The solicitor draws up identical wills for the couple and the couple have the same ultimate beneficiaries. The wills contain no statement in the wills as to whether the couple intend their wills to be mutual wills.

Which of the following statements best describes how the wills are likely to be treated?

Select one alternative:

The wills are likely to be regarded as mutual wills as they are identical and have the same ultimate beneficiaries.

The wills are likely to be regarded as mutual wills as the couple give joint instructions to the solicitor.

The wills are likely to be regarded as mirror wills in the absence of any evidence that the couple intended the wills to be mutual.

The fact that there is no statement in the will on this point means that the wills must be mirror wills.

The wills are likely to be regarded as mutual wills as this is the more common arrangement.

A

The wills are likely to be regarded as mirror wills in the absence of any evidence that the couple intended the wills to be mutual.

This was a question on Wills and Estates. This question tested your knowledge and understanding of mirror and mutual wills. Identical wills are likely to be treated as mirror wills in the absence of any evidence that there are intended to be mutual wills. Mutual wills are not very common and strong evidence is required that this was the intention because they effectively prevent a testator from amending their will.

250
Q

A deceased man’s validly executed will appoints his wife as his sole executor and leaves his house to her, with a substitution provision in favour of the man’s son. The residue of the estate is left to the man’s brother.

The man divorced his wife several months before his death but did not manage to amend his will before he died.

Which of the following statements best describes how the estate will be administered?

Select one alternative:

The wife can act as executor. The house will pass to the son.

The wife cannot act as executor. The house will pass to the son.

The wife cannot act as executor but will inherit the house.

The wife can act as executor and will inherit the house.

The wife cannot act as executor. The house will pass to the brother.

A

The wife cannot act as executor. The house will pass to the son.

This is a question on Wills and Estates. This question tested your knowledge and understanding of the effect of divorce on the provisions of a will. As the man and his wife were divorced, the wife is treated as having predeceased the man for all purposes but the rest of the will remains valid. The wife therefore cannot act as executor and is not entitled to inherit the house. The substitution provision in favour of the man’s son applies.

251
Q

A woman accidentally destroys the original version of her will by putting it through a shredder along with some other paperwork. The woman’s solicitor has a copy of the will.

Is the woman’s will still valid?

Select one alternative:

Yes, because a will can only be revoked by writing “revoked” on it.

Yes, because a will can only be revoked by an express statement in a later will.

No, because only an original will can be submitted to probate.

No, because the will was revoked by a physical act of destruction.

Yes, because she did not intend to revoke it.

A

Yes, because she did not intend to revoke it.

This was a question on Wills and Estates. This question tested your knowledge and understanding of the rules relating to revocation of wills. The woman’s will remains valid. A will can be revoked by a physical act of destruction but only if accompanied by an intention to revoke it. The woman has physically destroyed the original will but did so accidentally. As the solicitor has a copy of the will, this can be submitted to probate as evidence of the original will.

252
Q

A woman died recently leaving her entire estate to charity in her will. She left nothing to her ex-husband as they divorced several years ago and he has remarried. The woman has an adult daughter with whom she lost contact many years ago. The woman had been cohabiting with a new partner for 18 months before her death but the partner was not financially dependent on the woman.

Who is entitled to bring a claim under the Inheritance (Provision for Family and Dependants) Act 1975?

Select one alternative:

The husband and the daughter and the partner.

The husband and the partner.

The husband and the daughter.

The partner and the daughter.

The daughter only.

A

The daughter only.

This was a question on Wills and Estates. This question tested your knowledge and understanding of eligibility to bring claims under the Inheritance (Provision for Family and Dependants) Act 1975. The woman’s daughter is the only person entitled to bring a claim on the facts of the question as she is a child of the deceased. The partner is not entitled to claim as they have not cohabited for more than two years and the partner is not financially dependent on the woman. The ex-husband is not eligible to claim because he has remarried.

253
Q

A man has just died. His will leaves the residue of his estate to his daughter but contains the following specific legacies:

“my Jaguar car to my cousin”. This legacy is included in the original will, which was executed four years ago. Three years ago, the man replaced his car with a newer model Jaguar.
“my stamp collection to my neighbour.” This legacy is included in a codicil executed two years ago. Last week, the man was given a very rare Penny Black stamp to add to his stamp collection.
Which of the following statements best describes how his estate will be distributed?

Select one alternative:

The car will go to the daughter but the Penny Black stamp will go to the neighbour.

Both the car and the Penny Black stamp will go to the neighbour.

The car will go to the cousin but the Penny Black stamp will go to the daughter.

The car will go to the cousin and the Penny Black stamp will go to the neighbour.
Answered and correct

Both the car and the Penny Black stamp will go to the daughter.

A

The car will go to the cousin and the Penny Black stamp will go to the neighbour.

This was a question on Wills and Estates. This question tested your knowledge and understanding of the interpretation of specific gifts in wills. The legacy relating to the Jaguar is treated as referring to the Jaguar owned by the man at the date the will was executed. However, the execution of the codicil means that the will is treated as having been republished, meaning that the reference is now to the Jaguar owned at the date of the codicil. The provision is therefore effective and the Jaguar passes to the man’s cousin. Collections are treated differently under s24 Wills Act 1837 as they are capable of growing. The reference to the man’s stamp collection is therefore to the collection owned at death, meaning that the Penny Black stamp passes to the neighbour along with the rest of the collection.

254
Q

A man dies leaving a will which appoints his son as his sole executor. The will leaves the entire estate to the man’s son. The will does not contain any substitutionary provisions. The man’s son and the man’s father witness the will.

Which of the following statements best explains the position with regards to the administration of the estate?

Select one alternative:

The will is valid but the estate will pass under the intestacy rules. The son can act as the executor of the estate.

The will is valid and the estate will pass to the son under the will. The son cannot act as the executor of the estate. The father can apply to act as the administrator of the estate.

The will is valid and the estate will pass to the son under the will. The son can act as the executor of the estate.

The will is invalid and the estate will pass under the intestacy rules. Neither the son nor the father can apply to act as the administrator of the estate.

The will is invalid and the estate will pass under the intestacy rules. The son cannot act as the administrator of the estate. The father can apply to act as the administrator of the estate.

A

The will is valid but the estate will pass under the intestacy rules. The son can act as the executor of the estate.

This was a question on Wills and Estates. This question tested your knowledge and understanding of the provisions of s15 Wills Act 1837. This provision provides that gifts to an attesting witness of a will are void. As the entire estate has been left to the son under the will, with no substitutionary provisions, the entire estate now passes under the intestacy rules instead. Although the gift to the man’s son is void, the will itself remains valid, meaning that the provisions relating to the appointment of executors are still effective. There is no prohibition on a witness acting as executor. The man’s son is therefore able to act as executor of the estate.

255
Q

Three tax years ago a woman gave £6,000 to her son. Two tax years ago she gave £500,000 to the trustees of a discretionary trust. The woman has made no other lifetime transfers.

How much inheritance tax was payable on the payment into the discretionary trust?

Select one alternative:

£33,800

£0

£70,000

£34,400

£35,000

A

£33,800

This is a Wills and Estates question. This question tested your knowledge and understanding of inheritance tax on lifetime gifts. The gift to the woman’s son was a PET which had not failed by the time she set up the discretionary trust and so no annual exemptions had been used. The woman had a cumulative total of zero at the date of the payment into the discretionary trust. The payment into the discretionary trust was a lifetime chargeable transfer. This transfer benefitted from two annual exemptions (the current year and the previous year), reducing the taxable value to £494,000. The woman had a full NRB available to use against this transfer, resulting in a chargeable amount of £169,000 (£494,000 - £325,000 = £169,000). It was taxable at the lifetime rate of 20%, meaning inheritance tax of £33,800 was payable.

256
Q

At the date of his death, a man owns personal chattels (£1,000), a house (£300,000) held in his sole name and a holiday home held as joint tenants with his daughter (£100,000). The man is the remainder beneficiary of a trust fund (capital value £100,000) of which the life tenant is still alive. The man’s estate is entitled to receive £100,000 from a life insurance policy. The man’s children are entitled to a lump sum of £200,000 directly from his pension trustees.

What is the gross value of the man’s taxable estate for inheritance tax purposes?

Select one alternative:

£801,000

£551,000

£451,000

£501,000

£651,000

A

£451,000

This is a Wills and Estates question. This question tested your knowledge and understanding of the taxable estate for inheritance tax purposes. The personal chattels (£1,000), house (£300,000), half the holiday home (£50,000) and life insurance proceeds (£100,000) are all included in the taxable estate. Although the holiday home was held as joint tenants (and would therefore not form part of the succession estate) there is a deemed severance at the date of death for inheritance tax purposes, giving the man a half share. The life insurance proceeds are included because they are payable directly to the estate. The pension lump sum is not included in the taxable estate because it is payable directly to the children. The remainder interest in the life interest trust is also not included in the man’s taxable estate.


Insurance policies must be looked at carefully to see whether the proceeds form part of the succession estate.

If the deceased had a simple life insurance policy, the proceeds of the policy pass to the succession estate.

If the benefit of the policy waswritten in trustfor another person, the proceeds will not form part of the succession estate. The insured has no beneficial interest under the policy. The proceeds belong to the beneficiaries nominated in the policy andveston the insured’s death.

257
Q

A junior solicitor who works in the private client department of a law firm holds the belief that same sex marriages should not be legal. This belief is fundamental to the solicitor’s religion. The solicitor has been asked by their supervisor to attend on a same sex couple to advise them on their wills, including the implications if they marry. The solicitor has not previously acted for a same sex couple but is aware that the legal and practical considerations are virtually identical to those for a mixed sex couple.

Which of the following best describes the solicitor’s ability to refuse to advise the clients?

Select one alternative:

The solicitor can refuse to advise the clients because a solicitor has a duty to act in the best interests of each client.

The solicitor cannot refuse to advise the clients because a solicitor has a duty not to unfairly discriminate.

The solicitor can refuse to advise the clients because a solicitor must be competent to carry out their role

The solicitor can refuse to advise the clients because the solicitor’s firm must not unfairly discriminate against the solicitor.

The solicitor cannot refuse to advise the clients because a solicitor has a duty to act in the best interests of each client.

A

The solicitor cannot refuse to advise the clients because a solicitor has a duty not to unfairly discriminate.

This is a professional conduct question in the context of wills and estates. This question requires the student to apply the solicitor’s duty not to discriminate under CCS 1.1, their duty to provide a competent service (CCS 3.2) and to maintain their competence and keep their skills up to date (CCS 3.3). These were covered in the Wills induction session and the element ‘Client care – service and competence’.

258
Q

A solicitor acts for a client on the administration of the estate of their relative. The solicitor agrees with the client to send a particular letter as soon as possible. The letter falls behind a tray in the post room and is not posted until a week later. This makes no difference to the administration of the estate and is not detrimental to the client. A response to the letter was received four days after posting. At a subsequent meeting the client asks the solicitor when the letter was sent.

Which of the following statements best describes how the solicitor should respond?

Select one alternative:

The solicitor should tell the client not to worry about the posting date, as the letter was received, and the client needs to consider the response to the letter.

The solicitor should tell the client when the response to the letter was received as that is the important next step in the matter.

The solicitor should tell the client not to worry about the date the letter was posted as it does not matter as it has not affected the client’s matter.

The solicitor should tell the client the letter was posted on the day it was put in the post tray as that is when the solicitor intended it to be posted.

The solicitor should provide the date on which the letter was posted, explaining the issue with the post tray and steps taken if necessary.

A

The solicitor should provide the date on which the letter was posted, explaining the issue with the post tray and steps taken if necessary.

This is a WAE question. This is a professional conduct question in the context of wills and estates. This question required students to recognise the obligation not to mislead clients (paragraph 1.4 Code of Conduct for Solicitors (‘CCS’)) and to be open and honest when things go wrong (CCS 7.11). The obligation of trust and confidence is covered in the element ‘maintaining trust and acting fairly’.

259
Q

A woman died recently leaving an estate comprising her house (£500,000) and other assets worth £800,000 to her children. The woman’s husband died 10 years earlier leaving his whole estate to the woman. Both the woman and her husband had full nil rate bands available at the time of their respective deaths.

How much inheritance tax is payable on the woman’s estate?

Select one alternative:

£120,000

£380,000

£510,000

£260,000

£190,000

A

£120,000

This is a WAE question. This tests knowledge and application of the NRB, TNRB, RNRB and TRNRB. The total gross value of the woman’s estate is £1,300,000. Her estate has NRB of £325,000 and RNRB of £175,000. It also has the TNRB of £325,000 and TRNRB of £175,000 from the husband’s estate. The total NRB and RNRB are therefore £1,000,000 leaving £300,000 taxed at 40 % = £120,000.

An individual’s surviving spouse or civil partner can inherit the unused portion of their RNRB similarly to with the basic NRB.

260
Q

A deceased man’s estate consists of personal chattels (£25,000), cash (£75,000), a villa in Spain (£90,000) and a life interest in a trust (capital value £110,000). The deceased had made chargeable transfers in the past seven years totalling £18,000. The deceased had never married and left his entire estate to his two children.

Which of the following correctly describes the status of the estate?

Select one alternative:

A non-excepted estate.

A low value excepted estate because the gross value of the estate is less than £650,000.

A low value excepted estate because the gross value of the estate is less than £325,000.

An exempt excepted estate because the net value of the estate is less than £325,000.

An exempt excepted estate because the net value of the estate is less than £650,000.

A

A low value excepted estate because the gross value of the estate is less than £325,000.

This is a WAE question. This tests the category of estate, whether it is low value/exempt excepted estate or non-excepted estate. The total gross value of the estate is £318,000. This is an excepted estate as no more than £150,000 was immediately before death settled property, not more than £100,000 was situate outside the UK and the lifetime transfers did not exceed £150,000. No IHT is payable because the gross value of the estate is below the NRB threshold of £325,000. The estate therefore falls within the category of low value excepted estate.

261
Q

A woman died a month ago. She had made a valid will that appointed her spouse and her brother as the executors. The woman left the whole of her estate equally between her daughters (aged 20 and 19) and her son (aged 16).

The woman’s brother has arranged the funeral but has not otherwise been involved in the administration of her estate. He is now planning to renounce probate.

The woman’s spouse died before her but she was survived by her parents and her elder sister.

Which of the following correctly states who is entitled to apply to administer the woman’s estate?

Select one alternative:

The woman’s parents.

The woman’s sister.

The woman’s daughters.

The woman’s daughters with power reserved to her brother.

The woman’s daughters with power reserved to her son.

A

The woman’s daughters.

This is a WAE question. This tests the entitlement to a grant. The woman’s will does not have a willing or able executor as her spouse as died and her brother wants to renounce. The brother is entitled to renounce as he has not intermeddled in the estate. NCPR20 applies and the persons with the greatest entitlement can apply. Her children have the greatest entitlement, although her son cannot apply as he is not 18. The daughters can apply, and both are needed because of the son’s minor interest.

262
Q

A man dies leaving an estate comprising his home (£300,000), bank accounts (£60,000), a remainder interest in a trust of which the life tenant remains alive (£40,000), personal possessions (£30,000) and shares in a private company which he acquired three years before he died (£25,000).

The man left the whole of his estate to his sister. He had made no lifetime gifts and had never married or entered a civil partnership. He had no debts.

How much inheritance tax is payable following the man’s death?

Select one alternative:

£26,000

£36,000

£42,000

£52,000

£0

A

£26,000

This is a WAE question. This tests the calculation of Inheritance Tax on a death estate. The gross value of the taxable estate is £390,000 (£300,000 + £60,000 + £30,000). The remainder interest in the trust is excluded property. The shares in a private company qualify for BPR as he had owned them for more than 2 years. Allowing for the nil rate band of £325,000, the balance of £65,000 is taxed at 40 % = £26,000.

263
Q

A man dies intestate and is survived by his father, his spouse and his two children. His estate amounts to £680,000 cash, plus personal chattels worth £20,000.

Which of the following correctly states the value of the spouse’s entitlement if she survives the man by 30 days?

Select one alternative:

£495,000
Answered and correct

£485,000

£290,000

£505,000

£270,000

A

£495,000

This is a WAE question. This tests the application of the intestacy rules where there is a surviving spouse and issue. The spouse is entitled to the personal chattels (£20,000), statutory legacy of £270,000 and half the residue (£700,000- £290,000 = £410,000/2 = £205,000). The total entitlement is therefore £495,000.

264
Q

A man dies intestate, survived by his spouse and two adult children. The man’s estate is worth £2,000,000. Amongst the man’s possessions are a piano, a holiday home (worth £500,000) held solely in the man’s name and a computer used solely for business.

To which of the assets is the spouse entitled under the intestacy rules?

Select one alternative:

The holiday home only.

The piano and computer.

The computer only.

The piano, computer and holiday home.

The piano only.

A

The piano only.

This is a WAE question. This tests knowledge and understanding of the spouse’s entitlement to personal chattels under the intestacy rules. The definition is in section 55(1)(x) AEA 1925. The definition includes tangible movable property and therefore includes the piano but not the holiday home. The computer is excluded as it was used solely or mainly for business purposes.

265
Q

A man died intestate. He had never married or entered into a civil partnership and did not have any children.

The man had a sister and a brother. The man and his sister were the children of their father’s first marriage. Their brother was the child of their father’s second marriage.

The man’s parents and sister predeceased him. The man’s sister was survived by her two minor children. The man is survived by his brother, who has one minor son.

Who will inherit the man’s estate?

Select one alternative:

The brother and his son on the statutory trusts.

The brother only.

The brother’s son on the statutory trusts.

The sister’s children on the statutory trusts.
Answered and correct

The sister’s children and the brother on the statutory trusts.

A

The sister’s children on the statutory trusts.

This is a WAE question. This tests the application of the intestacy rules. As the intestate was not survived by a spouse or issue, the order of entitlement is set out in section 46 AEA 1925. The parents had predeceased. The sister had predeceased leaving two children. The sister’s children inherit on the statutory trusts. The brother (or his son) do not inherit as he is a sibling of the half blood (shares one parent), unlike the sister, who is a sibling of the whole blood (shares both parents).

266
Q

A woman dies intestate. Immediately before her death, the woman owned assets worth £600,000, including personal chattels (£10,000) and a house (valued at £400,000) held as joint tenants with her mother. The woman is survived by her husband and two children, aged six and four.

Which of the following best describes how the woman’s assets will pass under the intestacy rules?

Select one alternative:

The husband is entitled to assets worth £105,000. £95,000 is held on the statutory trusts for the children

The husband is entitled to assets worth £440,000. £160,000 is held on the statutory trusts for the children.

The husband is entitled to assets worth £435,000. £165,000 is held on the statutory trusts for the children.

The husband is entitled to assets worth £100,000. £100,000 is held on the statutory trusts for the children.

The husband is entitled to assets worth £200,000. The children receive nothing.

A

The husband is entitled to assets worth £200,000. The children receive nothing.

This is a WAE question. This tests the application of the intestacy rules. The right of survivorship applies to the house held as joint tenants. As there is a spouse and issue, the spouse is entitled to the personal chattels, a statutory legacy of £270,000 and half the residue. The spouse is therefore entitled to the remaining £200,000 (including the chattels) after the joint tenancy asset. The children receive nothing.

267
Q

A woman creates a trust over £50,000. Her wife has a life interest and their daughter (17) has a remainder interest. The woman’s daughter wants to use some of the £50,000 to fund a university course.

What powers do the trustees have in respect of this request?

Select one alternative:

The trustees may not advance any of the capital because the daughter has a contingent interest

The trustees may not advance any of the capital because the daughter has a remainder interest which has not yet vested in possession

The trustees may not advance any of the capital because the daughter is under 18

The trustees may advance some of the capital for the requested purpose if the woman consents

The trustees may advance some of the capital for the requested purpose if the wife consents

A

The trustees may advance some of the capital for the requested purpose if the wife consents

268
Q

A firm is acting in the administration of a woman’s estate. By her valid will the deceased appointed the firm and her sister to act as the executors.

The deceased left a cash legacy of £10,000 to her brother and the residue of her estate to her adult son.

The executors are ready to complete the administration and the firm has prepared the estate accounts.

Which of the following is the most accurate statement regarding the estate accounts?

As the estate accounts have been prepared by the firm (a professional executor) they do not need to be approved by anyone else.

Only the sister (as co-executor) will approve the estate accounts. None of the beneficiaries will be involved.

The estate accounts must be prepared in a specific form.

The sister, son and brother should approve the estate accounts.

The sister and the son should approve the estate accounts.

A

The sister and the son should approve the estate accounts.

Correct. Executors (whether lay or professional) should prepare the estate accounts. There is no specific form required. Only residuary beneficiaries would usually sign these and in doing so indicate their agreement /approval.

269
Q

Your firm is acting in the administration of an estate. The deceased’s will appointed the firm and the deceased’s son to act as the executors. The deceased’s will contained no express administrative provisions.

The deceased’s will left £50,000 to her mother and the residue of the estate to her adult granddaughter.

The firm would like to issue an invoice to the estate for the legal work it has carried out during the administration.

Which of the following most accurately summarises the right the firm has to charge the estate for their legal services?

The firm cannot charge the estate without consent from the deceased’s mother and granddaughter.

The firm cannot charge the estate without consent from the deceased’s son.

The firm may charge the estate without the need for any consent.

The firm may charge the estate without the need for any consent but only if the amount is reasonable.

The firm cannot charge the estate as this would be a breach of their fiduciary duties.

A

The firm cannot charge the estate without consent from the deceased’s son.

Correct.

S.29 Trustee Act 2000 permits professional trustees (such as solicitors) to charge for their services. Without this statutory provision charging for their services would be “self-dealing” and a breach of fiduciary duty not to profit from their role. The power to charge applies to personal representatives as well as trustees.

However, the statutory power to charge only applies where a firm is not acting alone, the charges are reasonable, and consent from the co-trustees is obtained. As such, the firm will need to obtain consent from the deceased’s son (the co-trustee). There is no obligation to obtain consent from beneficiaries.

incorrect
The firm may charge the estate without the need for any consent but only if the amount is reasonable.

Incorrect. While it is correct that professional trustees may only charge a reasonable amount, in addition, consents from co-executors are required. Review your materials relating to s.29 Trustee Act 2000 and ensure you understand the circumstances in which professional trustees may charge.

270
Q

A testator died three months ago leaving a valid will which appointed his two adult children as his executors and giving the whole of his estate to his spouse.

The estate assets include a rental property. The executors have just received £1,500 from the tenants of the property. The tenants were behind in paying their rent and this amount is the rent that was due for the six weeks before the testator’s death.

Which of the following best describes the income tax (‘IT’) position in relation to this rental income?

The rent is the testator’s income for the tax year of his death. The IT payable is a post-death expense and has no effect on the value of the testator’s inheritance tax estate.

The rent is estate income because it was received after the testator died. The IT due is a post-death administration expense and has no effect on the value of the testator’s inheritance tax estate.

The rent is income of the testator’s spouse because she will inherit the property. The IT payable is not an administration expense and has no effect on the value of the testator’s inheritance tax estate.

The rent is the testator’s income for the tax year of his death. The IT due is a liability owed by the testator at the date of his death and will reduce the value of his inheritance tax estate.

The rent is income of the testator’s spouse provided the executors transfer the property to her. If the property is sold during the administration the rent is taxed as estate income instead and the IT liability will be a post-death administration expense with no effect on the value of the testator’s inheritance tax estate.

A

The rent is the testator’s income for the tax year of his death. The IT due is a liability owed by the testator at the date of his death and will reduce the value of his inheritance tax estate.

Correct.

The rental income relates to a period before the testator died (even if received after this date). Therefore the income is taxed as the testator’s for the tax year of death. The IT will be a liability owed to HMRC by the testator on the date he died and the tax is deductible from the value of the IHT estate. NB: although not relevant to the question, the income itself will be treated as an asset of the estate.

incorrect
The rent is income of the testator’s spouse provided the executors transfer the property to her. If the property is sold during the administration the rent is taxed as estate income instead and the IT liability will be a post-death administration expense with no effect on the value of the testator’s inheritance tax estate.

Incorrect
Incorrect. Review your materials on how payments received post death but relating to a period prior to death are treated and review your notes on post-death administrative expenses.

271
Q

The executors of an estate (A and B) wish to arrange for the deceased’s bank account (with a balance of £60,000) to be closed. The deceased’s estate is solvent but the deceased had several creditors and owed a significant amount to X.

Which of the following is correct?

A and B should provide the bank with a copy of the deceased’s will confirming their appointment as executors and request a cheque for the closing balance.

A and B should provide the bank with the grant of probate confirming their appointment as executors and request a cheque for the closing balance.

A and B should provide the bank with a copy of the death certificate and the estate accounts and request a cheque for the closing balance.

X should contact the bank directly providing proof of the debt and request that the bank provide X with the amount owed.

A and B should provide the bank with a copy of the estate accounts, confirm the deceased’s banking details and request a cheque for the closing balance.

A

A and B should provide the bank with the grant of probate confirming their appointment as executors and request a cheque for the closing balance.

Correct. Although executors derive their authority to act from the deceased’s will the grant of representation is the official document that confirms they have authority to administer the deceased’s assets. The other options were incorrect as i) the estate accounts are not finalised until the end of the administration and ii) the bank has no authority to pay the deceased’s money to a creditor.

272
Q

A woman died and her estate comprises her house (£300,000), cash in bank accounts (£25,000) and her car and other personal possessions (£10,000).

On the date she died her debts were £40,000. Following her death, administration expenses of £10,000 have been incurred.

By her will the woman left a legacy of £30,000 to her neighbour and the residue of her estate to charity.

Which of the following is correct?

The woman’s estate is solvent because the total value of her estate is greater than her debts and the legacy to her neighbour.

The woman’s estate is not solvent because there is insufficient cash to pay the legacy to her neighbour.

The woman’s estate is solvent because the total value of her estate is greater than her debts.

The woman’s estate is solvent because the total value of her estate is greater than her debts and the administration expenses.

The woman’s estate is not solvent because there is insufficient cash to pay the debts.

A

The woman’s estate is solvent because the total value of her estate is greater than her debts and the administration expenses.

Correct. An estate is solvent if the deceased’s debts and administration expenses can be paid in full (here £50,000). The other options were incorrect because: i) it is irrelevant whether the legacies can be paid, and ii) if insufficient cash is available the non-liquid assets (house / chattels) can be sold/used for the payment of debts, expenses and legacies.

273
Q

intestacy statutory order (if no spouse and no issue)

A
  • Parents, but if none;
  • Siblings of whole blood (share both parents) on the statutory trusts, but if none;
  • Siblings of half-blood (share one parent) on the statutory trusts, but if none;
  • Grandparents, but if none;
  • Uncles/Aunts of whole blood on the statutory trusts, but if none;
  • Uncles/Aunts of half-blood on the statutory trusts, but if none;
  • Crown as bona vacantia
274
Q

definition of chattles

A

s. 55(1)(x) AEA defines chattels as: tangible movable property except:
* cash
* assets used at the date of death solely or mainly for business purposes
* assets held at death solely as an investment (some personal use means the item is a chattel, even if the deceased purchased the item hoping it would increase in value.

275
Q

intestacy and family home

A
  • PRs can choose which assets to appropriate to a beneficiary in satisfaction of their entitlement. However, a surviving spouse has a right to receive the intestate’s share of the family home in or towards the satisfaction of their inheritance, provided the right is exercised within 12 months of the grant.
  • The spouse’s right of appropriation is irrelevant if the deceased’s interest in the family home was held as joint tenants. The property passes by survivorship outside of the succession estate and the intestacy rules do not apply in any event.
276
Q

IPFDA and jurisdicition

A

Deceased (not applicant) must have died domiciled in England & Wales.

277
Q

IPDFA timing requirements

A

Claims must be made within made within six months of the grant of representation (subject to a court order permitting an extension of the time limit)

278
Q

IPFDA applicant

A

Only certain people are entitled to bring a claim:
* Spouse of the deceased
* Former spouse, provided they have not remarried
* Person living with the deceased in a relationship akin to marriage for the 2 years immediately prior to death (a cohabitee)
* Child of the deceased (includes adopted children). Both adult and minor children may apply.
* Person treated by the deceased as a child of their family (e.g. step-child)
* If none of the other categories apply, any other person who was financially maintained (wholly or partly) by the deceased immediately before their death. Financial maintenance means substantial voluntary contribution in money or money’s worth towards reasonable needs, and not under a commercial arrangement.

An applicant may bring a claim in the High Court (Family or Chancery Division) or County Court. Note that entitlement to bring a claim does not mean a claim will be successful.

279
Q

IPFDA ground

A

A claim must be brought on the ground that the distribution of the deceased’s estate under their will, intestacy, or a combination of both fails to make “reasonable financial provision” for the applicant.

280
Q

IPFDA standard

A

To determine whether “reasonable financial provision” has been made the court will apply one of two standards.

The surviving spouse standard is used where the application is brought by a spouse, and the maintenance standard is used for all other applicants.

  • Surviving spouse standard: what is reasonable in the circumstances for a spouse to receive, whether or not required for their maintenance.
  • Maintenance standard: what is reasonable in all the circumstances of the case for the applicant to receive for their maintenance.

The surviving spouse standard is more generous.

Note that a court has discretion to apply the surviving spouse standard where the three following conditions are satisfied:
* The applicant is a former spouse who has not remarried, or a spouse who is judicially separated from the deceased, and
* Divorce, dissolution, nullity or judicial separation occurred within 12 months of the death, and
* No order for financial provision has been made or refused in the ancillary proceedings.

The court must apply statutory guidelines when assessing firstly, whether ‘reasonable financial provision’ was made, and if it was not, to determine the form and quantum of an award. It is an objective process and all factors have equal weight.

281
Q

IPDFA common guidelines

A

The following guidelines apply to all applicants:
* applicant’s financial resources and financial needs
* financial resources and financial needs of any other applicants
* financial resources and financial needs of any beneficiary of the estate
* obligations and responsibilities the deceased had towards any applicants or beneficiaries
* size and nature of the net estate of the deceased
* any physical or mental disability of any applicant or beneficiary
* any other matter the court considers relevant in the circumstances (including the conduct of the applicant or any other person)

When considering financial resources and needs, the court must take into account resources and needs they are likely to have in the foreseeable future.

282
Q

IPFDA specific guidelines

A

in addition to common guidelines!

Application by spouse
* applicant’s age and the duration of the marriage
* the contribution made by the applicant to the welfare of the family of the deceased, including contribution made by looking after the home or caring for the family

Application by former spouse
If the court exercises discretion to use the surviving spouse standard they will consider, in addition to the two bullet points above:
* what provision the applicant might reasonably have expected to receive in divorce / dissolution proceedings if the couple had ended their relationship at the date of death.

Application by cohabitee
* age of the applicant and the length of the period of cohabitation.
* the contribution made by the applicant to the welfare of the family of the deceased, including any contribution made by looking after the home or caring for the family.

Application by deceased’s child
* the manner in which the applicant was (or might expect to be) educated or trained.

Application by someone treated as a child of the deceased
* the manner in which the applicant was (or might expect to be) educated or trained.
* the basis on which the deceased maintained the applicant, for how long and to what extent
* the extent to which the deceased assumed responsibility for the applicant’s maintenance.
* whether the deceased maintained or assumed responsibility for maintaining the applicant knowing that the applicant was not their child
* the liability of any other person to maintain the applicant.

Application by a person maintained
* the basis on which the deceased maintained the applicant, for how long and to what extent
* whether and to what extent the deceased assumed responsibility for the maintenance of the applicant.

283
Q

IPFDA order

A

Once the court has decided that the applicant’s claim should succeed, they have wide powers to make a range of orders, which can include lump sum and periodical payments.

284
Q

Assets subject to the IPFDA order

A

Court orders may be made in relation to the deceased’s ‘net estate’. This includes:
* deceased’s succession estate
* property over which the deceased had a general power of appointment which was unexercised
* property subject to a statutory nomination or given away as DMC
* deceased’s severable share of a joint tenancy (if ordered under s. 9)
* any property the deceased gave away during their lifetime if covered by the court’s anti-avoidance powers under ss 10 and 11.

285
Q

pre grant steps x13

A

1 – Obtain official copy of the death certificate (usually a family member will provide this)

2 – Arrange funeral (usually a family member will do this)

3 – Secure deceased’s property / notify insurers

4 – Locate original will / codicil
* Check date / confirm it is the “last will”
* Any issues with validity?
* Might affidavit evidence be needed ?
* Consider effect s.9/15/18/18A/21/24/33 Wills Act 1837
* Has it been revoked?

5 – Establish succession estate and consider the basis of the distribution (intestacy/will/both if partial intestacy)

6 – Identify/locate beneficiaries (consider whether a claim under the IPFDA is possible/likely?)

7 – Prepare Schedule of Assets & Liabilities
* Arrange valuation of property / chattels / shares (date of death values)
* Notify banks/financial institutions/life insurance companies etc of death & request final statements

8 – Prepare Schedule of Lifetime transfers

9 – Collect assets that can be collected without a grant
* Within the Administration of Estates (Small Payments) Act 1965
* Proceeds of life policies written in trust
* Discretionary pension schemes
* Statutory nominated property
* Property held as joint tenants
* Donatio mortis causa

10 – Calculate IHT due and complete IHT form to account to HMRC (IHT 400)
* Submit IHT400 to HMRC along with payment of any IHT due. HMRC send receipt Form IHT421 (probate summary) to the probate registry.
* Submit PA form to the probate registry together with relevant fees.
* IHT form must be submitted within 12 months of death. However, interest is due on unpaid IHT 6 months from end of month of death therefore this is effectively the deadline for paying IHT and submitting forms to HMRC.

11 – Raise £ to pay IHT ( This needs particular consideration because assets will not usually be released to the PRs until the grant has been issued, but the grant will not be issued until IHT is paid.)
* Loan from beneficiary (usually interest free)
* Direct Payment Scheme (banks release £ directly to HMRC – not to PRs)
* Bank loan (interest will be charged)

12 – Complete online probate application or PA1P where there is a will and PA1A where there is no will. Signed by applicants (or their legal representative if authorised).

13 – Submit documentation to the probate registry:
* Online / PA1P / PA1A
* Original will / codicil
* £ probate application fee
* £ fee per sealed copy of the grant
[Death certificate may not be required where a professional is submitting the application]

You may also need to provide:
* Affidavit evidence
* Form of renunciation
* Confirm notice of reservation of power was given
* Power of Attorney

286
Q

Post grant steps x7

A

1 – Notify HMRC about changes to the IHT account using Form C4:
* re extra assets or liabilities
* correction to original valuations
* corrections to amount of relief claimed
* if any additional IHT is due, send the extra £

2 – Place s.27 Trustee Act 1925 notice
* protect PRs from claims by unknown creditors / beneficiaries if they distribute the estate after the two month notice period has passed

3 – Collect assets
* once PRs have the grant a sealed copy is sent to each asset holder who will put assets into the PRs’ names or close accounts and send cash balance to PRs
* cash is usually collected in a PR or law firm client account

4 – Pay debts and distribute estate to beneficiaries, e.g.
* administration costs (includes legal fees, estate IT and CGT liabilities for the administration period)
* specific legacies
* pecuniary legacies
* interim distribution of part of residue
NB: Delay distribution:
* until after s.27 notice deadline expires
* if there is reason to believe an IPFDA claim may arise wait 6 months from the issue of the grant

6 – Prepare Estate Accounts
* signed by the PRs and residuary beneficiary(s)

7 – Make final distributions to residuary beneficiaries from client account
* PRs expected to complete administration within 12 months of death - known as the ‘executor’s year’.

287
Q

inheritance tax – death estate

A

7 steps

1 – cumulative total
2 – identify taxable estate
3 – value taxable estate
4 – deduct debts/liabilities
5 – exemptions and reliefs
6 – identify/ apply available RNRB
7 – Identify and apply basic NRB and calculate IHT

288
Q

inheritance tax – death – step 1

A

step 1: cumulative total
* Cumulative total = the total chargeable value of all the chargeable transfers made in the 7 years before death.
* Chargeable value = the value of a chargeable transfer after exemptions and reliefs have been applied – so is unlikely to be same figure as the amount transferred.
* Chargeable transfers = LCTs and failed PETs.
* The effect of the cumulative total is to reduce the basic NRB for the death estate. Therefore, if the cumulative total is greater than £325,000 there will be no basic NRB for the estate (assuming no transferred amount).
* The cumulative total from Step 1 is taken into account later at Step 7.

[Please refer to the IHT (Lifetime) Summary for more detail on how the LCTs and PETs are taxed]

289
Q

Inheritance tax – death – step 2

A

step 2: identify taxable estate
* The taxable estate comprises all the assets/ interests in property that an individual may have which are potentially subject to IHT following their death

  • The following are excluded from a person’s taxable estate (i.e. never taxable):
    o Life policy lump sum written in trust
    o Discretionary pension lump sum payment
    o If a remainderman dies and the life tenant is still alive, the remainder interest is not subject to IHT
    o Discretionary Trust assets on the death of beneficiary – these are not assets in which any beneficiary has a fixed right or interest.
  • Unless an asset is mentioned above it will form part of the taxable estate even if the asset later qualifies for an exemption or relief.
  • The following items are included in a person’s taxable estate even if you might think otherwise:
    o Subject matter of a gift with reservation of benefit (GROB) – the value of the item given away during lifetime is included in the taxable estate at its date of death (not date of gift) value
    o If a life tenant dies, the value of the trust fund at the date of death is included in the life tenant’s taxable estate if the life interest was created by will (or came into existence before March 2006)
    o DMC gift, statutory nominations, and all jointly owned property whether held as joint tenants or tenants in common (you may have thought these were excluded because they are excluded from the succession estate)
290
Q

inheritance tax – death – step 3

A

step 3: value taxable estate
* The assets in the estate are included at their market value on the date of death
* Related property: If assets owed by spouses or civil partners are worth more when valued together (e.g. because they form a set e.g. share holdings which together give control but separately do not) each party’s share is valued on death as a proportionate share of the combined total.
* Jointly owned property: The value of a person’s share of jointly owned property is the propionate share of the whole. However, where land or buildings are co-owned (whether as joint tenants or tenants in common) the proportionate value of the deceased’s share is reduced by 10%. This does not apply where the co-owners are married /civil partners.

291
Q

Inheritance tax – death – step 4

A

Deduct debt/liabilities
* Any debts the deceased owed on the date they died e.g. overdraft or bills are deducted before the IHT is calculated.
* As an exception, funeral expenses and the cost of a tombstone can be deducted before IHT is calculated even those these costs are incurred after death. Other post-death expenses may be paid using the estate assets but cannot be deducted from the value of the estate for IHT purposes.

292
Q

inheritance tax – death – step 5

A

Exempt beneficiaries (spouse / charity)
* 100% of the value of assets passing to an exempt beneficiary (whether under intestacy / will / outside the succession estate / any combination) can be deducted before IHT is calculated. For this module spouse/civil partner exemption and/or charity exemption may apply.
* Spouse/civil partner exemption will apply to the total value of assets passing into a life interest trust where the spouse will be the life tenant.
* You need to understand how the deceased’s estate will be distributed to work out the value that a spouse or charity receives.

Exempt assets (business or agricultural property)
* The value of exempt business or agricultural assets can be deducted before IHT is calculated.
* Certain criteria must be satisfied before assets qualify for the relief. If these are met the rate of relief is either 100% or 50%. [Please refer to the adapt elements for detail of the criteria for APR and BPR]

After Step 5 you will have the value of the estate which is chargeable to tax.

293
Q

Inheritance tax – death – step 6

A

Step 6: Identify and apply available Residence NRB
* Apply the total RNRB available at 0%
* Amount of the RNRB is £175,000 (or the value of the Qualifying Residential Interest (QRI) if lower).
* It can be claimed where a QRI is closely inherited by a direct (lineal) descendant. QRI: includes the deceased’s home or interest in their home (and excludes commercial and let property). Closely inherited usually means absolutely, rather than by e.g. trust. Lineal descendant: includes the deceased’s children and grandchildren but has a wider definition which you should know of. Note: the deceased’s spouse/civil partner, sibling or parent are NOT direct descendants.
* Does the deceased have a spouse/civil partner who died before them and who did not use their RNRB at the time? If yes, the unused proportion can be claimed by the survivor’s estate.
* The RNRB is reduced for estate worth £2m+ . There is no RNRB available for estates worth more than £2.35M (single RNRB) or £2.7M (if double RNRB applies).

294
Q

inheritance tax – death – step 7

A

Step 7: Identify and apply basic NRB and calculate Inheritance Tax
* Amount of basic NRB is £325,000
* Does the deceased have a spouse/civil partner who died before them and who did not use their own NRB at the time? If yes, the unused proportion can be claimed by the survivor’s estate.
* Reduce the total NRB by the value of the cumulative total (from Step 1).
* Apply rate of 0% to the value of the remaining taxable estate up to the total NRB amount.
* Apply death rate of 40% to the remainder – this is the IHT due.

295
Q

The following transfers made during a person’s lifetime are chargeable to IHT:

A
  • PET (transfer to another person) which has failed because the donor died within 7 years of making the transfer.
  • LCT (transfer to a trust), which is chargeable when it is made @ 0/20%, and later, charged again @ 0/40% if the donor dies within 7 years of making the transfer
296
Q

inheritance tax – lifetime – steps

A

5- 7 steps (A-F)
If, after the donor dies, IHT becomes payable in respect of lifetime transfers (failed PETs and re-assessed LCTs) this IHT is in addition and separate to the IHT due in respect of the death estate assets.

A – cumulative total
B – identify value transferred
C – apply exemptions and reliefs
D – Apply NRB and calculate tax
Steps E and F are only considered if the donor has died
E – Taper
F – Credit for tax already paid

297
Q

IHT – life – A

A

Step A: Cumulative Total
* Cumulative total = the total chargeable value of all the chargeable transfers made in the 7 years before the transfer being taxed i.e. chargeable value of any failed PETs or LCTs.
* Chargeable value = the value of a chargeable transfer after exemptions and reliefs have been applied – so is unlikely to be same figure as the amount transferred.
* The effect of the cumulative total is to reduce the NRB for the lifetime transfer under consideration.
* The cumulative total from Step A is taken into account later at Step D.

298
Q

IHT – life – B

A
  • The value of a transfer is assessed by reference to the loss to the donor at the date of the transfer.
  • If the transfer is cash the loss to the donor will be the same as the amount transferred. For other assets, it will usually be the market value of the item / same as the value received by the donee.
  • Where there is a transfer for less than full consideration the “loss” to the donor is the difference between the price paid and the market value.
  • If IHT is payable when an LCT is made and the donor pays the IHT (in addition to the gift itself), the reduction in the value of his estate includes the amount of IHT paid as well as the gift itself and the amount of the gift must be ‘grossed up’ to find the total ‘value transferred’ for IHT purposes before the tax due is calculated. You are not required to carry out any calculation where grossing up is required, but should be aware of the legal principles.
299
Q

IHT – life – C

A
  • Reduce the value of the transfer by deducting the value of any exemptions or reliefs that apply – the chargeable value is what remains after the exemptions and reliefs have been applied.
  • Consider the following for lifetime transfers:
    o Spouse/civil partner exemption: 100% of value of the transfer.
    o Charity exemption: 100% value of the transfer.
    o Family maintenance exemption : uncapped amount – transfer to a) child for maintenance, education, or training (full time required if child is over 18), or b) to provide care for dependent relative
    o Annual exemption: £3,000 per tax year (6 April – 5 April). Use tax year of gift first, and if more is needed, use of any from previous tax year that is still available – ie. maximum of £6,000
    o Small gifts allowance: £250 per tax year per person (no limit to number of different people). Cannot be combined with any other exemptions / does not apply at all if transfer is more than £250.
    o Normal expenditure from income: uncapped amount. Transfers are exempt provided they are made from income (not capital), part of a regular pattern of giving and do not affect the donor’s standard of living.
    o Marriage exemption : £5,000 if made by parent, £2,500 re grandparent, £1,000 re everyone else. Relief applies per marriage, and per donor.
    o Business property relief: 2 year qualifying period of ownership (unless special rules following death apply).
    o Relief at 100% for private company shares, partnership interests or sole trader business. Relief at 50% for quoted shares but only if donor had control, or if the donor owned the assets which were used by a business. NB: no relief for investment assets e.g. let property. Further detail can be found in the adapt element.
    o Agricultural property relief: 2/7 year qualifying period of ownership, depending on whether donor occupied the land for agricultural purposes, or if it was owned by the donor but occupied by someone else for agricultural purposes.
    o Relief applies to the agricultural value of the land (usually less than the market value). Most often at a rate of 100%, but in limited circumstances a 50% rate applies (for detail see adapt element).
300
Q

IHT – life – D

A

Step D: Apply NRB and calculate tax
* Establish the value of the NRB (and any transferred NRB if calculating IHT due following death of the donor).
* Reduce the total NRB by the value of the cumulative total (from Step A).
* Apply a rate of 0% to the chargeable value of the transfer up to the total NRB amount.
* Apply the lifetime rate of 20% (LCT when made) or death rate of 40% (for failed PET or re-assessed LCT) to the balance to establish the IHT due.
* RNRB does not apply to lifetime transfers.
* NRB amount applicable to an LCT when it is first made is the NRB at the date of the transfer. The NRB that applies to a failed PET or re-assessed LCT is the NRB amount at the date of death.

301
Q

IHT – life – E

A

Step E (Taper)
* If, following the donor’s death, a failed PET or re-assessed LCT results in IHT becoming due (i.e. at Step D the NRB is used up and there is a tax charge) taper should be considered. If there is no tax payable at Step D there is nothing to taper so Steps E and F are irrelevant.
* Taper relief reduces the tax charge by a % based on the number of years the donor survived after making the lifetime gift:

Up to 3 years before death
- reduction in IHT: nil
- amount of IHT still payable: 100%

3 to 4 years before death
- reduction in IHT: 20%
- amount of IHT still payable: 80%

4 to 5 years before death
- reduction in IHT: 40%
- amount of IHT still payable: 60%

5 to 6 years before death
- reduction in IHT: 60%
- amount of IHT still payable: 40%

6 to 7 years before death
- reduction in IHT: 80%
- amount of IHT still payable: 20%

302
Q

IHT – life – F

A

Step F (Credit for tax already paid)
* If an LCT is being reassessed following the donor’s death any tax that was paid at the lifetime rate can be deducted from the amount still due after Step E. Only the balance then needs to be paid to HMRC.
* If the balance is nil after crediting the lifetime amount already paid, there will be no further tax to pay.
* It is not possible to obtain a refund for the lifetime payment if the balance is negative.

303
Q

Testamentary capacity

A
  • There is no statutory presumption of testamentary capacity unlike the general test for capacity in the Mental Capacity Act 2005.
  • The burden of proving testamentary capacity lies with those propounding the will i.e. usually the PRs when submitting the will for probate.
  • However, there is a common law presumption of testamentary capacity if the will is properly executed, rational on its face, and there is no reason to doubt the testator’s testamentary capacity.
  • If real doubt is raised, the common law presumption does not apply and the propounder must demonstrate testamentary capacity was present.
304
Q

Knowledge & Approval (intention)

A
  • The burden of proof lies with the propounder of the will. However, there is a presumption of knowledge and approval where the will is duly executed, the testator had testamentary capacity, and the testator signed their own will having had the opportunity to read it.
  • The presumption does not apply (and the propounder must prove knowledge and approval) if:
    o the testator is blind or illiterate, or the will is written in a language the testator cannot read and understand
    o someone else signs the will on behalf of the testator
    o there are suspicious circumstances
  • If the presumption will not apply, the attestation clause should be amended to reflect any steps taken to ensure the testator had knowledge and approval. For example, where a will is read out loud to a blind testator who confirms their understanding and approval before signing. In this way, the attestation clause acts as evidence of knowledge and approval.
  • If the presumption does not apply, and the attestation clause does not expressly confirm that appropriate steps were taken, an affidavit of knowledge and approval should be submitted to the probate registry as part of the application for probate.
305
Q

Knowledge & Approval – Duress/ Undue Influence

A

A will executed under the duress or undue influence does not reflect the true intention of the testator and therefore knowledge and approval will not be present, and the will is invalid. The burden of proof is with the person alleging duress or undue influence occurred.

306
Q

s.9 Wills Act 1837

A

o Compliance with s.9 is a matter of fact. There is no legal obligation to include an attestation clause nor is any specific form of attestation required.
o However, a properly drafted attestation clause raises a presumption that the will was executed in accordance with the requirements of s 9 (a presumption of due execution). In the absence of, or a poorly worded, attestation clause proof of due execution is required, usually by way of an affidavit of due execution sworn by the witnesses.

307
Q

commencement

A
  • Identifies testator and purpose of document
  • Date can be included in the commencement or at the end of the will.
  • A will is valid without a date but a missing date causes confusion if there is more than one will /codicil
308
Q

revocation

A
  • ensures all previous wills/codicils are revoked so there is only one valid will
  • a will is valid without a revocation clause
  • if a testator has more than one will, the later will impliedly revokes the earlier will but only to the extent that it is inconsistent with the earlier will
309
Q

Executors and trustees

A
  • An executor’s power derives from their appointment under the will
  • Any adult with mental capacity may be appointed (solely or jointly)
  • A testator may name a substitute executor in the event that an original executor does not act
  • A sole executor may act even if life or minor interests arise, but a maximum of four may be named on the grant
  • Executors are often also appointed as trustees
  • S.18A/C WA: the appointment of the testator’s spouse/civil partner is ineffective if the testator later divorces/dissolves the civil partnership
  • If no executor is named/able/willing to act, NCPR 20 applies and an administrator will be appointed
  • Lay executors cannot charge for their time so an express charging clause should be considered
  • S.15 WA has no effect on the appointment of a beneficiary as executor
310
Q

Guardians for children

A
  • A testator with parental responsibility can appoint a legal guardian for their infant children by will
  • The appointment would not normally take effect until after the death of the surviving parent
311
Q

Specific legacy

A
  • Gift of a specific item which is distinguished from other property of a similar type
  • The subject matter is identified at the date of the will because the word “my” demonstrates a contrary intention to the general rule in s.24 WA
  • The gift will therefore adeem (fail) if the item is not owned by the deceased at the date of death and the beneficiary receives nothing unless an express substitution clause is as included
  • Unlike a gift of a specific item, a gift of a specific collection (“my chattels”, “my shares in [ ] plc”, “my rosewood antique furniture”) takes effect with reference to the items in the collection on the date of death
312
Q

general legacy

A
  • A gift of property which is not distinguished from property of a similar type. In this example, it does not matter which shares are given.
  • If the deceased does not own the items referred to the gift does not adeem, instead the PRs would have to buy the specified property
  • Most pecuniary legacies (gifts of cash) are general in nature e.g. “I give £1,000 to [ ]”
313
Q

demonstrative legacy

A
  • Where the will directs that the gift should be paid out of a specified fund
  • The legacy will not fail if there are insufficient assets in the specified fund
  • To the extent the specified fund is inadequate the balance is paid as a general legacy from other assets in the estate
314
Q

gift of residue

A
  • A gift of all of the testator’s property capable of passing by the will which has not already been disposed of under the will or any later codicil and is not needed to meet debts, liabilities, taxes and expenses.
  • The residue estate will often be defined before the terms of the gift are stated
  • If the residue clause or any part of it fails e.g. if a beneficiary of a specific share has pre-deceased there may be a partial intestacy if no substitution provisions apply
315
Q

[Administrative Powers]

A
  • If no express administrative powers are included only statutory and common law powers will apply
  • If express powers are included, they take priority over the default powers to the extent of any discrepancy
316
Q

attestation clause

A

Signed by [ ] in our joint presence and then by us in hers
Date: [ ]

  • The attestation describes the circumstances in which the will was signed and confirms the requirements of s. 9 WA are met. It should be amended to note special circumstances surrounding the execution e.g. if the will was read out loud.
  • A will is valid without an attestation clause but affidavit evidence may be required to demonstrate s.9 compliance.
  • If beneficiary (or their spouse) witnesses the will although the will remains valid, the beneficiary loses their entitlement under the will (s 15 WA)
  • The date usually appears with the attestation or commencement. A will is valid without a date but results in confusion if there is more than one will/codicil.
317
Q

Relevant to all dispositive clauses

A
  • Objects of a gift (i.e. identity of the beneficiary): the will is deemed to speak from the date of execution, unless there is express wording to the contrary. If a gift is given to a “class” of beneficiaries, unless express words clarify when members of the class are identified, ‘class closing’ rules apply and the class closes when the first beneficiary in the class obtains a vested interest.
  • Effect of codicils: a codicil republishes the will it amends to the date the codicil is signed.
  • IHT: gifts are made free of tax unless otherwise stated
  • Transfer costs: gifts are subject to transfer costs unless otherwise stated
  • Charges: a charged asset bears liability for payment of the secured loan unless will directs otherwise s.35 AEA
  • Survivorship provisions: there are no deemed survivorship provisions, so these need to be expressly drafted
  • Substitution provisions: there is no deemed substitution effect and a gift to a beneficiary who has pre-deceased would usually fail if there is no express substitution clause. However, s.33 WA is an exception; if a testator makes a gift to issue (children, grandchildren, great-grandchildren etc) who pre-decease, leaving their own issue, that issue can take by way of substitution the share the original beneficiary would have taken
  • Legacies can be drafted as contingent or vested - the most common contingency is the requirement to reach a specific age
    Trusts law: the equitable rules in relation to creation of a valid gift or express trust apply to the dispositive clauses within a will. You should be able to review these and conclude whether a valid gift/ fixed interest trust/ discretionary trust/ power of appointment has been created.
318
Q

different types of grants

A

grant of probate

grant of administration (with will)

grant of administration

319
Q

grant of probate

A

The grant of probate is required for estates where:
The deceased left a valid will
The will appoints executors
At least one of the executors appointed is going to act

The grant of probate is issued in the name(s) of those executors who apply.

A grant of probate is required even if the will does not dispose of any/all of the deceased’s property.

For example, the whole or part of the estate is left to a pre-deceased family member by will and no substitution provisions apply.

If the appointment of an executor under the will is limited e.g. to specific assets, jurisdiction or by time, then this will be reflected in the authority conferred by the grant.

Only those named may take out the grant and they cannot simply give that right to someone else. Yet a named executor may formally appoint someone to act on their behalf under a power of attorney.

320
Q

Grant of Letters of Administration (with will)

A

A grant of letters of Administration (with will) is the appropriate grant where:
the deceased left a valid will
but the will appoints no executors who are willing/able to act.

This is the correct grant even if the will fails to dispose of all the estate.

Administrators are appointed under the Non-Contentious Probate Rules 1987 (‘NCPR’). Rule 20 applies and lists, in order of priority, those entitled to apply for the grant.

Administrators (not executors) are appointed. Their entitlement to apply derives from Rule 20 NCPR not the will.

Rule 20 lists, in order of priority, those entitled to make an application for the grant.

The order in Rule 20 broadly follows the entitlement to the estate assets

321
Q

Grant of Letters of Administration

A

A grant of Letters of Administration is the appropriate grant where the deceased died without having made a valid will (i.e. died intestate).

This may be because they did not make a will at all, had revoked a will they did make, or the will they made is invalid.

Administrators are appointed under NCPR 22 which lists, in order of priority, those entitled to apply for the grant.

Administrators (not executors) are appointed.

Their entitlement to apply derives from Rule 22 NCPR.

Rule 22 lists, in order of priority, those entitled to make an application for the grant.

The order in Rule 22 follows the entitlement to the estate under intestacy.

322
Q

Administration: Assets without a grant

A

Where the following assets are included in the succession estate, no grant is required for the PRs to deal with them:
- Assets which can be distributed under Administration of Estates (Small Payments) Act 1965
- Personal household possessions
- Cash

323
Q

Administration of Estates (Small Payments) Act 1965
Administration: Assets without a grant

A

Orders made under this Act permit payments to be made to persons who appear to be beneficially entitled to the assets without formal proof of title.

There is a restricted category of assets to which the Act applies.
National Savings (inc. Bank accounts, Savings Certificates and Premium Bonds)
Friendly Society and Industrial and Provident Society deposit accounts.
Arrears of salary and wages
Pensions where the deceased was a member of the police, fire authority, air force or army.
Building society accounts

There is an upper financial limit of £5,000 per asset. If the value of the asset is greater than £5,000 a grant is required to establish title to the whole sum, not just that in excess of £5,000. The provisions only make payments permissible, they do not compel those holding the assets to do so.

In practice, these statutory provisions are rarely required because most banks and financial institutions adopt their own policy and will release sums (commonly up to £15,000 but sometimes larger amounts) without sight of the grant.
Each institution will have its own evidential requirements before closing an account. Most will require sight of the death certificate and will (or confirmation of entitlement under intestacy) as evidence that the recipient is entitled to the money. The PRs may also be expected to sign an undertaking confirming their right to administer and give an indemnity to the bank in the event the payment is made to the wrong person.

Example:
An estate incudes a
Building Society account worth £60,000
Premium Bonds worth £2,000
High street bank current account worth £10,000
Building Society Account: A grant is required because the value is greater than the statutory limit under the Act.
Premium Bonds: The Act would apply and these could be administered without a grant.
Current Account: although technically not subject to the Small Payments Act, the bank, following its own polices, may agree to close the account without sight of the grant.

324
Q

Personal possessions & cash
Administration: Assets without a grant

A

Title to personal household possessions passes by delivery and proof of ownership is not required when they are sold (with the exception of cars where registration documents are needed).
Therefore, PRs are normally able to dispose of chattels without having to produce formal proof of their authority.
This applies where items were owned solely by the deceased. If there was a joint owner they would need to consent before the PRs could effect a sale.
Before any items are sold PRs should check whether any particular items were gifted specifically by the deceased’s will.
A PR does not need a grant to take possession of cash found at the deceased’s home.

325
Q

Property that does not devolve on PRs

A

Assets which pass outside of the succession estate (in other words, cannot pass by a will or intestacy) do not require a grant in order for them to be released.

For these assets the PRs will need to produce the death certificate, together with any other documentation the asset holder requires.

Items passing outside of the succession estate and process for dealing with these following death: x4

  • Property owned as joint tenants(commonly land and bank accounts): On the death of one owner the property passes automatically to the survivor under the rules of survivorship and this does not rely on the issue of a grant. The land registry / bank will transfer title into the name of the surviving owner.
  • Donationes mortis causa(DMC): Here the deceased would have transferred ownership or control of the asset to the beneficiary during their lifetime.
  • Life policieswritten in trust,discretionary pension lump sumsnominated for a third party, and other nominated assets. On production of the death certificate these funds would be payable to the named beneficiary.
  • Assets held in a trust in which the deceased had an interest:The trustee should be notified of the death. The trust deed will determine what happens to the trust fund (if anything) following the death of a beneficiary.
326
Q

administration - PR - practical considerations

A

There are advantages to having some assets in an estate that can be administered without a grant:
releases money which can be made available to beneficiaries without waiting for the grant or administration to be completed
provides a source of funds to meet expenses, including IHT
if the estate is small or comprises only assets which do not require a grant, it can be a cost-effective way to carry out the administration

Note that unless an estate comprises only assets where a grant is not required the PRs will need to obtain a grant of representation.

Once a grant is required, it is usually simpler to administer the whole estate with reference to the grant, rather than try and administer some assets without it, even if in principle that might be possible.

327
Q

NCPR 20

A

The statutory order of entitlement to be appointed as administrator under a grant of letters of administration (with will) is shown below.

Legislation: NCPR 20
a) executor;
b) trustee of the residuary estate;
c) any residuary beneficiary (whether taking absolutely or for life), or, where there is a partial intestacy, a beneficiary of the estate under intestacy;
d) the PRs of anyone in (c) other than a trustee or life tenant of the residue;
e) any other beneficiary or a creditor;
f) PRs of anyone in (e).

Entitlement to the grant correlates with the distribution of the estate under the will, not the applicant’s familial relationship with the deceased.

A person in one category cannot apply if anyone in a higher category is able and willing to act as administrator. A trustee of the residue (b) cannot apply if there is an executor (a) willing and able to act. A residuary beneficiary (c) cannot apply if there is either an executor (a) or trustee of the residue (b) willing and able to act.

Those within the same category have an equal right to apply (a beneficiary with a vested interest is preferred over one with a contingent interest). If there are 3 residuary beneficiaries (c) [and no executor (a) or trustee of the residuary estate (b)] all 3 beneficiaries have an equal right to apply.

The applicant(s) must explain why anyone with a better right to apply is not doing so (known as “clearing off”). Applicants do not need to explain why a person in the same category i.e. with an equal entitlement is not making the application. If one of two residuary beneficiaries (c) applies for the grant, the applicant must “clear off” (b) by e.g. confirming no trustee of residue is appointed, and (a) by e.g. confirming the sole appointed executor has pre-deceased. But the applicant is not required to “clear off” anyone in (c), i.e. explain why the other residuary beneficiary is not applying.

Example 1
The deceased left a valid will that appoints their spouse as executor. The residue of the estate is left on trust for their adult children. The deceased’s brothers are appointed as the trustees. The deceased’s spouse has pre-deceased. Who will apply for the grant? Under what authority?
Check: Does NCPR 20 apply? Yes, the deceased left a valid will but because their spouse pre-deceased the will does not appoint an executor who can act.
So who will apply? Either or both of the brothers could apply as residuary estate trustees under NCPR 20(1) (b) (not in their capacity as the deceased’s siblings). The brothers would have an equal right to apply and will need to “clear off” the deceased’s spouse.

Example 2
The deceased left a will that does not appoint any executors. The will gives £10,000 to the deceased’s friend and the residuary estate absolutely to the deceased’s adult niece and nephew. Who will apply for the grant? Under what authority?
Check: Does NCPR 20 apply? Yes, the deceased left a valid will but the will does not appoint any executors.
So who will apply? Either or both of the niece or nephew would have an equal right to apply as residuary beneficiaries under NCPR 20 (1) (c). They would need to clear-off the executors (a) and trustees of the residue (b), by showing that there was no-one appointed

328
Q

NCPR 22

A

same as statutory order for intestact but logically include spouse/issue before! but must also show beneficial entitlement see example belwo

The statutory order of entitlement to apply to be appointed as administrator under a grant of letters of administration is shown under NCPR 22.

NCPR 22
a) surviving spouse or civil partner
b) children of the deceased
c) father and mother of the deceased
d) whole blood siblings (share both parents)
e) half-blood siblings (share one parent)
f) grandparents
g) uncles / aunts of whole blood
h) uncles / aunts of half blood

Issue of b, d, e, g, and h are included where their parent has pre-deceased.

If there is no-one who can apply from (a) – (h):
The Crown (claiming bona vacantia) may apply (22(2))

If the Crown does not apply, a creditor, or person who does not receive benefit (but would have done if the estate was larger) may apply (22(3))
Similarly to Rule 20: a person in one category cannot apply in priority to someone in a higher category, those within the same category have an equal right to apply, and applicants must “clear-off” anyone with a better (but not equivalent) right to apply.

Under Rule 22(4), the PR of any applicant who survived the deceased but died before taking a grant may apply on their behalf, although an application by a living person within the same category is preferred.

Applicants must demonstrate the nature of their familial relationship with the deceased (to evidence which category they fall within). In addition, they must also have a beneficial entitlement under the estate. Commonly, the applicant next in order to apply will have an entitlement to the estate, but this will not always be the case. Consider the example below.

Beneficial EntitlementExample: A man dies intestate leaving an estate worth £200,000. The man is survived by his spouse and two adult children.

The spouse has the best right to apply (a) but cannot do so until at least 28 days after the man’s death. This is because under the intestacy rules she is not beneficially entitled to the estate unless she survives the man by 28 days (s 46(2A) AEA).

And, if the spouse choses not apply….
The adult children are the next category entitled (b). However, the value of the man’s estate is less than the statutory legacy so the whole estate passes to the spouse. The children therefore have no beneficial entitlement.

Where no-one in a)–h) with a beneficial entitlement to the estate will apply, a person with no immediate entitlement may do so, provided they would have benefited if the estate were larger (22(3)). As the children would have taken a share of the estate with the spouse if the man’s estate was more valuable, they are able to apply.

Example NCPR 22
A deceased died intestate and was survived by their mother and brother. The deceased was never married or in a civil partnership, did not have any issue, and their father had predeceased. Who will apply for the grant? Under what authority?
Check: Does NCPR 22 apply? Yes, the deceased died intestate.
So who will apply? The deceased’s mother is entitled to the whole estate. She has the best right to apply under (c) and will clear-off (b) (explaining there are no issue), and clear-off (a) (explaining the deceased never married). She does not need to clear-off the father’s right to apply because he falls within the same category.
The deceased’s brother has no right to apply because he falls in a lower category of applicant (d). However, even if the mother decided not to act the brother would still be unable to apply. He would be next in order under Rule 22 but he has no beneficial interest in the estate. No matter how large the estate is, it all passes to the mother. The brother would only be able to apply if he were a creditor of the estate under Rule 22(3).

329
Q

entitlement to a grant (administrators) restrictions

A

Capacity to act
Someone who lacks mental capacity may not apply for a grant. Ideally, there would be someone else with an equal or greater entitlement to apply. However, if there is not, Rule 35 will determine who can act. The detail of Rule 35 is outside the scope of the module.
A minor may not act as administrator, although it is possible for someone to apply for a grant on their behalf. However, an application by an adult with equal entitlement to apply is given priority over an application made on behalf of a minor (Rule 32).
An application on behalf of a minor may be appropriate where:
No adult with equal or greater entitlement will act
The minor is the only person within the category having the greatest entitlement, or, all those within the category are minors.

Number of Administrators— for both letters of administration (with will) & letters of administration:
Only one administrator is required, unless, there are minor or a life interests in the estate, in which case two will be required (s.114 Senior Courts Act 1981). i.e. two are needed if any part of the estate is passing to a minor beneficiary or is to be held on a life interest trust.
Where two administrators are required but there is only one able/willing person in the category with best entitlement to apply, that person may apply for the grant with somebody from the next category of entitlement (Rule 25(1) NCPR).
A maximum of four applicants may apply.

330
Q

administration - renunciation

A

A potential administrator may renounce at any time before the grant is issued. They are not prevented from renouncing even if they have intermeddled with the estate.

unlike an executor

An executor who renounces their right to apply for a grant of probate, does not automatically renounce their right to apply as administrator under NCPR 20/22 so they may may need to renounce both rights.

331
Q

administration - appoint attorney

A

before or after grant has been obtained

Afterthe administrator has been appointed under the grant: s.25 Trustee Act 1925 confirms that a PR may delegate their functions to an attorney for a maximum of 12 months. This can be renewed if needed. Notice should be given to the other administrators. Once appointed the attorney can carry out administrative steps on behalf of the donor.

Beforea grant has been obtained: in this case the applicant is delegating the power to apply for a grant, which is permitted under Rule 33 NCPR. The power of attorney must be provided to the probate registry as part of the application.

332
Q

PR unwilling/unable to act

A

An unwilling executor may renounce probate (unless they have intermeddled), have power reserved (provided another executor takes out the grant) or may appoint an attorney.

An executor who has power reserved may apply later to administer the estate under a grant of double probate.

An unwilling administrator may renounce their right to apply for a grant (even if they have intermeddled) or appoint an attorney. They cannot have power reserved.

A minor or someone who lacks capacity cannot be appointed as a PR.

Citations may be used to force an unwilling PR to act, remove their right to act, or authorise another to act in their place.

333
Q

Administration: Completing the application

A

Needed forall applications:
Online application or PA1A or PA1P
Application fee + £ per sealed grant

Needed forsome applications:
Certified copy of the death Certificate
Original Will / Codicil (if testator made one)
Form of Renunciation (if an executor is renouncing)
IHT 421 (if the estate is not excepted)
Affidavit evidence
Power of Attorney (if an attorney is applying)

334
Q

affidavit

A

An affidavit can sometimes be required when an application is made for the grant of representation.
Affidavits can be needed where there is evidence to suggest a problem with the validity or enforcement of a will or codicil.
An affidavit:
of due executioncan be used to confirm compliance with s 9 Wills Act 1837, that knowledge and approval were present, and the date on which the will was signed.
as to alterationscan be used to confirm the timing of alterations made to a will.
of plight and conditioncan be used to confirm the physical condition of the will at execution and after death.
of searchcan be used to confirm steps taken to locate missing documents.

335
Q

Administration: Collection of assets & payments of debts

A

PRs should collect in the assets using the method prescribed by the asset holder.
Estate cash should be held in a PR bank account and/or a law firm client account.
The PRs owe a duty to estate creditors to pay the deceased’s debts.
An estate is solvent where its value is sufficient to meet debts, liabilities and administration expenses (even if legacies cannot be paid in full).
Subject to a contrary intention in the will:
an asset subject to a charge bears the burden of the debt secured to it
the application of assets to meet debts/expenses is prescribed by statute.

336
Q

Administration: Income and Capital Gains Tax during the administration

A

PRs are required to finalise the IT and CGT for the deceased.
PRs will pay IT in respect of income generated during the administration and CGT in relation to gains made on the disposal of estate assets during the administration.
Death does not constitute a CGT disposal.
Transferring assets to a beneficiary does not constitute a CGT disposal.
Post death gains are taxed in the hands of PRs if assets are disposed by PRs during the administration or taxed in the hands of a beneficiary who disposes of the asset after it has been transferred to them by the PRs.

337
Q

Administration: Distribution & Estate Accounts

A

PRs must work out who should inherit the deceased’s assets and what they are entitled to receive by reference to the deceased’s will and/or intestacy rules.
PRs must establish whether there are sufficient funds to pay all the debts/expenses as well as the legacies. If funds are insufficient to meet all the legacies, the residue followed by the general legacies abate in priority to the specific legacies.
PRs have the power to decide which assets to appropriate in full or part satisfaction of an entitlement to a general or residuary legacy.
PRs should obtain receipt from the beneficiaries when making distributions.
PRs are subject to a statutory duty to keep estate accounts. Signing the estate accounts indicates the end of the administration.

338
Q

Order of payment of legacies

A

In general law, unless the will states otherwise, legacies are paid in the following order:
specific
general
residuary

If it is not possible to pay all of the legacies they abate (reduce) in reverse order. So:
- If funds are insufficient to pay all other legacies the residuary beneficiary takes no benefit.
- If funds are insufficient to pay all specific legacies, the general beneficiaries take no benefit.
- If there are sufficient funds to meet all specific gifts but not all general legacies, the general beneficiaries take a reduced inheritance. (Demonstrative legacies abate with general legacies once the specific fund set aside has been exhausted.)
- specific gifts take priority.

Within each category, if not all of the legacies can be paid, they abate proportionately.

339
Q

A woman died intestate two months ago and her estate was distributed between her three adult children. The woman had never married or entered a civil partnership. On the date of her death she had been living with her long-term partner and his son (aged 15) for the previous 10 months. The woman provided financial maintenance for both her partner and her step-son throughout the period they all lived together.

Who is entitled to bring a claim against the woman’s estate under the Inheritance (Provision for Family and Dependants) Act 1975?

The woman’s children and step-child.

The woman’s partner only.

The woman’s step-child only.

No one is entitled to bring a claim because the woman did not leave a will.

The woman’s partner, children and step-child.

A

Correct
Correct. The woman’s partner could apply as a person maintained at the date of death under s.1(1)(e). However, he could not apply as a cohabitee under s.1(1A), as they were not living together for the two years immediately preceding her death.

The woman’s step-child could possibly apply as a person treated as a child of the deceased under s.1(1)(d), or if the facts did not support this, then as a person being maintained under s.1(1)(e).

The woman’s children could apply under s.1(1)(c). That they are adult children and have received her estate under intestacy does not preclude a claim but would affect the likelihood of them receiving any award.

340
Q

A man died 8 months ago and the grant of probate was issued four months later. By his will the man left all of his estate to his son and nothing to his partner with whom he had been living for the last 3 years. The man’s partner wishes to bring a claim under the Inheritance (Provision for Family and Dependants) Act 1975 (the ‘Act’).

Is the man’s partner able to bring a claim under the Act?

Only if a court order granting permission for the deadline to be extended is obtained.

No - because they were not married to or in a civil partnership with the man.

No - because the man died more than 6 months ago.

Yes

No - because it has been less than 6 months since the date of the grant of probate.

A

Yes

Correct. The deadline for bringing a claim (without court permission) is 6 months from the date of the grant of representation. A claim can be brought under the Act before the grant is issued.

341
Q

A woman died recently. The woman’s will, which she executed last year, included the following gift, which she then altered with correction fluid a month after execution:

“I GIVE £ to my brother”

Prior to the alteration the gift read “£2,500” but, afterwards, the amount could not be seen at all.

Which of the following is correct?

If the executors submit extrinsic evidence of the original amount the man’s brother will receive £2,500.

The will is valid but the man’s brother receives nothing.

As the alteration was made after the will was executed the will is invalid.

If the executors submit an affidavit of due execution of the will the man’s brother will receive £2,500.

The executors can decide how much the man’s brother should receive.

A

The will is valid but the man’s brother receives nothing.

Correct. Any alteration (the use of correction fluid to cover up the amount of the gift) made after the will is executed is invalid unless the effect of the alteration is an “obliteration” (s 21 Wills Act 1837). As this amendment has completely covered up (obliterated) the number and it is not possible to read what the original gift was, the will is submitted to probate with a blank space and the man’s brother receives nothing.